Sie sind auf Seite 1von 184

Major Topic

Abnormalities of Teeth
Disesses

ofthe Blood

Abbreviation

Major Topic

Abbreviation

Abn ofTth

Odontogenic Tnmors

Odont Tum

Bld Dis

Pigmetrted Lsions of
the Orat Covitv

Pig Les of
Oral Cav

CoDnectiye Tissue Lesions

Con Tis Les

Pseudocyst

Pseudocysts

Itrfl rmmatory Jaw Lesions

Infl Jaw Les

Red-Blue Lesiors

R-B Lcsions

Metabolic & Genetic

M&G Jaw Dis

Jaw Diseases
Miscellaneous
Neoplasms

Nerve & Muscl

Misc.

Terms

Neo

SG Tum

Terms

UlcratiYe.Conditions

Nrv&Mus Disord Vesiculo-Bullous

Disorders
Non-Odontogenic Cysts

Salivary Ghnd
Tumors

Ulc Cond

V-B Dis

Diseases

Verrucal-Prpillary

N-O Cyst

V-P Les

Lesions
Non-Odontogenic Tumors
Odontogenic Cysts

N-O Tum

White Lesions

W Les

Odont Cyst

ORAL PATHOLOGY

Abn of Tth

The congenital absence of many, but not all, teeth is known as:

. H,?odontia

. Oligodontia
. Diphyodontia
. Anodontia

CopyriSht O 20ll'1012

' Denul

Decks

Tlr o forms of anodontia:

L Complete true: is a rare condition in which alt of the teeth are missing. It may involve both the primary and permanent dentitions. It is usually associated with hereditarv ectodermal dysplasia. See picture #l in booklet
:. Partial anodontia (commonly referred to as congenitally missing teeth): is rather
cor non. Teeth usually affected include the third molars (maxillary more often than
nandibular), maxillary lateral incisors ald mandibular second premolars. Note: As a
general rule, if only one or a few teeth are missing, the absent tooth will be the most
distal tooth ofany glen type (if molat then it would be the third molar). See picture
#2 in booklet

Other terms to b familiar with include:

. Ofigodontia: refers to the congenital absence of many (usually

six or more) but

not

all, teeth
. Hypodontia: refers to the absence ofonly a few teeth
. Diphyodontia: is having 2 successive sets of teeth (as in humans) as opposed to
pol)?hyodontia fraving more than two sets of teeth in a ldetime)
. Hypsodontia: having teeth with high crowns /cusps). Hypsodont dentition is associated with a diet ofabrasive foo<is

PATHOLOGY

Which type of dentinogensis imperfecta features multiple pulp exposures,


perirpical rtdiolucencies, and a variable radiographic appearance?

. Type I

'

Type

II

'

Type

III

CoDrighr C 20ll-201: - Denral Decks

The permanent maxillary centrals in the x-ray below are vital.


What is the most probabl interpretation ofthe condition illustrated?

. Mesiodens
. Concrescence

. Fusion
. Dens in Dente

Cop).righr C 201 l'201 2 - Denlal Decks

Dentinogenesis imprfecta is an autosomal dominant condition in which there is an intrinsic


alteration in the dentin. lt affects dentin ofboth the primary and the permanent dentition. It is
also known as hereditary opalescent dentin.
Dentinogenesis imperfecta has been divided into three types:
. Type I: dentin abnormality occurs in patients that have ostogenesis imperfecta fcra,'acterized blt blue sclera or a history of bone frqchu?s). In this form, primary teeth are more
severely affected than permanent teeth.
. Tlpe lI: most common, only the dentin abnormality exists with no bone involvement.

'Tlpe III

(Brandl'whe

\pe)

bke Type

lI, only the dentin abnormality exists, however,

there are clinical and radiographic vadations in this type. Features of type III that are not
seen in type I and ll include multiple pulp exposures, periapical radiolucencies, and a

variable radiographic appearance

Important: Clinically, dentinogenesis imperfecta is usually easily detected and identified.


The teeth exhibit an unusual translucent or opalescent appearance with color variation fiom
yellorv-brorvn to gray. The entire crown appears discolored owing to the abnormal underlying
dentin. See picture #3 in booklt
Other characteristics ofthe teeth include:
. Although the enamel is structurally and chemically normal, it fractures easily, resulting in
rapid tvear (due to poor dentin support)
. There is excessive constdction at the CEJ, giving the crowns a tulip or bell shape
. Short roots
. Obliterated pulps (\'pe I and I1 only) *** Note' In Type IlI, the dentin appean thin, and
the pulp chambers and root canal are extremely large, giving the appearance ofthin dentin
shells. See picture #4 in booklet
Treatment is full crown coverage for cosmetic purposes. Note: These teeth should not be used
as abutments, because the roots are prone to fractue under stress.

Mesiodens is the most common suprnumerary tooth, appearing singly or in pairs as a


small tooth with a cone-shaped crown and a short root between the maxillary central
incisors; it may be erupted, impacted or even inverted.

\Iesiodens appear situated in the maxilla near the midline and almost always posterior to
the normal central incisors. Many of them, therefore, are bypassed by the permanent
incisors $'hich are permitted to erupt into their normal position in the arch.
Remember:

. Fusion:

is a developmental union of two or more teeth in which the dentin and one
other dental tissue are united /rn ay be the root). See picture #6 in booklt
. Concrescence: is a condition in which only the cementum of two or more teeth becomes united. See picture #7 in booklet
. Dens in dente /a/so called dens invaginatus)r means "tooth within a tooth"' it is
caused by a deep invagination ofthe enamel organ during formation, most likely found
associated with a maxillary lateral incisor. See picture #5 in booklet

. Molars

.lncisors

. Premolars
. Canines

4
Cop)riglt

\-,

201l'2012 - Dnral Decks

An enamel defect resulting from the incomplete


formstion of the enamel matrix is ealld:

. Enamel pearls
. Enamel hypocalcifi cation
. Enamel hypoplasia
. Regional odontodysplasia

Coplright O 201 I -20

12

- Denral Decfts

Hypercementosis is often confined to the apical half of the root but, in some instances,
may involve the entire root. In large majority of instances, it affecls vital teeth, is not associated with any one particular systemic disease and may be regarded as a dental anomaly. It rnay be seen when a tooth has lost its antagonist or when there is chronic
inflammation of the tooth. The premolars are most frequently involved. Next in frequency are the first and second molars.
Hypercementosis produces no significant clinical signs or symptoms indicative of its
presence. It is seen radiographically as a bulbous enlargement that has surrounding it a
continuous and unbroken periodontal membrane space and a normal lamina dura. See

picture below.
Note: There is a form of hlpercementosis which is a common feature in Paget's disease
that involves thejaws. In addition to Paget's disease, hypercementosis is associated with
supraeruption, apical periodontal infection, occlusal trauma, toxic thyroid goiter,
acromegaly, and pituitary gigantism.

Example of gneral hypercementosis: Maxillary premolars

Enamel hypoplasia is a developmcntal def'ect in which the cnamel ofthe tccth is hard in contcxt but
thin and dcficient in amount. It results from incomplete formation ofthe enamel matrix with a
deficiency in the cementing substance. Enamel hypoplasia a1l'ects both the deciduous and pcnnanent teeth. It is usually due to illness or injury during tooth formation or due to a genetic disorder
Note: Thc gcnetic lbrms ofcnamel hypoplasia arc generally considcred to be types ofamelogenesis imperfecta.
The clinical appearance ofenamel hypoplasia includes: l) the lack ofcontact between teeth, 2) the
rapid breakdown ofocclusal surfaces, 3) a yellowish-brorvn stain that appears whcre the dentin is
exposed. Note: lfonly one permancnt tooth is affected, it is usually caused by physical damagc or

periapical infections to the primary tooth that this pernanent tooth replaced. This is known

as

"Turner's h-vpoplasia."
Remember: Enarncl hypocalcification is a hereditary dental dcfcct in which thc enamel is soft and
undercalcificd in context yct normal in quantity. It is caused by the dcfective maturation of
amcloblasts ftrere r.r a defect in the mineraliz tion ofthe fonned matrir). The teeth are chalky in
consistency, the surfaces wear down rapidly and a yellow to brown stain appears as the underlying
dentin is exposcd. This condition affects both the deciduous and permanent teeth as well.
See picture #9 in booklt
involves the hard tissues that are derived from both epithelial
(enarnel) an<l mesenchymal (dentin and cemenhnt) components of thc tooth-fonning apparatus.
Thc tccth in a region or quadrant of the maxilla or mandible are affccted to thc extent that they
exhibit short roots, open apical foramina' and enlarged pulp chambers' The thinness and poor
mineralization quality ofthe enamel and dentin layers havc given rise to thl; term "ghost teeth."
The permanent teeth are aflected more than the primary teeth , and the maxillary anteriol teeth are
affccted more than other teeth. The cause is unknown, although nume.ous etiologic factors have
been suggested /1.e., trauma, nutritional defcie cies, inlbcliotts). Because ofthe poor quality ofthe
affected teeth, their rcmoval is usually indicated.

\ote: Regional odontodysplasia

ORAL PATHOLOGY

Abn of Tth

Match the following alterations in tooth rnorphology on the lft


with the correct diagram depicting the alteration on the right.

. Fusion
. Twinning

Concresence

. Gemination

Copyng}r i)

l0lt :0ll

Dental De.ts

ORALPATHOLOGY

Abn of Tth

All of the following statements concerning amelogenesis inperfecta


rre true EXCEPT one. Which one is the EXCEPTIOM

.It

is an inherited condition which is transmitted as a dominant trait

. Because ofthe enamel malformation, the teeth ofindividuals with amelogenesis imperfecta are often discolored, sensitive to temperature changes, and painful to brush

. It only affects
. It

the permanent teeth

causes the enamel of the teeth to be soft and thin

. The teeth appear yellow. because the dentin is visible through the thin enamel
. The teeth are easily damaged and susceptible to decay
7
CopyriShr C

l0ll-:0l2

- Derral Decks

Remember:
. Gemination is a division of

a single tooth germ by invagination. This results in the incomplete tbrmation oftwo tceth. Frcquently occurs in the incisor region
. Fusion is an abnormally shaped tooth that may appcar as an extra wide crown, a normal crown rvrth
an extra root. or other combinations fesulting from the union of two adjacent tooth gcrms by dentin
during development.
. Concresence is the union ofthe roots of two or more normal teeth caused by con'flucnce of their

cemental surfaces.

. Trvinning

is the completc division of a single tooth bud. The dividert teeth are secn as com-

pletelySepalatewithnoconnectiontoeachotherexcepteachtendstobemirrorimageofthe
other
. Ank:-losis is the fusion of the cementum or dentin to the surrounding alveolar bone after loss
oflhe intervcning periodontal membrane lt is associated \Pith hypodontia The most commonly ankylosed tooth is the pfimary second molar; the pemanent second premolar is the tooth that fails to de\ elop and crupt. Clinically, percussion ofthe anlllosed tooth produces a drrll sound
\ote: There is a change in the continuity ofthe occlusal plane which is caused by the continued eruprion ofthe non-ankylosed teeth and the growth ofthe alveolar process
clinicallJ,, gemination and fusion can look similar. when counting these tceth, the "larger" tooth should
It cc)unred as a single tooth. ln gemination, the patient has a larger tooth but a normal numbel ofteeth
o\ erall. In fusion, the patienl has a larger tooth, but one lcss tooth than normal

"

-{melogenesis imp erlecta (AI) is a relatively rare group ofinherited disorders characterized
bl abnormal enamel formation. The term amelogenesis imperfecta is reserved for hereditary
defects ofenamel that are not associated with defects in other parts ofthe body or other health
problems. lt is a hereditary ectodrmal defect, unlike dentinogenesis imperfecta which is a
hereditary rnesodemral defect.

AI enamel defects are highly variable and include abnomralities that are classified as hypoplastic defect in amount ofenamel),hypomaturatron (defect in fnal grov'tlt and mqhration
ol'euottel crystallites), andhypocalcified (defect in initial crystallite Jbnnation followed by dete. ti\ e gt'ovlh). The enamel in both the hypomaturation and hypocalcified Al rypes is not mineralized to the level of normal enamel and can be described as hypomineralized Al can be
intrerited as an x-linked, autosomal recessive (,4RJ, or autosomal dominant (AD) cor'ditiorr.
The

The color ofthe enamel ranges from white opaque to yellow to brown, it is reduced in volume
and pitled. Contact points between teeth are often open and occlusal surfaces and incisal edges

frequently are severely abraded. Although the enamel are soft and irregular, there is no increase jn caries mte. Note: Open bite is a common clinical finding.
The radiographic findings are fiequently distinctive and pathognomonic. when the enamel
is totally absent, the radiographic appearance makes the diagnosis obvious. When some enamel
is present, thin radiopaque coverings on the proximal surfaces ofthe teeth are noted. When the
anatonic crown forms are normal or nearly normal, the softness ofthe defective enamel may
not be easily distinguished from the dentin. ln all cases, howevel the dentin' pulp and cementum are unaffected by the disease process itself(arlr*e dentinogenesis inperkdq).
See

picture #8 and #10 in booklet

Exception: Amelogenesis imperfecta will only show pulp obliteration


abrasion with secondary dentin formation.
Other than cosmetic restoration, no treatment is necessary.

if

there is advanced

. Erosion
. Abfraction
. Attrition

. Abrasion

8
Cop).dght O 20ll-2012 - Denhl Decks

. Dentinogenesis imperfecta

. Erythroblastosis fetalis

. Porphyria
. Fluorosis
. Diabetes mellitus

. Pulpal injury
. Intemal resorption
. Tetracyclines
Coplrighr O

201

9
l'2012 - Dental Dcks

See

picture #13 in booklet

Types ofabrasion:

l. Toothbrush abrasion: most olten results in V-shaped wedges at the cervical margin
in the canine and premolar areas. lt is caused by the use ofa hard toothbrush and/or
a horizontal brushing stroke and/or a gritty dentifrice.
2. Occlusal abrasion: results in flattened cusps on all posterior teeth and wom incisal
edges. lt results from the chewing or biting ofhard foods or objects and chewing tobacco.

. Attrition is the wearing away of enamel and dentin due to the normal function or most
commonly, due to the excessive grinding or gdtting together oftecth by the patient lreJbrced
to a.r bruxism). The most noticeable effects ofattrition are polished facets, flat incisal edges.
discolored surfaces ofthe teeth and exposed dentin. Facets usually develop on the linguoincisal of the maxillary central incisors, the facioincisal of the mandibular canines and the
linguoincisal ofthe maxillary canines. See picture #14 in booklet

. Erosion

is the loss of tooth structure from non-mechanical means. It can result from
drinking acidic liquids or eating acidic foods. It is common in bulimic individuals as a result of regurgitated stomach acids. It affects smooth and occlusal surlaces.
See picture #12 in booklet

. Abfraction lesions are cervical erosive lesions that can not be attributed to any particular cause: causing the enamel to "pop'' off starting at the base ofthe tooth and exposing
the gum Iine ofthe tooth to excessive wear See picture #11 in booklet

Intrinsic Stain

Cause
Dentinogenesis imperfecta

Translucent or opalesccnt hue; usually gray to bluish-brown

Erv1hroblastosis fetalis

Bluish-black, greenish-bluc, tan or brown

Poryhyria

Red or brownish

Fluorosis

white opacities or light brown to brownish-black

Pulpal injury

Starts pink and usually becomes orangc-brown to bluish-black

Intcrnal resorption

Pinkish

Tetracvcline

Varies from light yel)ow-orangc to dark gray-brown; depending


on the tlDe oftetracycline used and the duration oltherapy

H1

pomineralized enamel

White to brown

E\trinsic stains can be caused by bacteria, iron, tobacco, foods, beverages, gingival hemorhage. restorative materials and medications.
Treatment:

. lIost

extrinsic stains can be removed with abrasives; reduce or eliminate cause

of

stain

. Intrinsic stains may

need bleaching, esthetic restorations, or prosthetic rehabilitation

ORAL PATHOLOGY

Abn of Tth

What is th most probable diagnosis of the x-ray below?

10

Coptright

(l

20l

l-:0 l 2'

Denral Decks

ORAL PATHOLOGY

Abn of Tth

A patient with which typ of amelogenesis imperfecta will have teeth


that demonstrate enamel that varies from thin and smooth to normal
thickness with grooves, furrows and/or pits?

. Type | (H.vpoplastic)
.

Ty-pe

II (H|pomqturation)

. Type III (Itvpocalcified)

11

CopyriShr C

20ll'101:

- Denral Dccks

Intrnal resorption is an unusual form oftooth resorption that begins centally within the
tooth. Resorption ofthc dentin ofthe pulpal walls may be seen as part ofan inflarnmatory
response to pulpal injury, or it may be seen in cases in which no apparent trigger can be
idcntificd. Most cases of intemal resorption present no early clinical symptoms. The first evidence of th lesion may bc thc appcarancc ofa pink-hued area on the crown of thc tooth,
which reprcsents the hyperplastic, vascular pulp tissuc filling thc rcsorbed areas. Ifthe condition is discovered before perforation ofthe crown or root has occured, endodontic therapy may be canied out with thc cxpectation ofa pretty high success rate.
Idiopathic external resorptive lsions arc charactcrized by thc invasion ofthe ccrvical rcgion ofthe root by fibrovascular tissue which progressively resorbs dentin, enamcl and cementum. Thc sourcc of the resorption is in thc attachmcnt apparatus. This is the key to
understanding diagnosis and treatment. The dental pulp remains protected by an intact layer
of dentin and predentin until late in the process. Since the source of the lesion is not in the
pulp, endodontic treatment by itselfis ofno assistance in dealing with the sourcc ofthe problcm.

Arrow points to external root


resorption of the maxillary
left cntral incisor

Crown size varies from


Varies fiom thin and smooth to
small to normal. small teelh normal rhickness with grooves,
nay Iack proximal confirrrows andlor pits
lacts, color varies from normal to opaque white-ycllow
Varies flom creamy opaque Normal thickness with enamel
to marked yello\r',/brown.
that ofien chips and ab8des
surfacc ofteeth sofl and
easily
rough. dental sensitivity
and open bite are common
Opaque nhite to yellowbrow_n. soft rough enamel
surface. dental sensiti.i,ity
and open bite are common.
hea\,1 calculus formation is

See

Normal thickness with enamel


that ofien chips and abiades
easily

Enamel has normal to


slightiy reduced contrast

Enamel has contrast

similar to or less than


clcntrn. uncrupted crowns

have normal morphologv


Enamel has contrast

similar to or less than


dentln, unerupted crowns
have normal morphology

pictures #15, #16, and # l7 in booktet

Remember:
L Amelogenesis imperfecta is a hereditary ectodermal defect, unlike dentinogenesis
imper_
fecta which is a hereditary mesodermal defect.
f. In all three types ofame)ogenesis imperfecta, the dentin, pulp and cementum are unaffected by the disease process itself /anlte dentinogenesis imperfecta).

. Normal

. Somewhat smaller
. Extremely large

. Completely obliterated

't2
Cop)right O 201l-2012 - Denral Decks

Leukemi! is a group ofbone mrrrow diset$es


involving an uncontrolled incrase ln:

. Red blood cells (Erythocytes)


. Platelets

Plasma cells

. White blood cells (Leukocytes)

13
Coplri8hr O

201 |

'2012 - Dnral Decks

Dentin dysplasia is another autosomal dominant trait that affects dentin. AII teeth ofboth
dentitions are affected. This condition has not been associated with any systemic connective tissue disorder. This is a rare condition that has been subdivided into type I or
radicular type and a more rare type II or coronal type:

. Type | (radicular

cl.,-.splasia): more common type

. Both dentitions

are normal in color and shape


. The teeth are generally mobile, frequently abscess and can be lost prematurely
. Teeth show greater resistance to caries than do normal teeth
Radiographic features:
. Extremely short roots
. Obliterated pulp chambers and root canals before eruption
. Residual fragments of pulp tissue appear typically as horizontal lucencies
(

cnevrons)

. Periapical radiolucencies (granulomo,s or


. Type

Il

c.vst-r')

around the defective roots

(coronal dy.splasia)

. Color of primary teeth is opalescent


. Color of permanent teeth is normal
. Coronal pulps ofpermanent teeth are
lilled rvith globules of abnormal dentin

r"rsually enlarged ("thi,\tle

lube") a

J arc

Radiographic features:
Deciduous teeth are similar in appearance to type I, but permanent teeth exhibit
enlarged pulp chambers ("thistle nbe") in appearance
. .\bsence of neriaoical radiolucencie"

is a lomr ofcancer that begins jn the blood-forming cells ofthe bone marrow (the soft,
innet part o.l'1he botret. Undcr nomal circumstances, the blood-fonning, or hematopoictic, cclls
of thc bone marrow make leukocytes to defend thc body against infectioLls organlsms such as
r inrses and bacteria. But il'sot'ne leukocytes are damaged and remain in an inmlature fbrm' they
becornc poor int'ection fighters that multiply excessivcly and do not dic olfas thcy should.

Leukemia

The leukcmic cells accumulate and lesscn the production of oxygcn-carrying red blood cclls /eryrlr r or r les/. blood-clo tting cells (plaletets), and normal lcukocytes. If untrcated, the surplus leukemic
cells or er$helm the bone marrow, enter the bloodstream. and eventually invadc other parts ofthe
btrdr. such as the lymph nodes, spleen, livcr, and central nervous syslern (brcin, "^Pfualcoldr' In this
* ar. the bchar ior of leukcmia is different than that of other canccrs, which usually bcgin in major
organs and ultimatcl), spread to thc bone marrow.

Thcrc are more than a dozen varictics ofleukenia, but thc follorving four types arc thc most comnlon:

L -\cute lymphocytic lcnkcrlta (ALL). most common type in children


le]rlemia (AML.): most malignant type
j. Chronic lymphocytic lcukernia fczlr: least malignant type
-1. Chronic myclogenous leukcmia (CML)t 2 distinct phases, invariably latal
1. .\cute myelogenous

lmportant:
. Leukemia is classified by thc dominant cell type
. Leukemia can modify thc inflammalory reaction

and by the

duration from onset to death

. Leukemia's affect on teeth/gingiva: Duc to a decrease in immune responsc, periodontal problenrs can be aggravated. Gingival enlargement can be a finding rvith leukemia Additionally,
deficits with platelets can bc scen intraorally with increased hemorrhaging,/bruising."

BId Dis

A 48 year old female patient walks into your office. She states that she is
diagnosed with some disease which she can't remember the name of. Her
physician wants her to follow up with you, her dentist, regularly to watch
out for cancer of the tongue and throat. She also has a bald tongue, and
states that her fingernails "look funny." What disease does she have?

. Aplastic anemia
. Plummer-Vinson syndrome
. Pernicious anemia
. Cushing's syndrome

14
Copvrighr ali 2011'201: - DenralDecks

ORAL PATHOLOGY

Btd Dis

What disorder is a result of a genetic mutation causing the


substitution of glutamic acid by a valine and results in
dental radiographs with enlarged marrow spaces?

. Cystic fibrosis
. Muscular dystrophy
. Polio

. Sickle-cell anemia

15

Copvrigl'r

aC

201l'2012

' DenlalDecks

Plummer-Vinson syndrome characterized by an iron-deficiency anemia, atrophic


changes in the buccal, glossopharyngeal, and esophageal mucous metnbranes, koilonycha
(spoon-shaped./inger rail.s), and dysphagia. The dysphagia is due to an esophageal stricture or web. SCC of the tongue and throat are complications. lt is most common in middle aged women, rarely in the male. The etiology unknown. Because ofthe predisposition
to the development ofcarcinoma ofthe oral mucous membranes, it is essential that the di-

agnosis be established early so that treatment can be given ASAP This includes administration of iron, vitamin B complex and a high protein diet.

Aplastic anemia is a form of anemia in which the capacity of the bone marrow to generate red blood cells is defective. Two types:

l. Primary: unknown cause, affects young adults. The signs and symptoms include
pallor, weakness, malaise, dyspnea (dilfcul4, breathing), headache and vertigo. Oral
symptoms include spontaneous bleeding,bnstsing (petechlaeT and gingival infections.

It

is usually fatal.
2. Secondary: caused by exposure to toxic agents, such as radiation, chemicals or
drugs (fbr exanple, chloramphenicol). It can occur at any age. Symptoms are the same
as primary. Prognosis is good once you remove the cause.

*** Aplastic anemia is the most serious and life-threatening blood dyscrasia associated
s ith drus toxiciw.

Sickle-cell anemia is an inherited disease in which the red blood cells, normally discshaped, become crescent shaped, As a result, they function abnomally and cause small

blood clots. These clots give rise to recurrent painful episodes called "sickle cell pain
crises." Sickle-cell anemia (also called sickle-cell disease) is the result ofthe production
ofabnormal hemoglobin (Hemoglobin S/ due to a genetic defect. It is carried as a trait
by 10% of African Americans and 0.2olo have sickle-cell anemia. lt is more common in females and usually clinically manifests itselfbefore the age of30. The typical signs ofanemia are present. The patient is weak, short ofbreath and easily fatigued. Muscle and joint
patns are common.

Denlal radiographs ar oftn of diagnostic value: marrow spaces are markedly enlarged because ofthe loss ofmany trabeculae; the trabeculae, which are present, are often
abnormally prominent. Occasionally, osteosclerotic areas are noted in the midst of large
radiolucent marow spaces. However, the lamina dura and the teeth are unaffected.
See picture #18 in booklet

\ote: The gene defect is a known mutation ofa single nucleotlde (th),minelbr an adenine)
ofthe beta-globin gene, which results in glutamic acid to be substituted by valine. Sicklecell anemia occurs when a person inherits two abnormal ger'es (one front each parent). lf
a person inherits one abnormal gene for the disease, they have what is called sickle cell
trait. The life span ofred blood cells is reduced from 120 to 20 days.

ORAL PATHOLOGY

Bld Dis

The category of pigmented (usually purple) lesions in the skin caused by


xtravasation ofblood from the capillaries is known as:

Petechiae

. Ecchymosis
. Varicose veins

16

Copyrighr al 2011-2012 - Dental Decks

ORAL PATHOLOGY

Bld Dis

All of the following statements are true EXCEPZ one.


which one is the rxcEPZl0M

.
.

Acute leukemias have a slow onset and progression


Acute leukemias are characterized by the appearance of immature, abnormal cells in
the bone marrow and peripheral blood and frequently in the liver, spleen, lymph
nodes, and other parenchymatous organs

The clinical picture of acute leukemias are marked by the effects ol anemia, which is
usually severe (/ittigue, molaise), an absence of functioning granulocles (proneness
to inlbction and inflammation), and thrombocytopenia (hemorrlrugic diathesis)

The spleen and liver usually are moderately enlarged, while enlarged lymph nodes
are seen mainly in acute lymphocytic leukemia. Fever and a very high ESR are
found

Leukocyte counts vary greatly in the acute leukemias


17
Copyrighr /(, 2011'2011 Dental Decks

Purpura spots (pinpoint spots) are purplish discolorations in the skin produced by small
bleeding vessels near the surlace ofthe skin. Purpura may also occur in the mucous membranes (such as the lining oJ the mouth) and in the intemal organs. Purpura by itselfis only
a sign of other underlying causes of bleeding. Purpura may occur with either normal
platefet counts hrcnlhrombocftopenic purpuras) or decreased platelet counts (thromboq)topenic purpltra.rr. Platelets help maintain the integrity of the capillary lining and are
irnportant in the clotting process. Note: Large purpura spots (> 3 mn) are called ecchymoses.

Major kinds of purpura:


. Thrombocytopenic purpura (Werlhol's tlisease):

a bleeding disorder characterized

by a deficiency in the number olplatelets. This resr.rlts in multiple bruises, petechiae,


and hemonhage into the tissues
. Thrombotic thrombocytopenic purpura (TTP): a severe and frequently fatal form
characterized by a low platelet count in the blood due to consumption of platelets by
thrombosis in the terminal arterioles and capillaries ofmany organs

Oral manifestations of thrombocytopenic purpura:


. Severe and profuse gingival hemorrhage
. Petechiae occur commonly on the palate

Important: Tooth extractions are contraindicated due to the tendency for excessive
bleedin s.

***

This is false; acute leukemias have a rapid onst and progression.

-{cute leukemia is characterized by malignant proliferation olwhite blood cell precursors


I hle.\ts) in bone marrow and lymph tissue and their accumulation
in peripheral blood,
bone rnanow, and body tissues. Leukemic cells inhjbit normal bone marrow production
rrf en throcytes. platelets, and immune function.
Orher important features ofacute leukemia:
. -A.brupt onset (fe11,noltlsl with sudden high fever, weakness, malaise, severe anemia,
and
lymphadenopathy; bone and joint pain common in children.
-ueneralized
. Principal organ involved: bone marrow (along witlt the spleen and liver)
. Petechiae and ecchymoses in skin and mucous membranes, hemorrhage from various sites: bacterial infections common.
. Laboratory findings: leukocytosis 30,000-1000,000 per cu.mm. with immature forms
r nn eloblasts and 1.,-mphoblasts) pred,ominating.
. ln 7-i9i ofthe cases olacute lymphocytic leukemia, the lymphocytes are neither B nor
T-cells and are called "null cells. "
. L ntreated patients die within six months; with intensive therapy (chemotherap,-,
rutliation, and bone marrow transplanls) remissions lasting up to five years may be
obtained; death is usually due to a hemorrhage (broin) or a superimposed bacterial

infection.

. Thromboc)'topenic purpura

. Agranuloc).tosis

. Sickle-cell

anemia

. Peutz-Jeghers syndrome

18
Copynghr @ 2011,2012, Dental Decks

The term used to describe a leukemia where leukmic cells rppear in the blood
but there is no significant lncrease in the number of white blood cells is callod: .,

. Aleukemic leukemia

. Subleukemic leukemia

Stem cell leukemia

't9
CopFight O 20ll-2012 - Dental Dects

Agranulocytosis is an abnormal condition of the blood, characterized by a severe


reduction in the number ofgranulocytes (particL arly neutroprilr./. Note: It may also
be caused by the antithyroid drugs fi.e., propylthiouracil, methimazole, qnd carbimazole).

Clinical features:

. Sudden onset of high feveq chills, jaundice, weakness


. Oral infection with rapid periodontal destruction
. Oral ulcers and gingival bleeding

and sore throat

The most characteristic feature of this condition is the presence of infection, particularly in the oral cavity. The signs and symptoms develop very rapidly, usually within a
fev days. and death may occur soon afterward.
The oral lesions are an important phase ofthe clinical aspects ofagranulocytosis. They appear as necrotizing ulcerations of the oral mucosa, particularly the gingiva and palate.
These lesions appear as ragged necrotic ulcers covered by a gray membrane. One imporranr aspect is that there is littl or no apparent inflammatory cell infiltration around the

lesions. Histologically, this is pathognomonic of agranulocytosis.

\ote: Cl clic nutropenia

is an unusual form ofagranulocytosis. These patients typically


erhibit severe gingivitis. The severe ulcerations usually seen in agranulocytosis usually
dLi not occlrr. See picture #19 in booklt

Other leukemia terms to know:

. ".A.leukemic" leukemia is a term used to describe a lorm of leukemia in which there


are leukemic cells present in the bone maffow, but the circulating white blood cells are
nither immature nor increased in number.
. Stem cell leukemia is a form of leukemia that is characterized by abnormal cells that
are poorly differentiated but are considered to be precursors of lymphoblasts, myeloblasts. or monoblasts. Note: these cells are too immature to classify.

. A "leukemoid" reaction is a term used to describe a marked increase in the number


of circulating granulocytes. This condition is seen in a variety of disorders inch.rding
cluonic infections and neoolasms.

. Denuded gingiva
. Glossitis
. Edematous buccal mucosa
.

Severe gingivitis

20
Cop),righr O 201 l-2012 - Dental Dcks

In your ofllce, you see a 6 month old child whose lirst teeth are eruptlng and
whose mother iJ concrned about the color. The mandibular incboB do show
a brownlsh-blue hue, You are golng to ask the mother about which
of the following conditions during her pregnancy:

. Sickle-cell

anemia

. Erythroblastosis fetalis
. Patent ductus arteriosus

. Low-weight preterm birth

21

CopFiglr O 20ll-2012 - Dental Decks

Pernicious anemia is a relatively common, chronic, progressive, megaloblastic anemia.


It is caused by the lack of secretion of the intrinsic factor in normal gastric juice. This
lactor is necessary lor adequate absorption of vitamin 81.,, which is necessary for the
maturation of erythrocytes. As a result, they produce fewer erythrocytes than normal.
The onset of pemicious anemia usually is insidious and vague. As the condition progresses, there will be a sore, painful ton g.ue (atrophic glossilr.r/, angular cheilitis, a tingling
numbness of the extremities. difficulty swallowing (d,,-sphagia), painful swallowing
(od,vnophagia). See picture #20 in booklet

A Schilling 24-hour urine test is done to evaluate whether vitamin B 1.' is being absorbed
by the body and is most commonly used to evaluate patients for pemicious anemia.
Remember: Thalassemia major and minor are hernolytic anmias that result from a genetic defect. Both are characterized bv a low level of ervthrocvtes and abnomal hemoglobin.

Oral manifestat ions of thalassemia:


. Oral mucosa may exhibit the characteristic anemic pallor
. Flaring of the rnaxillary anterior teeth with malocclusion

The fetus'blood is Rh-positive because the father passed along an Rh-positive trait. rvhich
is a dominant trait. The mother responds to the incompatible blood by producing antibodies against it. Thesc antibodies cross thc placenta into the fetus'circulation. whcrc thcy af

tach to and destroy thc fctus' red blood cells, leading to anenria
er.r

throblastosis fetalis.

\ote: It can also rcsult from blood typc

this is called

incompatibilities. For cxample, the mothcr may

har e ty'pc O blood and thc fetus has type A or B blood.

The most common form oferythroblastosis fctalis is called is ABO incompatibility, which
can ran in its severity. The less common form is called Rh incompatibility, which more
ottcn causes a very scvere anemia in the baby. The severity olthis condition can vary widcly.
In some instanccs, the baby has no symptoms of the disease. In othcr cases, it can lcad to
death of thc baby beforc or shortly after birth. lt can be treatcd in utero by intrauterine transt usion.

\\'hcn thc child is bom, signs may include an enlarged liver or spleen, generalizcd cdcma,
laundice. and anemia. After birth, depending on the severity, a transfusion usually nccds to
be performed. The most severc form of this disease, Rh incompatibility, can bc prcvcntcd
if thc mothcr takes a medicinc called Rhogam at certain times during and aftcr prcgnancy.

Oral manifestations of crythroblastosis fetalis:


. Teeth appear to havc a grccn, blue or brown hue due to thc deposition ofblood pigment
in the enamel and dentin.
. Enamel hypoplasia may occur. If it does, it affects thc incisal edges of the anterior
teeth and the middle portion ofthe deciduous cuspid and the first molar crown.

They have a rapid onset and progressron


They have a shorter, more devastating clinical course than the acute leukemias
They are characterized by proliferations of lymphoid or hematopoietic cells that are
more mature than those ofthe acute leukemias

They constitute

7 5o/o

of all leukemias

22
Copyright O 201l-2012 - Dental Deck

. Acute lymphocytic leukemia (ALL)

. Acute myeloid leukemia (AML)

. Chronic lymphocytic letkemia (CLL)


. Acute monoblastic

23
Coplright

20ll-2012,

Dnral Decks

Important: Chronic leukemias have a slower onset and progression. They also have a
longer, less devastating clinical course than the acute leukemias, and they constitute
about 507o of all leukemias.
Other important clinical features of chronic leukemias:
.Insidious onset witl't weakness and weight loss: disease may be detected during
exanination for some other condition (e.g., anemia, unexplained hemorrhages, or
rccw're nt intrqctab le inlbctio t1 )
. Organ involvement similar to acute type: massive splenomegaly is characteristic
of chronic myelogenous leukemia; lymph node enlargement is main pathologic finding in lymphocytic tyPe
lymph. Petechiae and ecchymoses' recurrent hemorhages, bactedal infections
anemia
hemolytic
by
autoimmune
complicated
ocytic anemia may be
. Laboratory findings: leukocylosis above 100,000 per cu. mm. with matur forms
(gronulocltes ancl lymphocytes) predominating; Philadlphia chromosome and low
levels of leukocyte alkaline phosphatase are corunon in chronic myeloid lukemia
(c.\'tL)
. \'ledian survival time for patients with chronic myelogenous leukemia /CMI/ is lour
(CIl)
l ears rvith death <lue to hemorrhage or infection; chronic lymphocytic leukemia
treatment'
years
without
even
patients
may
survive
older
runs a variable course;

The peak age for ALL is around four years old, and it is the form of actte leukemia that
is the most responsive to therapy. Cunent therapies for ALL include chemotherapy with
lbllo\1.up radiation, and possible bone-marrow transplant after particularly high dose
chemotherapy treatment or in cases ofrecurrence or ilnon-responsive to other treatments.

***

Acute myeloid leukemia (AML) and chronic lymphocytic leukemia (CLL) arc the

most common types in adults. See picture #22 in booklet

Allhough the exact cause ofmost leukemias remains unklown, increasing evidence suggests a combination ofcontributing factors. These factors include: familial tendency, congenital disorders (Dov,n syndrome, or the presence of Philadelphia chromosome
chronic myeloid leukemia), viruses (e.g., HTLVI, herpes-like viral particle,s lnve been
c ulared.fion potients ancl leukemic patients have high ontibod), titer to the Epstein-Barr
lirr-st. ionizing radiation and the exposure to the chemical benzene and cytotoxins such
as alk) lating agents.

Important: Oral lesions are most likely to be observed in myelogenous leukemia. These
oral lesions may be the initial manifestation ofthe disease. The oral lesions include gingivitis, gingival hemorrhage, generalized gingival hyperplasia, petechiae, ecchymoses.
and ulcerations. See picture #21 in booklet

Bld Dis

You have a new patient in your dental oflice who has just moved
from Denver. He srys his doctor told him that he has some disease
caused by living at a high altitude. When conducting an intraoral xam,
you lind that his tongue is a deep purple and his gingiva bleed easily.
What disease is a likely cause ofthese findings?

. Polycythemia Vera
. Hemophilia B
. Thallesemia Major

. Porphyria

24
Coplrighl

e 20ll':012 '

Denral Decks

ORAL PATHOLOGY

Bld Dis

The translocation from chromosome 22 to chromosome 9


is a finding of which leukemia?

Acute lymphocyttc Ierkemia (ALL)

. Chronic myeloid leukemia (CMZJ

. Acute myeloid

leukerma (AML)

. Chronic lymphocytic leukemia (CLL)

25
Copright

ae

20ll l0ll

DenklDeck!

Polycythemia is the condition oftoo many red blood cells in the circulation. The blood
can be too thick to pass easily through the small blood vessels ofthe body. This in tum
leads to clot formation and blockage ofthe small vessels which can lead to a stroke
There are two types ofpolycythemia:
I . Primary polycythemia (also called polycythemia vera or e4lhemia) occurs when
excess erythrocytes are produced as a result ofturnorous abnormalities. This occurs in
the tissues that produce blood cells. Usually accompanied by leukocytosis. Splenomegaly, as a result ofvascular congestion, is seen in 75% ofpatients.
2. Secondary polycythemia is an increase in the total number of erytluocytes due to
another condition. For example. chronic tissue hypoxia of advanced pulmonary disease, high altitude fo.r ler's disease) or the secretion oferythropoietins by certain tumors.

Oral manifestations of polycythemia:

. Oral

mucous membranes (especialb tlrc gingiva and tongue) appear deep purplish-

red.

. The gingiva are very swollen and bleed very easily.


. Submucosal petechiae (purplish spots), ecchymoses (same
and hematomas are common.

as

petechiae, but bigger)

is a rare syndrome of paroxysmal vasodilation with buming


oain. incrcascd skin tcmpcrature, and redncss ofthc feet and, less often, thc hands.

\ote: Erl-thromelalgia

\lmost 90% of patients with chronic myeloid (mteloctttic, n|'elogenous, granuloc,-tic)


leukemia have the philadelphia chromosome, an abnormality in which the long am of
chromosome 22 is translocated. usually to chromosome 9. Radiation and carcinogenic
chemicals may induce this chromosomal abnormaliry

C\IL is

characterized by the abnormal overgrowth ol granulocytic precursors


rn^elctblqsts and promvelocvtes) in bone marrow, peripheral blood and body tissues.
C\tL is most common in young and middle-aged adults and is slightly more common in
men than in women: it is rare in children.
The npical symptoms of

CML include:
. spongy bleeding gums
. t'atigue
. fer er
. \\ eight loss

. moderate splenomegaly
. joint and bone pain
. repeated infections

\ote: Acute myeloid leukemia (AML) rs a malignant disease of the bone marrow in
s hich hematopoietic precursors are arrested in an early stage of development. AML is
distinguished ftom other related blood disorders by the presence of greater than 30%
blasts in the blood and/or bone rnarrow. These blasts /m|eloblasts) contain Luer rods in
therr cvtoDlasm.

Which of the following is { troublesome librobLstic


neoplssm that is locally sggrer$ive rnd infiltrrtive?

. Peripheral fibroma

. Traumatic neuroma
. Nodular fasciitis
. Fibromatosis

26
Copynghr Q 20ll-2012 - Dntal

Deck

While in the OR on a general surgery rotation, a 3 month old is brought in with


alarge (20cm) flaid-lilled mass on her neck. The diagnosis is r cystic hygroma
(hygroma coli).This lesion is under whlch umbrella oflesions, which also
' contdns enlarged tissue on the postcrior rnd latral border of the tongoe?

. Angiomas
. Lymphangiomas
. Schwanaomas
. Fibrosarcomas

27
Coplright O 201l-2012 - Denlal Decks

Trcatmenl and Prognosis

Clinical Charictenstics

Etiolog)

Tumor

Most common sit ovI

mentll

foramen m edentulous moufts;

[xcision wjth snall proximal


podion ofinvohed neNei

nodule or swelling, wbich may


be painful to digilal pressure
Encapsulated mass lhal prese.ts
a5 an

Consenati!e e{cisioni

a*mptomatic lump. Tbe

longue ls the most common


location.

siy it

is denved from

1.

Solitlry nelrollbromr

2.

iltultiple

asympromaric ndule, occurs on


tongue. buccal mucosa and
lesions as pad

L Soli(ary: surSical excision


2. Neurofibromatosis:
removal is impractical. wrtch
lbr hiSh rate ofmalignant

oflhe

sFdrome neurofi bromatosis


P.esms as {irrn mass; xhibits
rapid gowlh; pain and

Su.gical excivont rccuncncc

Tbe mandible and condguous


soi lissues are most frqunlly

Aggressive sursical approach is


recommended. Rccurrence's

involved intrno.ally. LocaLly


aggressive and infi ltrarive.

\ traumatic ncuroma is a lesion causcd by lrauma to the periphcral ncnc. In thc oral catity, thc injury rnay be nr
rhr rirmr olrrauma fiom u surgical procedure such as a toolh cxlraclion, from a local ancslhctic injcclion or fionl an
.r..ideni. It is usually a vcry small nodulc 1/.\'r /,{utt 0.5 .r in didnrcter).In thc oral cavity it is most commonly sccn
.! rIc mentat foramen. Il is firm, movable ard wcll cncapsulatcd. ll is painfut when pslpaled. Prcssurc applicd to
rhr naurona clicits a resporsc oftcn dcscribcd as an "electric shock".
Hi!tologic features:
. \bundant ncnc lissuc a'rd collagcnous fibrous tissuc in haphazard arangcnrcri
. Chronic lillammalory ccll infilirate may be prcscnt
. Sch\ ann cclls rri1l bc prcscnt

mphangiomas are benign bamartomas of lymphatic channels that develop early in life
$ ith no sex predilection. They may occur on the skin or mucous membrane. ln addition to
the tongue, they occur commonly on the lips and labial mucosa.
Ly

Clinical features:
. Raised, diffuse. bubbly nodules or vesicles
. Range in color from clear to pink. dark red, brown or black

'

.As)'mptomatic

. Soft. fhrctuant
. \ aries in size
. Usuall.v painless
Histologic features: four types of lymphangiomas:
. Ll mphangioma simplex (capillory lymphangiono)

composed of small, thin-walled

lvmphatics
. Calernous lymphangioma - comprised ofdilated lymphatic vessels with surrounding adventitia
. cl stic lymphangiom? (cyslic hlgroma or hygroma col,
- consisting ofhuge, macroscopic lymphatic spaces with surounding fibrovascular tissues and smooth muscle
.Benign lymphangioendothelioma facquired progressive lltnphangiont) lymphatic
channels appear to be dissecting through dense collagenous bundles

Important: Lymphangiomas do not undergo malignant change. Some lymphangiomas,

es-

pecially congenital types, regress spontaneously during childhood. Aspiration is manda-

tory before surgical excision ola lyn.rphangioma to prevent complications associated with
the similar-appearing hemangioma

. Lipoma
. Pyogenic granuloma

. Epulis granulomatosum
. Peripheral fibroma

28
Coplright O 20ll-201: - Dental Decks

A patint presents to your clinic with multiple exophytic masses covering


the buccal mucosa, tongue rnd lips. A biopsy reveals that these are mucosrl
neuromas. The most importrnt rerson this patient should be referred to a
physician is bectuse ofthe risk of related:

. Squamous cell carcinoma ofthe tongue


. Pituitary hyperplasia
. Medullary carcinoma ofthe thyroid

. Sipple's syndrome

29
Coplaiglit O 201l-?012, Denral Deckr

slate during pregnancy

l0

days

of

Remember:
L Thc peripheral fibroma is a rcactive hyperplaslic mass that occurs in the Singiva and may be dcrivcd from connective tissue ofthc submucosa or the PDL. It presents as well-demarcatcd lbcal mass
\\ ith eithcr a sessile or pedunculated base. Ii is similar in qolor to the suffounding connective tissue.
It may be ulccratcd. The treatment for a pcriphcral fibroma is local excision. Recurrence is rare'

\ote:

Other variant forms ofthe peripheral fibroma includc:


. The peripheral odontogenic fibroma: which is gingival mass composed ofa well-vascularized,
flbrous connective tissuc. Thc distinguishing featlre ofthis variant is thc presencc ofstrands ofondorogenic epithelium, oftcn abundant, throughout the connective tissue. It is usrlally non-ulcerated.
. Thc peripher.l ossifying fibroma: is a gingival mass in which calcified islands. presumed to be
bonc. are seen. The bone is found qithin a non-encapsulated prolifcration of plump benign fibroblasts. The surface is often ulccrated. See picture #85 in booklet

Focal fibrous hyperplasia is hlperplasia oforal mucosa. It is also callcd traumatic fibroma, irrilarion fibroma. and hyperplastic scar. It is a rcactivc lcsion caused osually by chronic trauma to oral
mucous membranes. The giant cell libroma is a focal fibrous hlT'erplasia in which connective tissue
cells. many ofwhich are multinucleated, assume a stcllatc shapc.

:.

The multipfe endocrin neoplasia syndromes (olso called MEN Syndrome./ have been
classified into three distinct syndromes, each is inherited as an autosomal dominant trait:

. \ten I:

consists of tumors or hyperplasia of the pituitary parathyroids, adrenal


cortex, and ofthe pancreatic islets.
. \len Il (also c'alled Sipple's Syndrome ard sabn?e
is characterized by parathyroid hyperplasia or adenoma, but no tumors to the pancreas. However, in addition, these
patients have pheochromocltomas ofthe adrenal medulla and medullary carcinoma of
the thyroid gland.
. \Ien III (elso kno$n os sub\tpe IIB): rs characterrzed by mucocutaneous neuromas, pheochron'rocytomas ofthe adrenal medulla and medullary carcinoma ofthe thyroid gland.

lr:

Important: MEN I and II are related to MEN lll in that patients with types I and II syndromes have neoplasms ofvarious endocrine organs, but do not have the oral manifestarions ofmucosal neuromas. These oral lesions are most common on the lips, tongue, and
buccal mucosa.

\ote:

The most important aspect of this syndrome is the medullary carcinoma of the
thvroid because of its ability to metastasize and cause death. Therefore, the detection
of the rnucosal neuromas may alert the clinician for early diagnosis and treatment.

. Traumatic neuroma
. Neurilemmoma fsc hwannoma)
. Neurofibroma
. Nodular fasciitis

. Fibromatosis

30

Coplrigh O 20ll-2012

- Dental Decks

. Neurolemoma
. Neurofibroma
. Neuroma
. Fibroma

31

CopFight O 201l-2012 - Dental Deck

Neurofibromas may appear as solitary Iesions or as muitiple lesions as part ofthc syndrome neurofibromatosis 6,()// ne. klinghdusen:\ disease of skin). lr rs a benign ncoplasm, howcver the etiology of
solitary neurofibromas is unknown. Most researchers believe the cell oforigin is the Schu'ann cell; others believe dre pe neLlral fibroblasr is rcsponsiblc- For the solitary neurofibroma thc tongue. buccal mucosa, and vestibule are the oral regions most commonly affected.
Tumor

Clinical Chrracteristics

Etiology

Traumatic

pcnpheralnene

Neurilemmoma

BcniSn neoplasm
Schwann cells

of

Treatmntand Prognosis

Most common sile over mental


foramen in edentulous mouths;
nodule or swelling, which may
be painful lo digital pressure

Excision wilh small proxirnal


ponion of involved nen,e;
recurTence uncommon

Encapsulated mass that Fesents

Conservalive excision;

as an aslmptomatic lump. The


tongue is the mosl commot

localion.

Neurofibrona

Somc invcstigalors
say il is derived liom
the Schwann cell;
olhers say it is derived l-.om

fibroblasts

\odular fasciilis

Two formsl
1. Solltary neurofibroma -

L Solitary: surgical excision

asymplomaiic nodule, occurs on


tongue, bsccal mtrcosa and

2. Neurofibromatosisl
removal is impractical. wrlch
for high rate ofmalignant

vestibule

transformalion

2. Multiple lesions as pan ofthe


s)mdrome Deurcfi brohalosis

Reacti!e lesion;
lpserdosarconatous proliteration of
fibroblasts

P.esents as firm mass; exhibits


r8pid growth; pain and
lendemess common

Surgical excision; recurrence

Benign fibrous
proliferation of
fibroblasts

The mandible and conliguous


sofi tissues are most liequenlly
involved intraorally. Locally

Aggressive surgical approach is


recommended. Recurrence is

aggressive and inliltrative.

\bn

Reckfinghausen's disease (neut'oJibromatosis) is an autosomal dominant disease


associated rvith loss ofa tumor suppressor gene (NF/ or NF2l. lt is characterized by multiple neurofibromas of the oral cavity and on the skin, and pigmentations of the skin
r Ccrfi-au-luit .rpor.s/, iris freckling (Lisch spots), and axillary freckling Crou'e )s sign.).
\ote: The presence ofsix or more caf6 au lait macules greater than 1.5 cm in diameter
is generally regarded as being indicative of this disease until proven otherwise

Radiographic features:
. \\ ell-demarcated unilocular or multilocular radiolucency
. Possible root disturbance
. Possible jarv enlargement
Treatment:

. Surgical excision but may be impractical due to the number of lesions


. Best 1eft untreated because multiple recuffences may be associated with malignant
transformation to neurogenic sarcoma

\ote:

The importance of the lesions is the high risk (5%' to

l5%o.1

of mtlignanl trans-

formation
Remember: The single neurofibroma presents at any age, it comnronly appears as a sessile, firm, pink nodule that commonly occurs on the tongue, buccal mucosa and
lestibule. It is rerloved by surgical excision and rarely recur.

Con Tis Les

A 5S-year-old patient coms into your oflice for routine dental work.
You se that he has a tooth fracture (due to decay) of tooth #31,
A smooth, firm, asymptomatic lesion is noted on the lateral
border ofthe tongue adjacent to the sharp enarnel oftooth #31.
The patient states that the lesion has been there for years and is
annoying because sometimes he will bite it accidenta[y.
Name this most frequently encountered intraoral benign
neoplasm of connctive tissue origin,

. A leiomyosarcoma
. A traumatic fibroma
. A leiomyorna

. A rhabdomyoma
32
Copynghr C 2011-l0ll - Dental Decks

ORAL PATHOLOGY

Con Tis Les

Scleroderma is a systemic disease that alfects many organ systems.


The symptoms result from in{lammation and progressive tissue fibrosis and
occlusion ofthe microvasculature by excessive production and deposition of:

. Types II and IV collagens

Types I and

. Elastin

lll

collagens

and reticulin

. All of the above

Traumatic fibroma, also know as initation fibroma, focal fibrous hyperplasia, and h)?erplastic scar, is
a reactive lesion caused usually by chronic trauma to oral mucorls mcmbranes. Ovcrcxubcrant fibrous
connective tissue repair results in a clinically evident submucosal mass.

Reaclive lesion caused


tmuma to oral mucous

Painless, broad-basd swelling


lhar is lighter rhan surrounding
ii$ue; fiquenlly fotmd on
buccal mucosa, laleral border

oftongxe, snd tower lip


Orat liomyomas arc rare;
present as slow-growlng,
asymptomatic subrnucosal
oasses, usualiy in the longue,

Wid surgical exc's'oni rc'

bard palate, or bucal mucosa

Oral leionyosarcomas have


been reportd in all ag groups
and most intraoral regions.

Wide surgical excisioni

Rare bul have a prdilection


for the soli tissues olthe had
a..d neck Aoor of the nouth,
solt palate. totryue, and buccal
n!.a.ra). Presents as an
asymptomatic. well-dfi ned

Excisioni

Rare in head and neck; most


commonly affecled oml siles
ar th longu and soft palf,te.

Combinition ofsurgery.
radirtion, ard chemothempy.
melaslasis not common

Presents as a mpidly $owing


mass, which, ifrhere isjaw
involvemnt, may caus pa'n.

*"* Otlrcr lcvels of nracromolecules found in the conncctivc tissue fe.g., glytosantinoglycans,
:a,1u'(tn. lihronecti ) are also incrcascd.
scleroderma is an uncommon. autoimmune disease. It all'ects the connective tissues $hich sur:rrund thcjoinls, blood vcsscls and intcmal organs bcneath the af}'ected area ofskin. Womcn arc afi.tcd thrcc to ibur tinres morc oftcn than men. The diseasc usually starts between thc ages of25
llnJ :0 It onl-'- oqcasionally bcgins in children or in the clderly. Although there is no curc. propcr
lrearnlent and care can makc it possible for people rvith sclerodenna to lead full, productive livcs.
\ote: \1a) occur concomitantly with other autoimmune diseases. Such as lupus erythemalosus,
rheunaroid arthritis. dermatomyositis, and Sjogren's sundrome. \ote: Thesc changcs in thc conn.-iti\c trssue all'ect the fingers, trunk, face (producing u "purse-string" moil/i], and the nlore
rr(]\]]'nal parts of the cxtrenlitieS.
Oral radiographs ol'a patient with scleroderma would show an abnormal rvidening of the peri..Jrrnral ligament. This space is created by a thickening ofthe periodontal-membranc as a rcsult of
:n increase in size and number ofcollagen fibers. The enlarged space is almost uniform io width,
.rlrounds the cntirc root of the toolh and makes the tooth appear as if it is being cxlrudcd rapidly
:ilrir its socket. See picture #31 in booklet Note: Other oral radiographic l'eatures may include
]rlar.ral resorption ofthe angle ofthe ramus ofthe mandible or complete resorption 01-thc condylcs
:nd or coronoid process ofthe mandible. Remember: The abnormal widening ofthc periodontal:rembrane spacc is also a radiographic finding in osteosarcomas.

Thcrc arc tlvo major types ofscleroderma:


I Localized scleroderma; In this condition changes only occur in isolatcd areas of the skin
and the tissues beneath it. [t is rclativcly mild and docs not aft'cct intemal organs. Cutaneous
changes includc induration and rigidity. atrophy, and telangiectasias.

L Systemic scleroderma: With

this condition changes rnay occur in thc skin and also in a number of intemal organs. These might include blood vessels, joints, the digcstivc syst"em (esophagus, stomach and borlel/, and occasionally the lungs, hean, kidneys and muscles. Changes in the
connective tissue may affect the function ofany ofthese organs.

ORALPATHOLOGY

Con Tis Les

A newborn baby girl was delivered via caesarian section due airway patency
concerns. During ultrasound, there was th discovery of a tumor ofthe
oral cavity. Upon delivery, the pink, comprssible tumor of the anterior
maxilla was deemed to be a congenital epulis ofthe newborn. This lesion
is composed ofcells that are identical to those ofthe:

. A traumatlc neuroma
. A schwannoma
. A granular cell myoblastoma
. A lipoma

34
CopyriShr

201

l-201

- Dental Decks

ORAL PATHOLOGY

lnfl Jaw Les

An emergency patient walks into your o{fice with swelling ofthe left
submandibular space. He says his lower left molar recently 'broke down"
and has been very painful especially when something cold hits it or whcn
he chews down on it. What is the most likely etiology of this swelling.

. Orthodontics
. Trauma

. Infection of the pulp ofthe tooth


. Periodontal disease

Copyrighr C

20ll'l0ll

- Dental Decks

The congenital epufis of the newborn (also called congenital gingival granular cell
tumor).lt rsually appears on the gingiva (usually anteriof ofnewborns. It presents as a
noninflamed, pedunculated or broad-based mass. The maxillary gingiva is more often
involved than the mandibular gingiva, and ferrales are affected more than males. The
treatment is surgical excision with little possibility ofrecurence.
See

pictur #24 in booklet

The granular cell myoblastoma is a rare neoplasm of unknown etiology. Most researchers believe its origin is from the Schrvrum cell. It presents as an uninflamed. asymptomatic mass less than 2 cm in diameter. The most common location in the head and neck
region is the tongue. It may affect any age group and females seem to be affected more

than men. See picture #25 in booklet

Important: Both of these lesions are identical histologically. They both contain granular cells, however, the congenital epulis of the newborn does not exhibit overlaying
pseudoepitheliomatous hyperplasia. The pseudoepitheliomatous hyperplasia of the
or erlying epithelium is frequently seen in the granular cell myoblastoma.

***

This infection follows the carious involvement ofthe tooth. The cellular debris and/or
rnfection rvhich caused the tooth pulp to die, slowly filters out of the tip of the root and
produces an inflanmatory reaction around the root tip
also occur after traumatic injury to a tooth, which results
in necroiis of the pulp, and in cases ol irritation of the periapical tissues, either by mechanical manipulation or by the application of chemicals in endodontic procedures'

\ote: ,\ periapical abscess can

Clinical features:
. If acute, presents

as an

abscess:

See

picture #26 in booklet

Tooth is extremely painful to percussion


- May feel slightly extruded from its socket
- Tooth will exhibit mobilitY
If chronic, presents as a granuloma or cyst. There are usually no clinical features of

svmptoms

Radiographic features:
. If acut, only a slight thickening ofthe periodontal membrane is noticeable

.If

chronic (granuloma or q,st), there will usually be


the involved tooth. See picture #27 in booklet

radiolucent area at the apex

of

Treatment: Establish drainage either by opening the pulp chamber or extracting the
tooth.

Note: If

a periapical abscess is

not treated, it can lead to serious complications such as

osteomvelitis. cellulitis and bacteremia.

A [ealthy patient comes into your ollice for an inltial exam. On the full mouth
series of radiographs you see a radiopaque lesion perlaplcal to tooth #19. Tooth
#19 has a deep amalgam restoration with rcurretrt decay underneath. You csn
Nee the entire outline ofthe mesial root oftooth #19 - the lesion seems to stem
frorn the tooth. What is the most llkely diagnosis of the lesion?
N.

. Focal sclerosing osteomyelitis

. Cementoblastoma
. Cementoma
. Fibrocementoma

CopyriShr O 201 1,2012 - Dental Dcks

may be psrt ofthe etiologic picture, although


been the most frquently cited.

. Bacteroides and campylobacter

. Clostridia

and corynebacterium

Staphylococci ard streptococci

. Enterococci

and lactobacilli

37
Copyrighr O 201 l-2012 - Dental Decks

Focal sclerosing osteomyelitis is a relatively common phenomenon that is believed to represent a focal bony reaction to a low-gnde inflammatory stimulus. [t is usually seen at the apex
of a tooth in rvhich there has been a long standing pulpitis. Note: Synonyms for focal sclerosing osteomyelitis include bony scar, condensing osteitis, and sclerotic bone. The term focal
periapical osteopetrosis has also been used to descdbe the lesions associated with normal
caries-free teeth.
Focal sclerosing osteomyelitis may be found at any age but is typically discovered in young
adults. Patients are usually asymptomatic, and most lesions are discovered on routine radiographic examination. A majority are found at the apices ofmandibular first molars. The periapical x-ray demonstrates the pathognomonic, well-circumscribed radiopaque mass of
sclerotic bone surrounding and extending below the apex ofone or both roots. The entire root
outline is almost always visible, an important litature in distinguishing it from the benign cementoblastoma, which radiographically, it may resemble. The tooth with this lesion may be
treated or exftacted, since the pulp is infected and the infection has spread past the immediate
penapical area. The sclerosing bone constituting the osteomyelitis is not attached to the tooth.
and remains after the tooth is treated or removed.
DifTuse sclerosing osteomyelitis represents an inflammatory reaction in the mandible or maxilla. believed to be in response to a microorganism oflow virulence. lmportant in the etiology
and progression ofdiffuse sclerosing osteomyelitis is chronic periodontal disease, which appears to provide a portal ofentry lorbacte a. The condition tends to occur most ftequently in
middle-aged black females. The disease is typified by a protracted chronic course with acute
exacerbations of pain, swelling, and occasional drainage. Radiographically, this process is
dld'ure. 9*pically affecting a large part ofthejaw The lesion is ill-defined. Treatment consists
t idetermining and addressing the cause. Antibiotics are the mainstay of trcatment. Low-dose
.onicosteroids have also been used with some success. Hyperbaric oxygen therapy may prove
:.r be a r aluable adjunct.

Acutc inflammation ofthc bonc and bone manou'ofthe mandible and maxilla rcsults most fiequently
from rtension of a periapical abscess. The second most common cause of acute osteomyelitis is
phlsical injury as secn u,ith fractlte or surgery. Most cases of acute osteomyelitis arc infcctious.
Staphr-lococci and streptococci are the most frequcntly cited.

Pain is the primary feature of this inflammatory process. Fever, painful lymphadenopathy, leukocytosis. and other signs and symptoms ofacutc infcction arc also commonly found. Paresthesia ofthe lotler
lip is occasionally seen with mandibular involvcmcnt. Important: Unless the inflammatow process has
bccn prcscnt for more than I wcck, radiographic cvidcnce ofacutc ostcomyelitis is usualll_ not present.

\\'ith timc. diffuse radiolucent changes bcgin to appcar. Treatment includes antibiotics and drainage.
Chronic osteomyelitis (chronic osteitis) fiay bc one ofthc scquclac ofacule osteomychlis /ei er ultI eatetl o, in.tdequateb lrealed), or rt may represent a long-tenr, lorv-grade inflammatory reaction that
nevcr $ ent rhrough a significant or clinically noticeable acute phase. Most investigators believe that
bacleria /e.g., staph ococci, \treptococci, bacleroiles. acli on\rer'/ are responsible for thc vast majoriI ol chronic osteornyelitis cases. The mandible, especially the molar area, is much more lrequently
atl'ected rhan js thc rnaxilla. Pain is usually present, s\relling ofthe jaw is a comrnonly encountered sign;
loose teth and sinus tracts are less frequently seen. Anesthesia is very uncommon. Radiographically,
chronic osteomyclitis appears primarily as a radiolucent lesion that may show focal zones ofopacificarion. The luccnt pattem is often described as "moth-eaten" because ofits mottled radiographic appearance. Treatment includes antibiotics and scqucstrectomy.

Gar16's osteomyelitis or chronic osteomyelitis with proliferative periosteitis, is cssentially a subtype of


chronic osteomyclitis in which there is, additionally. a prominent periostcal inflammatory reaction. It
mosi olien results from a periapical abscess of a mandibular molar in a child. The child characterisricali.v presents with an asymptomatic bony hard swelling with normal-appcaring overlying skin and
mucosa. Radiographicallli thc lesion appears centrally as a mottled, predominantly lucent lesion in
a pattem consistcnt uith chronic osteomyelitis. The featrre that provides thc distinctive difference is the
periosteal reaction. This appears as an expanded cortex, oftcn uith concent c or parallcl opaque layels. Trabeculae perpendicular to the "onion skin" layers may also be present. Treatment includes tooth
extraction and antibiotics.

. First to third week

. Fourth to sixth week


. Eighth to tenth week
. twelfth to fourteenth week

38
Coplright O 20ll-2012 - Dental Decks

. Relatively normal torso

and long arms and legs

Short torso and long arms and legs

. Long torso and long arms with short legs

. Relatively normal torso and short

axms ajld legs

39
Coplrighr O 201l-2012 - Denral Decks

Cleft palate occurs in the eighth to tenth week of embryonic life. Isolated clefts olthe
palate are more common in females. It is characterized by a fissure in the midline ofthe
palate, resulting from the failure ofthe two sides to fuse during embryonic development.
The most severe handicap imposed by cleft palate is an impaired mechanism preventing
normal speech and swallowing. Note: It effects approximately I in 2000 births.

Cleft lip results when the medial nasal procss fails to fuse with the lateral portions of
the maxillary process olthe first branchial arch. Fusion normally occurs during the sixth
and sevnth weeks of embryonic development. The maxillary lip is most commonly affected. It may be bilateral (20a/o) or lunilateral (80%). Clefts ofthe lip are more frequent
in males. Lip clefi involvement is more frequent on the left sid than the right.
Note: It effects approximately I in 1000 births, but varies with race.

\ot<

L Speech problems associated with both ofthe above are usually the result of
the inability ofthe soft palate to close airflow into the nasal area.
2. It is not unusual for teeth especially the lateral incisor to be missing in the

cleft area.
3. More than 250 syndromes have been identified that may be associated with
cleft lip and palate, or cleft palate alone.
4. Lip pits are rare anomalies that can occur in the upper lip, lower lip, or the
oral commissure. Although lip pits may be seen near the oral commrssure or
midline upper lip, most occur on the Iower lip and are associated with van der
\lbude syndrome. This syndrome is an autosomal dominant condition with
80% to 90% penetrance consisting of lower lip pits and cleft lip and/or cleft
Dalate.

.\chondroplasia is the most common type of dwarfism. The upper an.ns and thighs are
more shortened than the forearms and lower legs. Generally, the head is large, the forehead is prorrinent, the nose has a saddle-like appearance and the mandible exhibits progn

al

ism.

Potential problems in children with achondroplasia include overcrowding of the teeth,


speech problems (articulatio ), and frequent ear infectior:Ls (otitis media). Dental malocclusion is treated \\. ith orthodontics. All children with achondroplasia should be evaluared b\ a speech therapist by two years of age because ofpossible problerrs with the
de\eiopment ofclear speech. Articulation problems may be caused by onhodontic probIems. Due to the abnormal shape of the eustachian tube in an individual with achondroplasia, they are very prone to ear infections (otitis media).

\ote: The teeth are of nomal size but there is limited space within the maxillary and
mandibular arches for them to erupt into which causes overcrowding and subsequent malo!'cluS ron.

.oI
. Marfar's syndrome

. Ehlers Danlos
. Cystic Fibrosis

40
Coplrighl O 2011,2012 - Ltental Decks

Hypophosphrt.sia is a genetic metabolic disorder of


bone mineralizstion caused by a deficiency in:

. Acid phosphatase

. Vitamin K
. Alkaline phosphatase

. Phosphorus

41

Copyrighl c' 2011,2012 - DentatDcks

The characteristic features olosteogenesis imperfecta (OI) vary greatly from person to
person and not all characteristics are evident in each case. The chief clinical characteristic ofosteogenesis imperfecta is the extreme fragility and porosity ofthe bones, with a
proneness to fracture. Other features include: pale blue sclera, deafness due to otosclerosis, abnormal teeth, loosejoints and low muscle tone. a triangular face, and a tendency
toward spinal curyature.

Clinically the teeth have:


. Crowns that are bulbous: with a cervical constriction
. Pulps that are obliterated, either partially or completely
. Roots that are narrower and shorter

***

The deciduous teeth are more sverlv affected that the permanent dentition

T! pes of ostogenesis imperfecta:


. Type l: most common and mildest fonn
. Type ll: most severe lonn resulting in multiple fractures just from birth process
. Type lll: most severe form beyond the perinatal period
. Type IV: mild to moderately severe bone fragility
There is no known cure for osteogenesis imperfecta. Treatment is directed toward pre\ enting or controlling the symptoms.

\ote:

The cause of osteogenesis imperfecta is believed to be due to a genetic defect that


causes imperfectly-formed, or an inadequate amount ofcollagen. The affected person has
either less collagen than normal, or a poorer quality of collagen than normal, leading to
\r eak bones that fracture easilY.

Hr pophosphatasia is an inherited metabolic (chemical) bone disease that results from low
lerels ofan enzyme called alkaline phosphatase. This enzyme is essential to the calcification ofbone tissue. The severity of hypophosphatasia is remarkably variable from patient to
patient. Some patients have blue sclera that resembles ostognsis imperfecta. There may
be delbrmity ofthe arms, legs and chest. Frequent bouts ofpneumonia can occur as well as

rccurrent tiactures.

J h pes of h) pophosphatasia:

. \eonatal:

severe manifestations, respiratory failure, marked hypocalcification of the


skeletal structures
. Infantile: h-vpercalcemia, premature loss of deciduous teeth, skeletal malformations,

failure to grorv

. Childhood: short stature. frontal bossing, usually normal calcium and phosphate levels
. Odontoh! pophosphatasia: children and adults rvho have only dental problems

Important: The premature loss of teeth in children and adults is usually characteristic.
These teeth also exhibit hypocalcification. Radiographically, the teeth display enlarged
pulp chambers and pulp canals, deficient root development as well as alveolar bone loss.
Rememtrer: Patient's with Paget's disease also have high levels of serum alkaline phosphatase.

. Gigantism

. Acromegaly
. Achondroplasia
. Dwarfism

Coplrighi C

201

42
l'2012 - Denral Decks

A new 6 year old pediatric patient walks into your operatory with hls mother,
You initirl physical assessment nots ! prominent foreh""d
;;;;;;;;;.
""drerlize thst the
The prtietrt initirlly seems to have no eyebrows but you lrter
hair is just very fine and sptrse. When you shake her hand and she smiles,
you llso notice that she is n.lssing teeth and the ones she has are cone
shaped. Whst fu her most likely systemic condition?

. Piene Robin syndrome


. Ectodermal dysplasia

. Cleidocranial dysplasia
. Peutz-Jeghers syndrome
. Osteopetrosis
43
Coptrighr O

201

l-2012 - Dntal Decks

In over 90% of acromegaly patients, the overproduction ol GH is caused by a benign


tumor of the pituitary gland, called an adenoma. Whether or not the epiphyses of the
long bones have lused with the shaft is the main detenninant of whether gigantism or
acromegaly will occur when there is oversecretion of growth hormone by the pituitary
gland.

Remember:

. Gigantism: tumor prior to adolescence (non-fusion o.f epip\,ses)


. Acromegaly: tumor after adolescence (fusion of epiphyes)

Oral manifestations ofacromegaly and gigantisrn include: enlarged tongue, mandibular


prograthism, spacing of the teeth rvhich are usually tipped to the buccal or lingual side,
oNing to enlargement ofthe tongue. Roots may be lorger than normal. See picture #33
in booklet.

\ote: Dn arfism (pttuitary dvtar/i)

is characterized by arrested growth. Frequently these


people have limbs and features not properly proportioned or formed. It is caused by undersecretion ofgfowth homone. Oral manifestations include: eruption rate and the shedding ofthe teeth are delayed, clinical crowns appear smaller as do the roots of'the teeth,
rhe dental arch as a whole is smaller causing malocclusion and the mandible is underder eloped.

Ectodermrl dvsplasia is an x-rinked recessive condition charactedzed by abnormar deveropmenr ofthe


.i1n 3nd associated struct]lfes lhdir noi!!, and teeth, ond s$eat gldnd.rl. It involves
all structures which are
::nr ed tiom rhe ctodcrm. It affects mares more than females. common clinical findings include hvpothrichosis /./e./?d.re in hait,).
ho s\\,eat gland.r, leoding to h(,at iuh )l..r,La. c/lanodontia t,r
oligodontia konplere or partiar^nhidrosis
absenk of teeth). depresscd bridgc of nose, Iack of salivary grands
l.,l the child appears much oldcr than rvhat he or shc is. There is no treatment lor the disease. hoNever
;::rrures can be fabricated for these patients. Keep in mind that they will need to be replaced periodicallv
jaw growth. Scc picturcs #28 and #29 in booklet

:-r r..!rmmodate the palient's

Clcidocranial d]splasia is an autosomal dominant condition ofbony developmenr characterized by hypoplasia or aplasia ofthe clavicles' cranial bossing, ocular hypertelorism, and dental abnonnalilies which
::rrlrJe retaincd primar) teeth, malaligned leeth, the presence of multiple supcrnumerarv teeth and
unerupted reeth- lmportant: The dentition itself, as obsen,ed by radiographs alone, often suggests the di:5o.rs. Sec picturc #30 in booklet
Pierre Robin svndrome is an inherited disorder that presents the fblowintl in the neonate: severe micrognsthia and mfndit ular hypoplasia. se\ ere glossoptosis //),,, ariot rlt,fLttenent ofthe tongtrc), and
high-arched or cleft pal:tte. This condition is characterized by respiratory problems.
Peutz-.Icghrs syndrome /PJSI is a genetic condition rnarked by hyperpigmentation (r"rfltrg ofthe lips
.rnd \ometlmes other parts ofthe face, hands, and feet followed by the development ofbenign polyps
called

arn'anomas throughout the intestines but primarily in lhe small intestine. See picture #35 in booklet

osteopetrosis /a&o.a lled Albers-Se honberg disease or narble bone./iredr., is an uncommon bone conJiiron thal mav be inherited as an autosomal dominant /&rr--reno .!, or recessive trait 6drle seri.n/s./. The
.r3racteristic feature ofosteopetrosis is an abscnce ofphysiorogic bonc resorption owing to rcduced os(eoclastic activity' The lack ofbone resorption results in accumulation
ofbone mass and manifesls itself
:. ikeletal disturbances. including bone cavity occlusion. decreased hematopoietic activity, and growth re1:rrdation. Bone pain is the mosl liequent symptom. Blindness and deafness from sclerosis
ofostia. anemin
riom sclerosis ofbone marroq and osteomyeliris due to diminished vascularity are aiso seen. Dental findings include delayed enrprion. congenitally absent teeth. unerupted and marormed teeth, and enamer hyooDldsla,

M&GJawDis

ORAL PATHOLOGY

An old patient presents to your oflice with ill-fitting dentures. Radiographs reveal hypercementosis on roots and the patient is speaking embarrassingly loudly
to you that she is more conscious ofhis baldness now that his hat doesn't fit
anymore." These signs all point to what possible diagnosis?

. Paget's disease
. Osteosarcoma
. Fibrous dysplasia

. Albers-Schoenberg

CopyriShr C 2011

:0ll

Denral Decks

ORAL PATHOLOGY

M & GJawDis

You are consulting on a pathological case for a fellow dentist. The biopsy of
the lesion sho\fls multinucleated giant cells and perivascular collagen cuffing.
After asking about the clinical signs, your colleague mentions that the young
patient seems always to be (staring offinto spac. and that she has.puffy
cheeks." The most likely diagnosis of this case is:

. Aneurysmal bone cyst

. Central Giant Cell Granuloma


. Tumor of hyperparathyroidism
. Cherubism

45
Coplrighr ie

20ll-l0l:

Denlal Decks

booklet
Paqet's tlisease ofbone is also called Osteitis Deformans' See picture #32 in
appears
Paget's discase ofbonc is a chronic, slouly progressive condition ofunknown ctiology(it
cranium' pelvis'
l'elnur'
thc
spine'
include
ofinvolvemenl
sitcs
frecluent
most
The
toie lanitiatl.
and fbrand stemum. Tbe bone may become dcnse, but liagilc, because of cxcessive breakdou'n
of
50. Thc
thc
age
ovcr
adults
affccts
mation ofbonc. The disease is morc conmon in males and
area'
thc
irffected
in
tlre
bone
of
arca,
defomlily
signs and symptoms inclucle pain in thc aflectcd
arca
affccted
ifthe
loss
hearing
and
headache
and
arca'
su"sceptibility to fractures in lire affected
slowly'
dcvelop
is the skull. Note: Thesc symptoms

***

When the jarvs are affected:


n-Iaxilla progresPatient who rvears a maxillary denture cotnplains that thcy do not fit aS thc
sively cnlarges
. Ultimatelyl the alveolar ridge wiclens. r,irh a rclative flatrening ofthe palatal vault
. When tecth are prescnt, incrcased spacing as wcll as loosening is notcd
. Hypcrcemcntosis of tooth roots, loss of lamina dura, oblilcralion of the PDL space and re-

sorption ofthe roots


and multinuclcated osleoclasts arc lbund in abundance As the lcsion
numeious reversal or growth lines are sccn' giving the tissue a mosaic

]licroscopically, ostcoblasts
adr ances. dense bone

wlth

parrem.

Other important features of this disease include:


. Palicnts may also give a history ofprogrcssively increasing sizc ofhats or nc'$ dentttrcs being
made at progiessively more frequent intervals. Note: This is due to bony changes'
. Boncs are warm to touch due to incrcased vascularity

.\-raysofthesku|landthejawsdcmonstlatethctypical''cotton-wooll.appearance
. Lab iests: Drastically increased serum alkaline phosphatase. Scrum phosphale and calcium
calciun and hydroxyproline are increased'
. Treatment: Recently. thc use ofcalcitonin and bisphosphonates as parathofmone antagonlsls
latal
ha\ e bcen eilective in suppressing bone resorption and dcposition lt is seldom

are normal. Urinary

Cherubism is a benign, autosomal dominant condition ofthe maxilla and mandible, usuall-,- found in children by 5 years ofage (it olfects males 2:i). The vast majority ofcases
occur in the mandible. The bony expansion is most frequently bilateral, although unilar
eral involvement has been reported. The clinical appearance may vary from a barely discemible posterior swelling ofa singlejaw to marked anterior and posterior expansion of
both jau s. resulting in masticatory, speech, and swallowing difficulties. Intraoralh', a
hard. non-tender swelling can be palpated in the affected area. There are no associated
sl sterric manifestations. The deciduous dentition may be spontaneously shed prematurely,
beginning as early as three years of age. There is often delayed eruption of the pennanenr dentition uhich is olten defective with the absence ofnumerous teeth and displace-

mcnt of those present.

Radiographicalll', the lesions characteristically appear as multiple, well-deltned, multilocular radiolucencies ofthejaw. The borders are distinct and divided by bony trabeculae. \ote: An occlusal radiograph of the maxilla may give a "soap bubble-like" picture
rr rrh

m.rrillrq

anrrum obliteration.

Histologicalll, the lesions bear a close resemblance to those seen in central giant cell
granulomas. There are nlrmerous fibroblasts and rnultinucleated giant cells with prominent nuclei. A distinctive feature is eosinophilic perivascular cufling of collagen surrounding small capillaries throughout the lesion. Although this is not always present,
perirascular collagen cuffing is regarded as pathognomonic for cherubism.
The treatment is cautious waiting as there is spontaneous regression of the tumors at
around 25 - 30 years ol age. With increase in age and size of the patient. the defotmity
produced is less noticeable.

. Vitarnin A
. Vitamin C
. Vitamin D
. Vitamin K

Coprighr O

201

45
l'2012 - Dental Decks

. Toxic nodular goiter


. Graves'disease
. Hashimoto's disease
. Addison's disease

17
Copyright @ 201 t-2012 -

De

al

D4ks

Hypoparathyroidism is a rare disorder associated with insufficient production ofparathyroid hormone, the inability to make a usable form ofparathy'roid hormone, or the inability ofkidneys and bones to respond to parathyroid hormone production.
Hypoparathyroidism can result from congenital disorders, iatrogenic causes (e.g., drugs,
rcmoval ol the porathyroid glands during thyroid or parath!-roid sto'qery, radiation), rnfiltration ofthe parathyroid glands (e.g., metqstatic cqrcinoma. Il/ilson disease. sarcoid),
suppression of parathyroid function, HIV/AIDS, or idiopathic mechanisms.

Hypocalcemia is the most important consequence of hypoparathyroidism. Symptoms


occur when ionized calcium level drops to less than 2.5-3 mg/100 mL. The clinical manifestation is tetany. A positive Chvostek's sign (nitching oJ the.fdcial muscles v'hen
rapped on the facial nen e near the parotid gland) is characteristic ofhypoparathyroidism.

lmportant: The dental manifestations of hypoparathyroidism (i.e., delayed eruption,


euantel ht,poplasia and blunted root apices) may be prevented by early treatment with
vitamin D.
Remember: The term muscular dystrophy refers to a group ofgenetic diseases marked by
the progressive weakness and degeneration olthe skeletal, or voluntary muscles, which confiol molement. Oral manifestations include an increase in dental disease iforal hygiene is
neglected. weakness in the muscles of mastication leading to decreased maxillary biting
ibrce and a higher-incidence of mouth breathing and open bite.

The rerm hr..perthyroidism refers to any condition in which there is too much thyroid hormone
tln rorr'r, in the body. This most commonly results fron a generalized overactivity ofthe entire
ihlroid gland. a condition also known as diffuse toxic goiter or Graves'disease. Altematively,
one or nrore nodules or lumps in the thyroid may become overactive, a condition known as toxic
nodular goiter or Plummer's disease, The primary role ofthyroxin is to stimulate cellular merabolism. grorvth and differentiation of all tissues. In excess, it leads to high basal metabolism,
tilrgue. \! eight loss, excitability. elevated temperature and generalized osteoporosis. Oral manifestations are not too remarkable, but ifthe disturbance begins in the early years of life, the
F.en'rature eruption of the teeth and the prernature loss of the deciduous dentition are common

dndings.
T! pes of hi perthJ-roidisml. (Note: Besic symptoms of htperth,'-/oidisn dre present as h,ell as

lJdi|io

aI \nptonts).
. Gra\'es'disease: is the most common form, occurs most frequently in women under
- goiter knlarged thyoid which moy catrse a bulge in the netkl

50

erophthalmos is common
thickened
skin over the shin area
. Plummer's disease (toxi. nodular goiler; afibcts both genders usually over 50
Erophthalmos is rare
Ofien uni-system, may present with only cardiac disease
causes temporary hypcrthyroidism, usually followcd with hypothl roidism. Thyroiditis is an inflammation ofthe thyroid gland.

lmportant: Thyroiditis

Therc are three main types of thyroiditis:

. Hashimoto's thyroiditis
. Subacute granulomatous thyroiditis
. Silent lymphocytic thyroiditis

. Dwarfism
. Mlredema
. Cretinism
. Acromegaly

48
Coprighr C

201

l-2012 - Dental Decks

The clinical features ofthe primiry form of which disease is


classically described as "stones, bones, groans, and moans?"

. Paget's disease
. Hypophosphatasia

. Hyperparalhyroidism
. Hyperthyroidism

49
Coplright O 20ll-2012 - Denral Decks

Hypothyroidism refers to a condition in which the amount ofthyroid hormone in the body
is below normal. This is the most common form ofthyroid function abnormality, and is
far more common than hyperrhyroidism. This condition is considerably more comn.ron in
women than in men. The most common cause of h)?othyroidism is Hashimoto's thyroiditis. The second most common cause is the treatment of hyperthyroidism. Hypothyroidism is characterized by pufiiness of the face and eyelids and swelling of the tongue
and larynx. The skin becomes dry and rough and the hair becomes sparse. The individual
has a low basal-metabolic rate and a low body temperature. The affected individuals
also have poor muscle tone, low strength and get tired very easily. Mentally they are very
sluggish. The treatment of hypothyroidism is straightforward and consists of administering thyroid hormone (t hyt oxin).
Se\re hypothyroidism in a child is called cretinism. Due to a lack of thyroid hormone,
there is a retardation of growth and an abnormal development of bones. Mental retardation
is caused by the improper development ofthe CNS. If this condition is recognized early, it
can be markedly improved with the use of thyroid hormones. Note: Extreme hypothyroidism in adults is called myxedema.

\ote:

Dental findings in a child with hypothyroidism include an underdeyeloped


mandible with an overdeveloped maxilla, enlarged tongue which may lead to malocclusion. delayed eruption ofteeth and deciduous teeth being retained longer.

H) perparathyroidism (von Recklinghausen's disease of bone) is a tnctabolic disorder in which the


parath.lroid glands produce too nruch pamthyroid honnonc. Too much parathyroid horrnone causes
loo much calcium to be released from bon. It may be caused by a f'unctioning parathyroid tunor
or conlpensatory parathyroid hyperplasia due to renal lailure, malabsorption, or vitamin D deficrency. Thcrc is a ferralc prcdilection and it affccts middlc agcd adults. Thc symptoms include:
Ioss ol'appetite, increasing thirst, lrequent urination, Jethargy and fatigue, muscle weakness, joint
pain and constipation. Important: pathologic fraettre (due lo the marked rcsorplion of borc)
mal bc thc first symptom ofthe disorder Intraorally, thcrc is diffuse bonc loss causing malocclusron and shifting

ofthc teeth.

The chie I radiographic finding is the appcarance ofwell-defined cystic radiolucencies ofthejaw,
*hich mav be unilocular or nultilocular Partial loss ofthe lamina dura is seen around thc tccth.
See picture #34 in booklet

lmportant: Histologically, multinucleated giant cells are scattered within a dclicate fibrocellular
Accumulalions ofhcmosiderin and extravasated red blood cells also rc present. As a result, thc
ussue-i appear rcddish-brown, accounting for the term "brown tumor." These lesions arc miuroscoprcall) idcntical to qentral giant cell granulomas.

'iroma.

Thc discasc spectrum ofprimary hyperparathyroidism rangcs from asynptomatic cases ldiagnosetl fion routine senon calcium determiralrotrs/, to severe cases oflcthargy and occasional coma.
Earl) synrptoms include faligue. weakness, nausca, anorexia, polyuria, thirst, depression, and constipation. Frequcntly, bone pain and headaches are present. There are several clinical f'caturcs associated primary hyperparathyroidism, classically described as "stones, bones, groans, and moans."
Lcsions ofthe kidneys, skclctal system, CI tract, and nervous systcm are responsible lor this syndrome complex. The rcnal component includes the prcsenc ol'renal calculi.
Management of primary hyperparathyroidism is aimed at eliminating the parathyroid pathology.
Surgery is the treatmcnt ofchoice.

. Vitamin A

. \4tamin D

. Vitamin

. Mtamin E

50
Coplrighr

2011,2012 - Dntal Decks

. Difficulty with mastication and swallowing


. Higher incidence ofperiodontal disease and caries
. Attrition ofthe teeth

. Multilocular radiolucencies of the jaws

Coprrishr

201l-2012, Denral Decks

Osteomalacia involves softening of the bones caused by a deficiency of vitamin D or


problems with the metabolism ofthis vitamin. This softening ofthe bones occurs because
the bones contain osteoid tissue which has lailed to calcily due to the lack of Vitamin D.
Note: Osteomalacia may occur as a complication ofsteatorrhea secondary to chronic pancreatrtls.
Clinical findings:
. Osteopenia

. Bone softening/deformity: hourglass thorax, bowing of long

bones

. Increased fractures. biconcave vertebral bodies

. Mottled skull
All

bones are affected, specifically their epiphyseal growth plates. Osteomalacia appears
to be more common in women. This condition mav be asvmDtomatic until fracture occurs.

Blood tests may show:


. Lou levels of vitamin D
. \la) also show low calcium and phosphorus levels
. .\lkaline phosphatase levels can be high
Rickets is osteomalacia in children. It causes skeletal deformities. It is usually accompanied bl listlessness, initability and generalized muscular weakness. A child with rickets
may hare borvlegs and develop a pigeon breast and a protruding stomach. The teeth in a
child l ith rickets are affected as follows: delayed eruption, malocclusion and developmental abnormalities ofthe dentin and enamel alone with a hi{:her caries rate.

Cerebral palsy is a term used to describe a group ofdisorders affecting body movement
and muscle coordination. It is due to an insult to or anomaly ofthe brain's motor control
centers. This damage interferes with messages from the brain to the body, and from the
bod\ to the brain. The effects vary widely from individual to individual. Cerebral palsy
rs characterized primarily by spastic paralysis or impaiment of control or coordination
!r\ er \ oluntary muscles and is often accorrpanied by mental retardation, seizures and dis!rrders of Yision and communication.

\ote: No intraoral

anomalies are unique to persons with cerebral palsy. However, several


the ltont ofthe card) are more comnton or more severe than in

conditions /r/rose /isted on


the normal population.

Remember: Down syndrome is a congenital defect caused by a chromosomal abnorrnaliy.- (trisomy 2 l). It is marked by various degrees of mental retardation and characteristic phvsical features such as a short, flattened skull, slanting eyes, a thickened tongue
r.tissured), broad hands and feet and other anomalies. Other oral rnanifestations include
mandibular prognathism. increased incidence ofperiodontal disease, delayed eruption of
teeth. higher incidence of congenitally missing teeth, malocclusion, and enamel dysplas

ia.

. Sjdgren's syndrome
. Cystic fibrosis
. Cerebral palsy
. Down syndrome

52
Cop,righr O

201

1,2012 - Dental Dcks

. Hepatitis A
. Hepatitis B

. Hepatitis

CoP)rght O 201l-2012 - Denlal Decks

*** Important:

There is a significantly reduced caries rate in patients with cystic fibrosis. This is probably the result of alterations in saliva and the long{erm use of antibiotics.

Cystic fibrosis is a congenital metabolic disorder that causes the exocrine glands (vhich
are glands thqt seqete.fluids into a duct) to produce abnormal scretions, resulting in
several symptoms, the most important of which affect the digestive tract and the lungs. In
some glands, such as the pancreas and those in the intestines, the secretions are thick or
solid (an ext essivef;, viscous mucous/ and may block the gland completely. The mucousproducing glands in the airways ofthe lungs produce abnormal secretions that clog the airrvays and allow bacteria to multiply. The sweat glands secrete fluids that have a high
sodium and chloride content. Note: The staining olthe teeth is most likely due to the lact
that patients with cystic fibrosis are usually subjected to large amounts of tetracyclines

during childhood.
Si mptoms of CF include:

.
.
.
.
.
.

Poor growth despite good appetite


Malabsorption and foul, bulky stools: steatorrhea
Chronic bronchitis (COPD) with cough
Recurrent pneumonia: respiratory infections
Clubbing offingers and toes
Banel-chested appearance

l4tlLlhighly contagious infectious disease involving the liver It is usually transrnined by dre fecal-oral route. However, it may also be transmitted parenterally. tlepatiris -{ usually rcsults liom ingestion ofcontaminated food. milk. or water. Many outbreaks
of tl'ris tvpe are traced to ingestion ofseafood from polluted water. It is caused by an RNA
enteroYirus. [t most often occurs in young adults. The initial symptoms (bver malaise,
,tbdoninal pain. anorexia, jaundice.)of Hepatitis A appear after an incubation period of
l{epatitis A is

-1-6

eeks.

Darnage to the liver cells. also results in increased serum levels of enzymes, such as
transaminases, nonnally active in liver cells. The detection ofincreased serum levels of
rhese enzlrnes is used in diagnosing this disease. In most cases ofHepatitis A, the infecrion is selt'-limiting and recovery occurs within 4 months.

Oral complications: The only oral complication associated with hepatitis is the potential for abnormal bleeding in cases ofsignificant liver damage. Ifsurgery is required, it
is advisable to check with the patient's physician prior to scheduling the sLrrgery.
I . The presence of surface antigen (A or B) in a patient's serum indicates that
the patient is potentially infectious for Hepatitis (carrier state).
...,.:-,,,a:. 2. Hepatitis viruses are very heat-resistant (morc so than the AIDS virus).
3. Autoclaving properly will kill these viruses.

\otei

. Hepatitis A
. Hepatitis

. Hepatitis D
. Hepatitis E

54
Coplright O 201 l -20

12

- Deotal Decks

Atl of the following aid in wound healing Excepr one.


Which one is the EXCEPTIOM

. Highly vascular areas (i.e., tongue)

. Hyperthermia
. Younger

age

. Cortisone

Copyrighr O

20ll-2012,

Dental Decks

The Hepatitis B virus (HB V) rs a double-stranded DNA virus with worldrvide distribution,
transmitted by parenteral and sexual contact. Risk factors include multiple sexual partners,
intravenous drug abuse, and rcccipt ofblood products. Its incubation pcriod is 40 to 100
days, and it can be recovered from all body fluids, most importantly, blood, brcast milk, and
amniotic fluid. The signs and symptoms are similar to hepatitis A (fever abdominal pain,
nausea, etc.) but there is a longer incubation priod f6-8 ueefs). The symptoms arc slorver
in devcloping but are ofa longer duration. Most patients rccover fully, however, some develop chronic liver disease.

Previously, viral hepatitis that was not caused by the type A or type B virus was callcd "nonA. non-B hcpatitis." Rccently three more viruses havc becn identilied that causc some of
these non-A. non-B infections.
These new Yiruses includ:

. Hepatitis C: is a serum hepatitis that is caused by a virus antigenically different from


Hepatitis viruscs A and B. Most cases olpost-transfusion hepatitides are ofthis type. It
is usually much milder than A or B but is otherwise clinically indistinguishablc from
thcm. Thcrc is a higher incidence of chronic disease (chroni< lrcp41llrt, cirrhosis and
hepatocellular carcinoma. Note; Hepatitis C is now the most common reason for liver
trirnsplantation in the U.S.
. Hepatitis D is found only in patients with acute or chronic episodes of Hepatitis B,
and it makes the Hcpatitis B infcction morc sevcrc. Drug addicts are at relatively high
. Hepatitis E: is transmittcd cntcrically, much like Hcpatitis A. It causes occasional epid.'nics similar to those caused by Hepatitis A. So far these epidemics have occurred only
in undcrdcvcloned countries.

Other factors that influence the rate ofhealing:


. Location of the wound: wounds in an area in which there is a good vascular bed
hc'al considerably more rapidly than wounds in an area which is relatively avascular
. Phl sical factors: severe trauma to tissue is a deterent to rapid woLlnd healing. The

local temperature in the area of a wound influences the rate of healing. ln environ
rnental hyperthemia, rvound healing is accelerated; while in hypothermia, healing is
dela1,ed.

. Circulatory factors:

anemia and dehydration have been found to delay the healing

of q ounds

. -{ge of patient: wounds in younger


r ounds in elderly persons

persons heal considerably more rapidly than

.Infection: bacterial invasion will retard healing


. Hormonal factors ACTH and cortisone are substances that have been known to interlere rvith the healing of wounds. Diabetes mellitus (in.sulin deJiciencl) is one ofthe
most $ idely recognized diseases in rvhich there is significant, clinically evident. retardation in repair of wounds.

\utritional factors

such as the amount ofprotein a patient is consuming is one ofthe most


rmpofiant factors which may effect the speed ofwound healing. Hypoproteinemia has been
shos'n to delay wound healing, while having a high protein diet has been shown to accelerate u ound healing. Vitamins, especially Vitamin C, have been shown to be important in
proper wound healing.

. 3 mm well-encapsulated fibroma
. Necrotizing sialometaplasia ofthe hard palate
. 2 mm papilloma of left commissure of lips

. Aneurysmal bone cyst

Copyrighr O 20ll-2012 " Dental Deckr

epestien bar virus

. Nasopharyngeal carcinoma

. Oral hairy leukoplakia


. Burkitt's lymphoma
. Koplik spots

Copright O 201l-2012 - Denral Decks

A biopsy is a procedure perfonred to remove tissue or cells from the body for examination under a microscope. Biopsies are usually performed to detemine whether a tumor is
malignant or to detemine the cause of an unexplained infection or inflammation.
When the entire tumor is removed, it is called excisional biopsy technique.
tion ofthe tumor is removed, it is called incisional biopsy technique.

lfonly

por-

An incisionaf biopsy (also called diagnostic biop.st) is done when lesions are too large
to excise initially without having established a diagnosis or are of such a nature that excision would be inadvisable.
An excisional biopsy is preferred ifthe size ofthe lesion is such that it may be removed along
u ith a margin ofnomral tissue and the wound can be closed primarily. Erample: A l -cm exophl-tic mass flfrich is a lesion that grovs ouf,rordfrom an epithelial surface) on the cheek.

\ote*

l. Biopsy

is the most reliable technique to diagnose soft tissue lesions.


2. The fixative ofchoice to preserve biopsy specimens lor routine histological

examination is l0%o buffered formalin.


3. The scalpel is the instrument of choice since it cleanly removes the tissue
and does not dehydrate it as cautery or the high-frequency cutting knife may.
.1. The rationale for surgical removal and biopsy of a large periapical lesion
suspected to be of inflammatory origin is that a clinical diagnosis can be confirmed microscopically. This is the only way to distinguish between a granuloma and a cvst.

The Epstein-Barr virus (EBV) is a member olthe herpes virus group. It causes infectious
mononucleosis and has been associated with the subsequent development of two forms
of cancer: Burkitt's lymphoma and nasopharyngeal carcinoma. EBV is also associated rr ith hairy leukoplakia, an opportunistic infection resulting in white patches ofthe
Iareral tongue. Hairy leukoplakia is a nonmalignant lesion seen almost exclusively in
AIDS patients. The virus specifically infects Blymphocytes and some epithelial cells.
Ir is associated rvith the production of atypical lymphocytes and lgM heterophile antibodies identified by the heterophile test (also callecl the mononucleosis spot lest).
This antibody eventually appears in the serum of more than 80% ofthe patients with irteL-rious mononucleosis, hence it is highly diagnostic of the disease

\otes

l.There are no specific oral manifestations ol infectious mononucleosis, although secondary Iesions do occur
2. Neck srvellings are cl.raracteristic ofinfectious mononucleosis, Hodgkin's disease and tuberculosis.

Remember:

. Rubella viruses cause German measles (rubella),which present with


'ash (flat, pink spots on theface and then spreads to other bodl,parts).

characteristic

. Paramyxoviruses can cause measles (rubeola) and mnmps. Rubeola is characterized


by fever. malaise and by the formation of Koplik's spots in the oral cavity. These spots
are small. bluish-white lesions sunounded by a red ring. They cannot be wiped offand
occur opposite the molars. Mumps cause enlargement of the parotid glands. Serious
complications include deafness in children and orchtlis (in/lammation ol the testis) in
males past puberty.

Misc.

Epithelioid cells and giant cells are derived from macrophrges


and are important in the development of:

. Initial inflammation

. Granulomatous inflammation
. Acute inflammation

Subacute inflammation

CopyriSht O 201l -201: - Dental Decks

ORAL PATHOLOGY

Misc.

A patient you saw yesterday had minor swelling of the submandibular space
associated with a carious #31. You prescribed amoxicillin and sent him home.
II called today to say the swelling has gotten worse. You squeeze him into the
schedule and notice that he has trouble breathing. You call an arnbulance to
escort him to the ER and tell the paramedics that he has Ludwig's angina,
Ludwig's angina is a severe and spreading infection that involvs the:

. Submental

and sublingual spaces only

Submandibular, submental, and sublingual spaces unilaterally

Submandibular and sublingual spaces only

. Submandibular, submental,

and sublingual spaces bilaterally

59
Copyrighr (}

20ll

20ll

DentalDecks

Granulomatous inflammation is a subtype of chronic inflammation and is characterized


by granulomas, which are nodular collections of specialized macrophages referred to as
epithelioid cells. A rirn of lymphocytes usually sunounds granulomas. Granulomas are
produced by multinucleatd giant cells (Langerhans giant cells and./brcign bod1, giant
cells). All the other cell types characteristic of chronic inflammation, including plasma
cells, eosinophils, and fibroblasts, may also be associated with granulomas.

Note: Granulomatous inflarnrnation is characteristically associated with areas of


caseous necrosis produced by infectious agents, particularly Mycobacterium
tuberculosis,

. Tuberculosis:

caused by the inhalation of Mycobacterium tuberculosis. Oral nonhealing chronic ulcers lollow lung infection. Important: Characterized by caseating
granufomas with multinucleated giant cells (Langerhans giont cells andJbreign body

giqnt cells).
Etiologic agents associated with granulomatous inflammation;

.
.
.

Infectious agents
- TB and leprosy, which are mycobacterial diseases

Fungal infections (blastomyco,sis, histoplasmosis, qnd coccidioidom)'cosis)


Spirochetes (Treponema pallidum, which causes syphilis)
Cat scratch disease (caused h; an unnamecl gyam-negdive organisnt)
Foreign material (e.g., suture or talc)
Sarcoidosis (unknown etiology; it is non-necrotizing)
Crohn's disease (il ls non-caseating)

Lud$ ig's angina oflen results fiom an odontogenic infection. As a result, the bacteriologl of theses infections generally involves oral flora, particularly anaerobes. Other recounized etiologies of Ludwig's angina include poor oral hygiene, IV drug abuse, trauma,
and tonsiliitis.

It

is characterized by:

. Rapid onset
. The three facial spaces are involved bilaterally
. Board-like swelling offloor olthe mouth and no fluctuance is present
. Tlpical "open mouthed" appearance
. FIP\ siiAn of the tnnore

. Drooling. trismus, and fever


. Ditllculty eating, swallowing, breathing
. Tachvcardia. increased respiration rate
. Can lead to glottal edema; asphyxiation

-\inr av management, massive antibiotic coyerage (IV),

and surgical incision and drainage

are the mainstays oftreatment.

Verv important: The most serious complication of Ludwig's angina is edema of the
glottis (i'hich is a slit-like opening behreen the true vocal corcls).

. Inflammation
. Administration of hydroconisone
. Necrosis

. Suppuration
. Pregnancy

60
Coptrighr O

201

l-2012 -

Dflld

Decks

. l-2 mglkg
. 5- l0 mg/kg

.8-10 mg/kg

. t2-15

mg/kg

61

Copright O 20ll-2012 - Denbl Deck

The ESR is the rate at which red blood cells settle out in a tube of unclotted blood, expressed in millimeters per hour Blood is collected in an anticoagulant and allowed to sediment in a calibrated glass column. At the end ofone hour, the lab techniciar measures the
distance the erlthrocytes have fallen in the tube. Elevated sedimentation rates are not specific for any disorder but indicate the presence of inflammation. Inflammation causes
an alteration ofthe blood proteins which makes the red blood cells aggregate, becoming
heavier than normal. The speed with which they fall to the bottom ofthe tube corresponds
to the degree of inflammation.
ESR rises during:
. lnflammation

. Tissue degeneration

Suppuration

. Necrosis
l\Jote: Certain non-inflammatory conditions, such as pregnancy, are also categorized by
hish sedimentation rates.

***

Death may result from ingesting as little as 2 g offluoride in an adult and l6 mg,&g in children. Symptoms may appear with 3-5 mg/kg offluoride.
S)

mptoms of acute fluoride poisoning include nausea, abdominal pain, vomiting, diarrhea,

conr ulsions. and hypotension.


The treatment for acute fluoride poisoning includes: 1) Call poison control center, 2)
iigns. 3) Initiate basic life support as needed and 4) Get patient to the hospital

Monitor vital

Fluoride poisoning may be ac\te (caused b1' a single larye close offluoride) or chronic (caused
hr long-term ingestion offluoride). The characteristic signs ofchronic fluoride poisoning are:

l. Osteosclerosis of the bones: which results fiom long-term ingestion of water with l0
to 15 ppm offluoride.
:. Dental fluorosis (enamel hypoplasia): which is due to fluoride intake during the calcification stage oftooth development. This can occur in permanent and deciduous teeth.

\ote: [t has been estimated that the average American diet contains about 0.2 to 0.3 mg of
fluoride per day. lf I ppm of fluodde is added to the drinking watel about I to 2 mg of fluoride will be added to the diet daily. Balance studies have shown that when quantities offluoride ingested do not exceed 4 to 5 mg daily, little is retained by the body. The finding indicates the safety of the preventive dentistry programs based on the addition of fluoride to
drinking water in concentrations of approximately 1 ppm.

Important: Fluoride normally accumulates slowly in bones as a person ages. However, if


ingested in very high amounts, it accumulates rapidly. The intake of calcium in high doses
rvill reduce the absomtion ofdietarv fluoride.

. Hereditary

. Medications
. Tooth decay
. Mouth breathing

62
Coplright O

. Tongue

201 1-201 2

, Dental Decks

and mandibular alveolar ridge

. Buccal mucosa and pharyngeal pillars


. Palate and maxillary gingiva
. Mucobuccal fold of lower lip

63
Copyriglr

201l-2012 - Dntal Decks

Xerostomia (dry nouth) is not a disease, however, it can be a symptom of certain diseases. Many times xerostomia is caused by failure ofthe salivary glands to function normally, but the sensation can also occur in people with normal salivary glands. Xerostomia
can cause health problems by affecting nutrition, as rvell as psychological health. At its
most extreme, it can lead to rampant tooth decay and periodontal disease.
Perhaps the most prevalent cause

of xerostomia is medication. The main culprits are


antihistamines, antidepressants, anticholinergics (e.g., atropine and scopolamine),
anorexiants, antihypertensives, antipsychotics (e.g., chlorpromazine and prochlorperazine), anti-P arl<tnson agents, diuretics and sedatives.
The most common disease causing xerostomia is Sjiigren's syndrome /S.9/, a chronic
inflammatory autoimmune disease that occurs predominantly in postmenopausal women.
Sarcoidosis and amyloidosis are other chronic inflaurmatory diseases that cause xerostomia. Other systemic diseases that can cause xerostomia include rheumatoid arthritis,
sl stemic lupus erythematosus, and scleroderma. Remember: Xerostomia is the most
common toxicity associated with radiation therapy to the head and neck.
Comrnercially available saliva substitutes in general, contain an agent to lncrease vlscosiq. such as carboxymethylcellulose or hydroxymethylcellulose, minerals such as calcium
and phosphate ions and fluoride, preservatives such as methyl- or propylparaben, and fla\ oring agents. Examples include: Xero-Lube,i Salivarti and Optimoist.s

\lalignant melanoma accounts for only 4% of all skin cancers; however, it


ofskin cancer rclatcd dcaths worldwide.

causes the

createst numbcr

Clinical features:
. Malc' morc oftcn than fcmalcs

. L sually palalc or rnaxillary gingiva


. 5th decade or older
. A. B. C. D's of melanoma:

-\ = asymmctry
B = border inegularity
C : color variability (brown, black, blue, gray, pink)
D = diameter of /o/ief greater than l/4 inch
Excessive exposure to UV radiation from the sun may bc the primary cause ofmclanoma.
\falignant melanoma has bcen linked to both a lot of sun exposure over a lifetime and to
painful sunbums during childhood.

\{alignant mclanoma is an uncommon neoplasm ofthe oral mucosa. It exhibits a definite


predilcction for the palat and the maxillary gingiva/alveolar ridge. Unfortunately, oral
mucosal melanomas have a dismal prognosis. See picture #36 in booklet

\ote: A nevus

is a mole. Almost all moles are normal. Atypical (dl,splastic) nevi are unusual moles that are gencrally larger than normal moles and are eithcr flat or have a flat part.
They have irregular bordcrs and oftcn are variable shades olcolor, particularly brown. The
presence ofdysplastic nevi may mark a greater risk ofmalignant mclanoma developing on
apparently normal skin. See picture #46 in booklet

Name the malignant, epithelial cell tumor thrt chancteristically begins as


a papule and enlarges peripherally, developing a central crater that
erodes, crusts and bleeds. An example ofthis tumor is shown below.

Copyrighr C 20ll-2012 - Dental Decks

ORALPATHOLOGY

A 62-year-old patient comes into your oflice complaining of his loose lower
molars. The health history reveals recent lower back pain. On a hunch, you
send him down the hall to the orthodontist to take a lateral skull radiograph
which reveals "punched-out" radiolucencies. You will refer this patient to the
physician suspecting a diagnosis of:

. Non-Hodgkin's lymphoma

. Hodgkin's lymphoma
. Multiple Myeloma
. Langerhan's Cell Disease

Basal cell carchoma is a common low-grade skin cancer that rarely metastasizes, but the
local invasion by direct extension destroys the underlying and adjacent tissue. lt frequently
develops on the exposed surfaces of the skin, face and scalp in middle-aged or elderly
persons, it is very rare in mucosa. The primary cause ofthe cancer is excessive exposure
ofthe sun or to x-rays.

Clinical features:
. Presents as a nonhealing, indurated chronic ulcer

.
.

Males are more affected than females


4th decade of life or older
. Located primarily on sun exposed areas ofhead and neck with the nose being the most
common srte

Risk factors: childhood sun exposure, blistering sunbums, fair skin, blue eyes, blonde
or red hair.

Important: It is locally destructive and metastases are exceptionally rar.


The treatment for basal cell carcinoma is eradication ofthe lesion, often by electrodessication or cryotherapy. Recurrence is unconmon ifproperly treated.
r'-ote: MOHS micrographic surgery is the most effective and state-of-the-art procedure
even if the skin
for skin cancer today, which offers the highest potential for recovery
cancer has been previously treated by another method and recurred. With the MOHS technique. physicians are able to see beyond the visible disease, to precisely identify and remoVe the entire tumor layer by layer while leaving the surrounding healthy tissue intact
and unharmed. As the most exact and precise method of tumor removal, it minimizes the
chance ofre-growth and lessens the potential for scarring or disfigurement.

f.r lso known as "Plastna Cell Mveloma') is a disease ofabnormalplasma cells that
mosr often build up in the bone marrorv. The cells form tumors in many bones ofthe body.

\Iultipfe mleloma
Clinical Features:

. \Ien 2:l

(40-70 rears o/r// Vcrtcbrae, ribs, and skull are most frcquently involvcd; pain in
lumbar or thoracic region is a common early symptom.
. Jaws are rarely a primary site, but become involvcd in 70% of the cases, molar ramus area
most common site. Symptoms include swelling, pain, loosening ofthe teeth, and paresthesia.

Radiographic features: variable; slight demineralization to extensive dcstruction, characteristic


finding rs multiplc. small, discreet "punched out" radiolucencies in involvcd bones ln a patient
:u:pccted ofhaving multiple myeloma. a lateral skull radiograph is best to confirm the diagnosis.

Laborator-'-

fi

ndings: important in establishing diagnosis:

. Abnormal monoclonal immunoglobulin protein


is usualll

ofthe IgG or IgA class. with

. Lrinary monoclonal light

peak known as an M spike. The immunoglobulin


monoclonal Jight chain cofrponent.

chains, so-called Bence Jones

proteinu

. Anemia
Treatment and Prognosis: chemotherapy, radiation; prognosis poor.

Important: A form ofamyloidosis occurs in l0% of multiple myeloma patients. It is due to the formarron of complex proteins in which immunoglobulin light chains are precursors. Amyloid ptotein is deposited in various organs an can lead to organ dysfunction fspeciallr kidnq', hedfi, GI ltucl, liver and
I Plasmacytoma is a localized collection ofmonoclonal plasma cells. The discasc is dividcd

\61*,-'intoprimaryplasmacytomaoftheboneandextramedullaryplasmacytoma.Thcimportance

-i.';$t:l:
,.:rr' ofthe diagnosis restswith thc potential forthese disorders to progress to multiple myeJoma.
2 Primary presentation of intraoral non-Hodgkin's lymphoma is uncommon. In gencral,

the oral manifestations occur secondary to a morc widespread distribution throughout the
bodY
3. Hodgkin's disease involving the oral cavity is also considered a raity, but it still can appear here initially.

. Superficial spreading
. Lentigo maligna
. Acral lentiginous
. Nodular

Cop"ighi

@ 201 I

66
'2012 - Dental Decks

. The presence of Nikolsky's sign

. The presence ofnodules


. The presence ofregional lymph node involvement
. The presence of erythroplakia

CopFight O 20ll-2012 - Dental Dcks

Malignant melanoma involves the cells /melqnoc),tes) that produce pigment (zrelarirl, which
is responsible for skin and hair color Melanoma can spread very rapidly and is the most deadly
form ofskin cancer. lt is the leading cause of death from skin disease. Melanoma may appear
on normal skin, or it may begin at a mole frevusl or other area that has changed in appearance.
Some moles present at birth may develop into melanomas. The development of melanoma is
related to sun exposure, particularly to sunbums during childhood, and is most common among
people with fair skin, blue or green eyes, and red or blonde hair.
There are four major types ofmelanoma;
I . Superficial spreading melanoma is the most common type of melanoma. It is usually
flat and irregular in shape and colol with varying shades ofblack and brown. lt may occur

at any age or site and is most common in Caucasians.


2. Nodular melanoma usually starts as a raised area that is dark blackish-blue or bluish red,

although some lack color Poorest prognosis,


3. Lentigo maligna melanoma usually occurs in the elderly. It is most cornmon in sundamaged skin on the face, neck, and arms. The abnormal skin areas are usually large, flat,
and tan with intermixed areas of brown. DeveJops from pre-existing lentigo maligna

(Hutchinson freckle).
Acral lentiginous melanoma is the least common form of melanoma. It usually occurs

,1.

on rhe palms, soles, or under the nails and is more common in African Americans.

Important: Malignant melanoma exhibits either initial radial (do not metastqsize) or yertrc^l lnetastasis mdf occur) gtowthpattems within the skin. Radial growth phase melanomas
include the superficial spreading, lentigo maligna, and acral lentiginous melanomas. Vertical
qro\\th is characteristic of nodular melanoma.

\ote: Skin cancer is a very common malignancy in the United States. Olthe different types,
basal cell carcinoma is most common followed bv souamous cell carcinoma and the least
common is malisnant melanoma.

Clinical staging of malignant neoplasms:


T:-Nl method of assessing the prognosis and thempy of malignant neoplasms is based
upon 1) the size ofthe primary tumor, 2) the presence ofregional lymph node involvement
and 3) the presence of distant metastases. This is represented as:
The

.T

.
.

size of the primary tumor


= presence ofregional lymph node involvement

\I :

\ote:

presence ofdistant metastases

The use of this classification system is becoming less common. A major shortcorring ofthe TNM classification is the lack ofquantification of depth/penetration ofthe

umor.

. Dorsum
. Ventral surface

'Tip
. Posterior lateral border

68
Coplrighr O 201l-2012 - Dnral Decks

Squamous cell carcinoma

ofthe palate

. Squamous cell carcinoma of the nasopharynx


.

Squamous cell carcinoma

ofthe oropharynx

Squamous cell carcinoma

ofthe maxillary sinus

69
Coplaight

201l'2012 - Dental Dcks

Squamous cell carciloma i'SCC) accounts for greater than 9070 oforal cavity and oropharyngeal malignancics. Lip carcinomas account for 25,q/o b 30o% ofall oral cancers. Approximately 909/0 occur on the
lower lip due to increascd sun exposure. IntraorallJ', the most common site is thc posterior lateral border of the tongue, it is particularly uncommon to have lesions develop on thc dorsum or in the tip ofthe
tonglle. Metastases from tongue cancer are relativcly common at thc time ofprimary trcatmcnt. In gcneral, metastatic deposits from SCC ofthe tongxe are found in the lymph nodcs ofthe neck. usually on
the ipsilateral side. The first nodes to become involved are the submandibular orjugulodigastric nodes
at the angle ofthe mandiblc. Noter The floor of the mouth is the second most common intraoral lesion

of SCC.
f,liolog/

Clinic!l Chrrrcterirtics
NIen betwee. 50 10.

Lip
surlighl- pjpe

Tongue

rermil-

Lo*er liP,

lion oflower lip. chronrc

nonhe!liig ulcer, keratotic

Upper lip

Men oler 60. posterio.


Irter|l border . indurared
non healing ulcersith

Fair;

leukoplak'a, erytbroplakia

indurated ulcer: leukoplakia.

Me! in their 70

s:

painless

and girgiva

exophyl'c'nas verrucons
Soft palate-10-20 %

asympbmatic. red or sh'te


plaqres as ul.erared and

Locrtion
\asopharyrL\

Incidence

Etiology

Less than 2% of all Tobacco, alcohol


cancers in fhe U.S.

Clinical Charrcteristics

Treatment

Men 30-40 years old, roof


or latcral wall is most common site, cervical mass,
earache sore throat, nasal

Su.gery and

irradiation

obsnucion
Palate

l0% ofalloral

Men over 60, soft palatc is


more common than hard
palate, painful ulce., Ieukoplakia, cxoph'1ic mass

Surgery and

Tobacco, alcohol

Men over 50. sore throat,


dysphagia, paintul ulcer,
cewtcal mass

Surgery and
irradiation

Unknown

Men over 40, chronic


sinusitis, butging of the
palate, loosening of teeth,
paresthesia in cheek

Tobacco, alcohol,

dntue irritation

Orophartr{

l0% ofall hcad


and neck cancers

\laxillary sinus 30% ofallhead


and neck cancers

inadialion

Surgery and

iradiation

. "Horizontal" growth

phase

. "Circular" growth phase

. "Vertical" growth

. "Radial" growth

phase

phase

70
Cop)'righl O 20ll-2012 - Dental Decks

' Lip
. Tongue
. Floor of mouth
. Buccal mucosa

71

Coprrighr O

201 |

-2012 - Dental Decks

Melanomas develop initially as a flat phase without competence for metastasis called
the "radial growth phase". This refers to the initial growth of a melanona in a horizontal plane. It is clinically macular or only slightly elevated. They then may evolve focally
an elevated part, the vertical growth phase, with metastatic competence.
The "vertical" growth phase is the phase that begins when neoplastic cells populate the
underlying dermis. lt is characterized clinically by an increase in size, a change in color,
nodularity and, at times, ulceration. Metastasis is possible once the melanoma reaches this
phase. Overall, patients with vertical growth phase melanomas have a cure rate of 7070.
Note: This phase predoninates in nodular melanoma.

Radial growth phase melanomas, although invasive, have a cure rate that approaches
with surgery alone. Radial growth phase melanomas include the superficial spreading. lentigo maligna, and acral lentiginous melanomas.
1009'o

Generally, the radial growth phase is described by the A,B,C,D's of melanoma:

: asynmetry
: border irregularity
C : color variability (brolvtr,
A
B

black, blue, gray, pink)

D = diameter of (olexl greater than 1/4 inch


Remember: Malignant melanoma is an uncommon neoplasm of the oral mucosa. It exhibits a definite predilection for the palate and the maxillary gingiva/alveolar ridge.
Untortunately. oral mucosa melanomas have a dismal prognosis. The five-year survival
rate for such tumors is < 20olo (/br skin lesions it i,s > 65%o).

Cancer ofthe tongue causes more deaths than do malignant lesions in other regions ofthe head
and neck. This has been attributed to the fact that it is a highly mobile organ that is richly endorved rvith lymphatics and blood vessels which facilitate metastases. IntraorallY. the most

c'ommon sjte is the posterior lateral border of the tongue,

it is panicularly uncommon to

har e lesions develop on the dorsum or on the tip ofthe tongue. Remember: Metastases tiom
ronsue cancer are relatively common at the time ofprimary treatment. In general. meiastatlc
deposits from SCC ofthe tongue are found in the lymph nodes ofthe neck, usuaily on the ipsilateral side. The first nodes to become involved are the submandibular or j ugulodigastric

nodes at the angle ofthe rnandible.

Cancer ofthe lips: lip carcinomas account for 25 to 30% ofall oral cance$. Approximately
900 o occur on the lower lip due to increased sun exposure. Pipe smoking has also been imphcated in the etiology.
Cancer of the lloor of the mouth: is the second most common intraoral location of SCC. lt
occurs most comlronly in the anterior segment on either side ofthe midline, near the orifices
ofthe salivary glands. Metastasis to submandibular lymph nodes is not uncommon for lesions
tbund here. Note: Prognosis is very poor for lesions found here.

Cancer of the buccal mucosa: generally occurs along the plane of occlusion, midway anteropostenorly.

Cancer of the gingiva: is more common in the mandible than in the maxilla, and posterior
siles are seen more frequently than anterior

Note: Overall the 5-year suwival rate for SCC is 45% to 50%; with neck metastasis it is 25%.

ORAL PATHOLOGY

IIIV*

patient is referred frorn his physician to you because ofan


exophytic growth in the maxillary lft mucobuccal fold. Unable to find an
odontogenic source, a triopsy was done. A starry-sky morphology was
found and a dismtl diagnosis of Burkitt's lymphoma was made.
Which virus is thought to be responsible for this lymphoma?

A 47-year-old

. Herpes virus

. Epstein-Barr virus
. Cytomegalovirus
. Human papillomavirus

72
CopyriShr

lil20l

l l0l:

DenralDecks

ORAL PATHOLOGY

Neo

At the

age ofnine, poet Lucy Grealy was diagnosed with Ewing's srrcoms
the jaw. Although rare in fernales, and rare in the jaw, Ewing's sarcoma
most oftn presents radiographically as:

. Multiple

of

radiolucent/radiopaque lesions resembling "cotton ball" or "cotton wool"

appearance

. Multiple "punched-out" radiolucencres


. "Moth-eaten" destructive radiolucencies ol medulla and erosion of the conex witlr
expanslon

. Lytic lesion that rnay

be ill-defined or sharply defined

73
CoDriShr C

20ll l0ll ' Dental

Decks

Burkitt's lymphoma is a high-grade, non-Hodgkin's lymphoma that is endemic in


Africa and occurs only sporadically in North America. It is manifested most often as a
large osteolytic lesion in the jaw (African

fom)

or as an abdominal mass (Non-African

form).

Burkitt's llmphoma is the first human cancer with strong evidence of a viral etiology. The
Epstein-Barr virus (t herpes-4tpe virus) has been isolated from cultures of tumor cells
and patients with Burkitt's lymphoma have high titers ofantibodies against EBV Also, an
antibody against a surface antigen on the tumor cells has been demonstrated,

Note: The Epstein-Ban virus is also associated with infectious mononucleosis, oral
hairy leukoplakia, and nasopharyngeal carcinoma.
Two forms of Burkitt's lymphoma:
1.

2.

African: younger (mean age 3), male predominance, typically involves the jaws.
\on-African: older (mean age I I), no sex predilection, presents most often as an ab-

dominal mass.

***

Both forms are histologically identical.

jaw lesions usually present as expanding intraoral masses on the palate and
ringir,a. Lesions appear as soft tissue nodular masses and many are hemorrhagic.
See picture #37 in booklet. Radiographically, there is a moth-eaten, poorly marginated
destmction ofbone. See picture #38 in booklet

\ote:

The

Eling's sarcoma is an uncommon, highly lethal, round cell sarcoma ofbone ofuncertain origin. The most common sites for Ewing's sarcoma are the pelvis, the thigh, and the
trunk ofthe body. When the jaws are involved, there is predilection for the ramus ofthe
mandible. Ninety percent of Ewing's sarcoma occur between the ages of5 and 30 years.
and over 607o affect males. See picture # 39 in booklet
Pain. usually ofan intermittent nature, and swelling ofthe involved bone are ollen the earliest clinical signs and symptoms of Ewing's sarcoma. Involvement of the mandible ol
maxilla may result in facial deformity, destruction of alveolar bone with loosening of
reerh. and mucosal ulcers. Feveq leukocytosis, raised ESR, and anemia are also present.

Radiographically. the most characteristic appearance is that ofa moth-eaten dstructive


radiolucency of medulla and erosion ofthe cortex with expansion. A variable periosteal
"onion-skin" reaction may also be seen. See picture #40 in booklet
Histologically, it is often difficult to distinguish this tumor from a neuroblastoma or a
reticulum cell sarcoma, however, the cells of Ewing's sarcoma contain glycogn.
The highly malignant nature ofthis sarcoma is reflected in its propensity for metastasis,
especially to lungs, other bones, and the lymph nodes. Multiple method treatment protocols. including surgery or radiation for local control and chemotherapy for systemic micrometastases have dramatically improved the formerly dismal l07o 5-year swvival rate
to a 60olo 5-year suryival rate.

Important: The most common osseous malignancies are osteosarcomas, followed by


chondrosarcomas, fibrosarcomas and Ewing's sarcoma.

ORAL PATHOLOGY

Neo

A {ifteen-year patient presents to his physician because of localized pain


in his right femur and rapidly enlarging swelling. A radiograph
ofthe area shows a "sun-ray" apparance, Which is a likely
diagnosis for this patient based on incidence?

. Chondrosarcoma
. Osteosarcoma
. Sclerodema
. Chronic osteomyelitis

74
Copyrighr..C

20ll':012

- Dental Decks

ORAL PATHOLOGY

Neo

While doing a dental mission trip in f,thiopia, you notice purplisb-brown


nodules on th hard palate of a 32-year-old female patient. You notice more of
these spots on her arms and legs. Given that HMnfection has an almost 59lo
prevalence in urban Addis Ababa, what neoplasm are you suspicious ol?

. Nicotinic stomatitis

. Hemangioma
. Kaposi's sarcoma
. Leukemia

Cop).right ia:

20ll-:012 - Dentd

Decks

Osteosarcomas bl.so called osteogenic sdrcomas) account for approximatcly 20Yo of all sarcomas and, alier plasma ccll rnycloma, arc thc most common primary bone trnnors. Conventional ostcosarcomas involving thc rnandible and maxilla display a predileclion for malcs f629l,,/. Thosc
arising in the jaws have a mean age of34 years. The majority (60o/o) ofnrandibular osteosarcomas
arise in thc body ofthe mandible. In the maxilla, there is nearly equal incidcnce ol-tumors involving the laveolar ridge and maxillary antrum, u,ith feu, a1'ficcting the palate.
Osteosarcomas involving the mandible prcscnt most commonly with swelling and localized pain.
ln some cases, there may bc loosening and displacement of teeth as well as paresthesia duc to
involvement of the inl'crior alveolar nerve. Maxillary tumors display similar clinical symptoms
but nlay cause paresthesia ofthe infraorbital nervc, epistaxis. nasal obstnlction, or cyc problems.
nray be characterized by localized widening ofthe periodontal ligament space of onc or two tccth. Advanced tumors can be visualized as moth-eaten
radiolucencies or irrcgular. poorly marginated radiopacities. See picture #42 in booklet

Radiographicalln early ostcosarcomas

Osteosarcomas are best treated by radical mandibulectomy or maxillectomy, with radiothcrapy


and chemotherapy for rccuncnces, soft tissue extension, or metastatic diseasc. As with most maLgnanl jaw tumors. initial radical surgery resulls in superior survival ratc of 807o compared with
l'o D sunival lbr local surgery. Osteosarcomas ofthc jaws frcqucntly recur l/.;/0 lo 709'0), \Nith as
n'relastalic rate of 25 to 50o/o. The most common sites ofmetastases arc lung and brain.
\ote: Prognosis is better for mandibular tumors than lor maxillary tunors.

\ote: Chondrosarcomas arising in the mandible and maxilla are extren'tely mrc and havc countcd
li'r ipproximately l9lo of all chondrosarcomas ofthe entire body. Cartilagc is produced by tumor
.ells. The most common signs are a painless swelling and expansion ofthe affected bones, re:rltrng in loosening ofteeth or ill-fitting dentures. Radiographically, the lesions vary fiom mothlaten radiolucencies that arc solitary or multilocular to diflusely opaquc lcsions. The trealment is
ir rdc Iocal or radical excision. Note: They are radioresistant.

Kaposi's sarcoma is a malignant ncoplasm ofcndothelial cells. It is characterized by abnormal vascular


profiferation (it is o concer oftl1e lining of blood t'esselsi. It occurs on multiple sites, cspecially thc
lo\ er extremities. lnitial lcsions are srnall, red papules, which enlargc and fuse to form purplc_to-broq'n.
ipLrngl nodules. It spreads to lymph nodes and intemal organs. Note; Human herpes virus 8 has an eti-

oloclc role.
Kaposi s sarcoma is a uniquc form of angiosarcoma and is the most common cancer associated
AIDS patients and has a predilection for the palate.

*ith

Tbree different clinical patterns:

. Clas\ic lfediterranean type


. Endcmic African type

'

mmunodeficiency rJpc (AIDS-tetuted)


Classic Type

Gcogmphy

Afiica

Rare

Endenic

Older nren

Children and adul$

lmmunodeficienc! T\pe

Relari\ely (ommon

Extremilies

Skin tesions

\ote:

Africeo Typc

Mediterranean basin

Oml lesions

Rare

Rare

Common

Olher organs

Occasionally

Occasionally

Frequently

Coulse

Indolent

Prolonged

Prognosis

l:air

Fair

Inrraorally. the hard palate is the most common localion, followcd by the gingiva and buccal mu-

RmemberAIDS is caused by the RNA refovirus, HIV (d&o tdllel HTLV-lll). The HIV infection
acqnircd with IV drrrg use

blood Droducts.

as we1) as

is

by scxual conlact (homosexual and heterosexnal) and contaminated

. Lipoma
. Ranula
. Lymphoma
. Oral lymphoepithelial cyst

76
Coplrighr O 201l-2012 - Dnral Decks

. They may be completely asymptomatic


. The patient is usually aware ofslight
discomfort or pain
. The maxilla is affected far more frequently
than the mandible
. The molar region is predominantly involved

77
Coptright

@ 20 t 1,20

t2 - Denral

Deck

Almost all oral lymphoepithelial cysts arc lcss than 0.6 cm. in diameter at thc timc ofdiagnosis, which
is usually during the teen years or the third decade oflife. The site most colnmonly affected is thc floor
of the mouth, but the lateral and ventral tongue are not uncommon sites ofoccurrcnce. nor is thc soft
palarc, espccially lhe mucosa abovc thc pharyngcal tonsil. This cyst has a clinical appearance similar to
that ofan epidermoid cyst or a dcmroid cyst ofthc oral/phanngcal mucosa, but its growth potential is
much lcss than ihc othcr cysts.
Lymphomas arising rvithin the oral cavity account for less than 59/o oforal malignancies. In the head and
ncck. most lymphomas start in the rcgional Iymph nodes or rvithin cxtranodal lymphoid sites in areas
known as mucosa-associated lymphoid tissues (MALT h'nphonrur./. Within thc oral cavity. lymphoid
tissue is chiefly represented in Waldeyer's ring fu ring oflvnphdtic tissue fonied br lhe tvo palatine
tonsils. Ihe phanngeal tonsil,theli gual tonsil, dnd inteh^ening htnphoid tissre). Ofimponancc rclative to lymphoma is that in both nodal and extranodal sitcs, neoplasia can occur A ftrndamental aim is
Io separatc lymphomas into two grollps:

. Ilodgkin's t)pe:

is charactcrizcd by painless enlargcmcnt of lymph nodes or extranodal tissue.


\\'ithin Ihc oral cavity, tonsillar enlargement. usually unilateral. may be seen in the carly phascs. His-

tologicalll-, cominon to all forms ofHodgkin's diseasc is the prcscncc ofmalignant l)'rnphoid cells and
non-ncoplastic inflamrnatory cells. including Iymphocytes, macrophages, eosinophils. and plasma
cclls. Ofgreatest significance is thc identification ofthe Reed-Sternberg cell, which must be present fbr thc diagnosis of Hodgkin's disease to be established. Note: The Ann Arbor system of clinical stsging is uscd to determine the clinical extent ofthe disease and is an impofiant factor fbr dcciding
!he tJ-pe and intensity oftherapy and helps dcterminc thc prognosis.
. \on-Ilodgkins tlpei when primary oral soft tissues lesions are present (\'hich i.t rare). thcy are
charactcnzed by an absence ofsymptoms and by a relatively soft charactcr, oftcn $'ilh overlying r.rl.erations. If bone is the primary site. alveolar bone loss and tooth mobility are oftcn prcscnting
.rgns. S\!elling. pain, numbncss ofthc lip. and pathologic fracture may also be associatcd with bone
lesrons. In general. thcrc arc t\r'o groups ofnon-Hodgkin's lymphona noddar (bllicular"/ and diffuse
ibrms. For non-Hodgkin's Iymphomas. numerous classification schemes have cvolved. Thcsc in;lude the classifications ofLuke and Collins, the WHO, Rappaport, and thc NIH International Worklns Fonnulation.

***

This is false; the mandible is affected far more ftequently than the n.raxilla.

The most cornmon malignancy affecting skeletal bones is metastatic carcinoma. Howe\ er. netastatic disease to the mandible and maxilla is unusLra'l (onl.v about I %o). Most importantll, a tumor of the jaws may be the first evidence of dissemination of a kno$'n
rurlrrr from its primary site.
\letastases to the jaws most commonly originate from primary carcinomas of the
breasr (ade o(arcinona), kidney (renal cell carcinona).lur'g (carcinona), colon (adetr()Ltr(i .) t.t). proslale (adenocarcinona) and thyroid gland (ode ocarcinonn), in decreasing order of fiequency.

\ote:

Clinical features of metastatic jaw lesions:

. Ofder adults (average age is

561,ears)

. The mechanism ofspread to the jaws is usually hematogenous from the primary visceral neoplasm

. \\'ithin

the jar.v, the angle and body of the mandible are more commonly involved
Bone pain, loosening of teeth, lip paresthesia, bone swelling, gingival mass, and
pathologic fracture may be clinically evident

. The radiographic appearance of mostjaw metastases is poorly marginated' radiolucent, irregular, moth-eaten, expansile defects

\ote: Microscopicalll',

the diagnosis of metastatic carcinoma in difficult cases can be


for cytokratin, which is present in all carci-

r erified u,ith an immunoperoxidase stain

noma cells.

Important: The prognosis for patients with metastatic carcinoma to the jav's is grave,
rvith a disrnal l0olo 5-year survival and over two-thirds dead within a year.

Neo

Explain what is meant by a carcinoma ofthe oral


cavity having the following TNM designation:

Tl,

N2,

Ml

tumour size more than 2cm


lymph nodes cenralateral bilateral palpable not fixed metastasis suspected
metastasis present other than resional lymph node

7A

Coptrighr e 2011 201? Denral Decks

ORAL PATHOLOGY

Neo

A 65-year-old patient ofEast Indian origin presents to your clinic with a


complaint of a lesion on the maxillary alveolar tuberosity. The lesion is a
thick white, exophytic mass with a cauliflower appearance. A social history
reveals that this woman has been chewing a betel-nut concoction from her
native India for over 40 years. \Yhat is the likely diagnosis of this lesion?

. Papilloma
. Erythroplakia
. Vemrcous carcinoma

. Hyperkeratosis

79
Cop)righr O

20ll 201?

Denral Decks

Clinical Staging of Carcinoma of the Oral Cavity


. T = Size of the primary tumor
TX: Primary tumor can not be assessed
TO: No evidence of tumor

Tis: Carcinoma in situ


T1: less than 2 cm in greatest diameter
T2: 4 cm in greatest diameter
T3: greater than 4 cm in greatest diameter

. N = Regional lymph node involvement


- NX: Regional lymph nodes can not be assessed
NO: No clinically palpable lymph nodes, or lymph nodes palpable but metastases
not suspected
Nl: Palpable homolateral lymph node(s), not fixed but metastases suspected
N2: Palpable centralateral/bilateral lymph node(s), not fixed but metastases
suspected
N3: Palpable lymph node(s), fixed metastases suspected

\I

= Distant metastasis

- !tX: Presence of distant metastasis can not be assessed


lfo: No distant metastasis
fI1: Clinical and/or radiographic evidence of metastasis other than

regional

ll mph nodes

-\ r'errucous carcinoma is a well-differentiated squamous cell neoplasm ofsoft tissue of


the oral or laryngeal cavity. The lesion may invade or infiltrate the borders of adjacent
structures but it rarely metastasizes. Verucous carcinoma may transform into an
invasile fom of carcinoma or coexist with other squamous cell carcinomas. It is often
misdiagnosed histologically as a benign lesion.

Clinical Features:
. \{ale predilection
. Broad-based, exophytic, indurated lesion
. Diffuse, whitish, cauliflower or coral papillary mass
. \{andibular mucobuccal fold, alveolar mucosa and palate
. Slo*-growing, continuous enlargement
. Painless
See

picture

#,13

in booklet

\ote: Tobacco and human papillomavirus


tors. It

(subtypes 16 and 18)may be etiologic fachas


a good prognosis.
is treated by surgical excision and

Neo

A 73-year-old patient prsents to your office with complaint of a lasting


ulceration on th right side of his tongue. H has a history olhypertension
and high cholesterol and a 40-year pack history of srnoking. The patient takes
anti-hypertensives and anti-lipidemics and has no allergies. You cannot lind
any sources oftrauma. After two weeks, the ulcer has grown in size. What is
the likely diagnosis of this most common malignancy ofthe oral cavity?

. Adenoid cystic carcinoma


. Mucoepidermoid carcinoma

. Basal cell carcinoma

Squamous cell carcinoma

80
CoDrighr

Lq

20ll l0ll

Denral Decks

ORALPATHOLOGY

Nrv & Mus Disord

While attempting to give in inferior alveolar nerve block, ifyou inject the
anesthetic solution into the capsul of the parotid gland, you may cause a
Bell's palsy like feeling for the patient by anesthetizing the:

Triuern

inrl newe

. Glossopharyngeal nerve
. Hl.proglossal nerve

. Facial nen'e

81
Copynghr C 201 l,:01 2 - Dentat Decks

Squamous cell carcinoma is a malignant neoplasm ofstratified squamous epithelium that is capable oflocally destructive growth and distant metastasis. It is the most common type oforal
cancer, accounting for over 90%o ofall malignant neoplasms ofthe oral cavity. lt is two times
more prevalent in m ales (40-65 years of4ge./. See pictures #44 and #45 in booklet.

Possible sites:
. Lower lip (most common site)
. TottgU.e (posterior leteral border is the most common loccttion, dorsum least common locqtiotr)
. Floor ofthe mouth (least favorqble plognosis)
. Sofl palate (u conmon)
. Gingiva/alveolar ridge

Buccal mucosa

Clinical fatures:
. Early presentation ofleukoplakias and erythroplakias
. Painless ulcer, tumorous mass, or verrucous (papillary gro\tth)
. Occasional loosening or loss ofteeth
. Possible paresthesia ofthe teeth and lower lip
Risk f'actors identified include smoking, alcohol consumption, painful and ill-fitting dentures,
chronic inflammation and the use ofsmokeless tobacco. Important: Tobacco use is the primary risk factor Note: Causes ofgenetic alterations which results in loss ofcell cycle control. abnonnal signaling, increased cell survival, and cell motility include tobacco, human
papillomavirus (subq,pes 16 and l8), and heredity.
Remember: The most reliable histologic criterion for a diagnosis oforal squamous cell carcinoma is invasion,

Bell's palsy is a form offacial paralysis resulting from damage to the facial nrve.It can
strike at any age; however, it disproportionately attacks pregnant women and people who
have diabetes, influenza, a cold, or some other upper respiratory ailment.
a unilateral paralysis ofall facial muscles with loss ofeyebrow and
forehead wrinkles, drooping ofthe eyebrows, flattening ofthe nasolabial furrow, sagging
of the comer of the mouth and the inability to frown or raise the eyebrows. The upper
and loler lips may also be paralyzed on the side affected.

Clinical signs include

.{tier its sudden onset the paralysis begins to subside within two or three weeks, and gradual. complete recovery occurs in over 85yo of patients.

Triggering events related to Bell's palsy are acute otitis media, atmospheric pressure
change. exposure to cold, ischemia of the facial nerve near the stylomastoid foramen,
\lelkersson-Rosenthal syndrome, and multiple sclerosis. Note: Melkersson-Rosenthal
slndrome is the term used when cheilitis occurs with facial palsy and plicated tongue.

A 2s-year-old college student comes into your olfice cornplaining that when she
wakes up she has trouble opening her mouth. When conducting a TMJ exam,
you note tenderness ofthe right latral pterygoid and non-reciprocal clicking
of the right TMJ. What is the most likely cause of the patient's myofacial pain?

. Trauma
. Muscle spasm
. Periodontal disease
. Tumor

a2
Copyrighr O

ORAL

20ll'2011-

Dental Decks

PATHOLOGY

Nrv & Mus Disord

A S3-year-old patient comes to your o{Iice and notes that sometims when
he swallows, he gets a sharp 'jolt" on the right side of his throat. He says the
pain is seyre and he can even feel it in his ear. The most likely diagnosis is:

. Postherpetic neuralgia
. Orolingual paresthesia

. Frey's syndrome
. Glossopharyngeal neuralgia

83
Coplriglrt C

20ll ?012

Denral Decks

Such spasm may arise in one ofthree ways: muscular overextension, muscular over- contraction or muscle fatigue. The most frequent cause ofthe spasm seems to be muscle fatigue. This syndrome is seen predominantly in women, usually in the 20 to 40 age range,
and generally occurs unilaterally.

There are four cardinal signs and symptoms ofthe syndrome:


1. Pain
2. Muscle tendemess
3. A clicking or popping noise in the TMJ
4. Limitation in jaw motion (especially in the morning)

The pain itself is usually unilateral and is described as a dull ache in the ear or preauricular area, rvhich may radiate to the angle ofthe mandible, temporal area or lateral cervical area. The muscle most apt to exhibit tenderness is the lateral pterygoid muscle.

\ote:

There are no radiographic findings associated with MPS (mtofacial

pain syn-

droDte)

Treatment: Most cases are selflimiting. Soft diet. limited talking, no gum chewing,
moist heat. NSAIDs and Diazepam help relieve symptoms.

Glossopharyngeal neuralgia refers to pain similar to that of trigeminal neuralgia, which


arises frorn the glossopharyngeal newe (CN IX).lt is not as common as trigeminal neuralgia. but the pain may be as severe when it does occur. It occurs in both sexes, in middle-aged or olcler persons and is described as sharp, jabbing, electric, or shock-like pain
located deep in the throat on one side. lt is generally located near the tonsil although the
pajn may extend deep into the ear. It is usually tiggered by swallowing or cherving. It is
almost ahvays unilateral.

Frel 's sl ndrome (also called the auriculotemporal syndrome) ts an unusual phenomenon. * hich arises as a result of damage to the auriculotemporal nerve and subsequent
reinnen ation of the sweat glands by parasympathetic salivary hbers.The syndrome follo* s some surgical operation such as removal ofa parotid tumor or ramus ofthe mandible,
trr an infection of the parotid that has damaged the auriculotemporal newe (a branch of
---l
I
/.

ofFrey's syndrome. The patient


rypically exhibits flushing and sweating of the involved side of the face during eating.

lmportant: Gustatory sweating

is the chiefsymptom

This syndrome is not a common condition.


Postherpetic neuralgia is a persistent burning, aching, itching and hyperesthesia along
distribution of a cutaneous nerve following an attack ol herpes zoster. It may last for a
ferv rveeks or many months. lnvolvement of the facial nerve and geniculate ganglion
produces the Ramsey Hunt Syndrome, which is characterized by facial paralysis and
otalpia karache).

ORAL

PATHOLOGY

Nrv & Mus Disord

Which of the following is a relatively rare autoimmune disorder of


peripheral nerves in which antibodies form against acetylcholine
(ACh) nicotinic postsynaptic receptors rt the myoneural junction?

. Myasthenia gravis

. Myelofibrosis
. Multiple sclerosis

. Graves'

disease

E4

Copynghr C

201

l'201 2 - Dental Dects

ORALPATHOLOGY

Nrv & Mus Disord

A 34-year-old dentophobe is your patient for the morning. After giving a


very hesitant health history, you decide to begin your oral exam, As you
reach towerd her face, she immediately flinches and puts her hands up.
She lets you know that ifyou tough a particular point above her lip,
she gets sharp, stabbing jolts ofpain. You let her know that a
neurologist can work her up for:

. Glossopharyngeal neuralgia

. Trigerninal neuralgia
. Postherpetic neuralgia

. Diabetic neuralgia

Copy.ighr (]

20ll ?011

Denral Decks

Myasthenia gravis is an autoimmune disorder in which antibodies form against acetylcholine (ACh) nicotinic postsynaptic receptors at the myoneural junction. The muscles
are quickly iatigued with repetitive use. lt is tlpical for a myasthenic patient to have a flattened srrile and droopy eyes. with slow papillary light responses. Xerostomia and rampant caries may accompany myasthenia gravis. The acetylcholine that is necessary for the
proper transmission ofnerve impulse is destroyed, with the result that salivary glands do
not receive adequate stimulation. Note: Head and neck manifestations include inability
to focus eyes, drooping eyelids, double vision, difficulty in chewing and swallowing, and
slurring ofwords.

Multiple sclerosis is a chronic, often disabling


lhrain

ancl spinal cord).

disease that randomly attacks the CNS

It is believed to be due to an autoimmune response in which the

inxnune system attacks a person's own tissue. Twice as many women as men have MS,
s ith the onset of symptoms occuring most often between the ages of 20 and 40. Symptoms may range from tingling and numbness to paralysis and blindness. Patients with
multiple sclerosis sometimes have facial andjaw weakness. In addition, both Bell's palsy
and trigeminal neuralgia may develop more frequently in patients with MS.

Trigeminal neuralgia is an excruciating painful illness in which the afl-licted f'eels sudden stabJike pains in the face. The pains usually last only moments, but are among the
most severe pains humans can feel. The pain is provoked by touching a "trigger zone,"
n picallv near the nose or mouth. lt is caused by degeneration ofthe trigeminal nerve or
b1 pressure being applied to it. Pain distribution is unilateral and follows the sensory dis-

tribution of cranial nerve V, typically radiating to the maxillary (I'-2i or mandibular


/ I --l/ area. At times, both distributions are affected. The momentary bursts ofpain recur
in clusters. lasting many seconds. Paroxysmal episodes ofthe pains may last for hours.
\eccssary criteria for diagnosis of trigeminal neuralgia:
. Onser ofpain initiated by a trigger point
. Pain extreme. paroxysmal, lanclnatlng
. Durltron is less thrn 2 minutes
. -{ refractory period experienced for several minutes after attack
. Pain limited to krown distribution of one or more branches of trigeminal nerve with
no motor deficit in that area
. Pain diminished with use ofcarbamazepine ttegretol

Spontaneous remissions occur lasting tnore than six months during the early phase
the disease

of

The dnrg ofchoice for treating trigeminal neuralgia is carbzmazepine (Tegrelor. lt is an


analgesic and anticonvulsant. It is also prescribed in the treatment ofcefiain seizure dis-

orders. When used for the trcatment of trigeminal neuralgia, it usually rclieves the pain
u'ithin 48 hours aftcr treatment is startcd.

non odontogenic cyst

. Median alveolar cyst


. Globulomaxillary cyst
. Nasolabial cyst
. Nasopalatine cyst

86

Copliiglt

O 20l l-2012 -

Dotal Deck

Afl ofthe followlng cysts are congenit EXCEPT one.


Which one is the EXCI'PIIOM

. Thyroglossal duct cyst

. Branchial cyst
. Globulomaxillary cyst
. Dermoid cyst

87
CopynShr O 20ll-201? - Dental Decks

Edited by Foxit Reader


Copyright(C) by Foxit Software Company,2005-2007
The nasolabial cyst has been called, inaccurately, nasoalveolar cyst.
latter designaForThis
Evaluation
Only.
tion is inappropriate because the entity is not a true cyst ofthe maxilla. Rather, it represents a soft tissue cyst without involvement ofthe alveolus, hence the prelerence for the
designation "nasolabial cyst." Note: Because thjs cyst is extraosseous, it is not likely to
be seen on a radiograph.

Ouick reference for develoomental cvsts:


Median AlYeolar - IAIE
I

- teeth vital

*Nasolabial
- in sofi tissue

Nasopalatine
- most common
- heart shaped
Hard Palate

lI
I

Median Palatal
- rare

Thc thl roglossal duct cyst, which may arise fiom any portion of the thyroglossal duct. This
is therefore lound in a midline position and is usually dark in color It may bc vascular as to
rcsenrblc a hemangioma. One lrequent important symptonr is hemorrhage into the mouth, resulting from thc rupture of the overlying veins. Complctc cxcision of the tracl to the base of the
Iongue. frequently including a portion ofthc hyoid bone, is nccessary tbr a curc. Note: Whcn mirration ofthc thyroid gland lails and the gland rernains in thc basc ofthe tongue. it is called lingual thl roid or eclopic lingual thyroid. Lingual thyroids are lour tjmes more comnon in females
than in rrales.
cy st

. The branchial /.1eli/ cyst or cervical lyrnphoepithelial cyst is located in the latcral portion ofthe
ncck. usuallv anterior to the sternocleidomastoid muscle. Thcy may also appear in the submandrbLrlar area. adjaccnt to the parotid gland. or around lhc stcmocleidomastoid muscle. The majorily
rr: lhese t-vpes arise ftom remnants ofthe second branchial cleft or pouch. This cyst is lincd with
.rlified and slratified squamous epithelium and conlains a l11ilky or mucoid fluid. The treatment
aon5isrs ofconlplele surgical excision. Note: The branchial cyst has an intraoral countcrpart known
a: Ihe lt mphoepithelial cyst. The floor ofthe moulh is the most common site lor thcse lcsions, follo\\ ed bv the tongue

. Thc dermoid cyst

is rclativcly uncommon in the oral cavity. This cyst frequcntly contains hair,
\ebaceous and sweat glands, as well as tooth structures. The most common site is thc midline of
the floor ofthe mouth ifabove the mylohyoid muscle.lt appcars as a mass in the upper neck
if belol\ the mylohyoid muscle. The treatment is the surgical removal ofthe entirc tulnor

Important: Globulomaxillary cysts are thought to dcvclop fiom epithelial rcmnants remaining tbllos ingjoining ofthc globular portion ofmcdian nasal process with thc maxillary process. They arc
characterized by a large "pear-shaped" radiolucency between the maxillary lateral incisor and
cuspid. Thcy arc asymptomatic, all regional teeth are vital. They tcnd to cause divergence ofthc
roots. However, thcrc is considerable controversy as to whether this cyst actually exists. Many of
these are, in reality apical cysts associated with non-vital lateral incisors. Olien thcsc apical cysts
havc a tcndcncy to cxtcnd betwccn thc two tccth to simulatc a globulomaxillary cyst.

All of the following cysts are developmenttl (or ftssural)


EXCEPT one. Which one is the EXCEPTIOM

. Nasopalatine duct (canal) cyst


. Nasolabial (nasoalveolar) cyst
. Branchiogenic cyst
. Median palatal cyst

. Median alveolar cyst

88
Copynghr

q 20ll'2012

- Dental Decks

ORAL PATHOLOGY

N-O Cysts

The circular radiolucent area seen in this radiograph is clinically


seen as a marked swelling in the region of the palatine papilla. It is
situated mesial to the roots of the central incisors. The pulps of the
anterior teeth in this patient tested yital. These findings would be
compatible with what diagnosis?

globulo maxillary cyst or incicive


canal cyste

89
Copyrighr C 2011'2011- Dental Decks

. Nasopalatine duct (canal): "hearrshaped" radiolucency in midline, most frequent


type of nonodontogenic cyst. Usually asymptomatic or may produce an elevation in the
anterior part ofthe palate.Teeth are vital, Treatment is enucleation. Note: It is caused by
cystification ofthe nasopalatine duct remnants. Remember: It is also known as incisive
canal cyst and may be located within the nasopalatine canal or within the palatal soft tissues at the point ofthe opening ofthe c anal (in this loccrtion it is called a cyst ol the palatine papilla).

. Nasolabial (nasoalveolar): is superficially located in soft tissues of the upper lip. This
is an extraosseous cyst. Treatment is surgical excision.

. Nledian palatal: rare, may occur any'rvhere along median palatal raphe. May produce
elling on palate. Treatment is enucleation. Note: Many oral pathologists now believe
that this cyst represents a more postedor presentation of a nasopalatine canal cyst .
s$

. \Iedian alveolar: rare. occurs in bony alveolus between central incisors. Distinguished
liom periapical cyst by the fact that adjacent teeth are vital. Treatment is enucleation.

Of the cysts ofthejaw, those that arise from epithelial remnants in the incisive canal are
the most common type of maxillary developmental cyst. Histologically, this cyst is lined
rvith vessels, nerves and mucous glands in the wall. They most often remain limited as to
size and are asymptomatic. Some of them, however, become infected or show a
tendency to grow extensively. When tl'tis occurs, surgical intervention is indicated.

\ote:

The soft tissue (ancl.far less common)

v^ri^nt of the nasopalatine canal cyst

is the

cr st of the palatine papilla.

\\'hen making

a diagnosis

ofthis cyst, the following two cysts should be ruled out:

. The globulomaxillary cyst: usually appears between the roots of the lateral incisor and those ofthe canine. lt is "pear-shaped" and often causes the roots ofinvolved
reerh to diverge. See picture f87 in booklet
. The median palatal cyst: usually situated in the midline of the hard palate, posterior to the premaxilla. Clinically, Ihis lesion presents as a firm swelling, which is usually painless. Note: Some investigators now believe that this cyst represents a more
posterior presentation ofa nasopalatine duct cyst, rather than a separate cystic degeneration ofepithelial rests at the line ollusion of the palatine shelves.
See picture ff48 in booklet

median palatal cyste

so radigraph best to diffrentiate bet max cyste

The soft tissue, and far less common, varlant


the nasopalatine canal cyst is the:

of

incicive canal cyste

. Median mandibular cyst


. Nasolabial cyst

. Cyst ofthe palatine papilla


. Aneurysmal bone cyst

90

Coprighr O 2011,2012, DeninlDecks

An &year-old girl who looks like she is 14 years old comes with her fathor
into your ollice. Her frther states during the health history that she hNs
Mccune-Albright syndrome. Which of the following would you
NOT EXPECT the p.tient to have?

. Heart

disease

. Polyostotic fibrous dysplasia


. Cafe-auJait spots

. Endocrine dysfunction

91

Coplright O 201 I -20 l2 - Denral Decks

The nasopalatin dvct cyst (NPDC) is a developmental non-odontogenic cyst that only occuIS in the anterior maxilla from the embryonic remnants ofnasopalatine ducts. lt is usually
located between teeth #s 8 & 9 but can be as posterior as the mid-palate and as anterior as the incisors
incisive papilla. It is the most common non-odontogenic oral cyst of the oral caviry lt occurs at any age, but is most common in the fourth to sixth decade oflife and is more common
in males. It is usually asymptomatic and is discovered during routine dental examination. If
infected, the patient complains ofpain with swelling and even drainage; however, swelling is
not common. When present, swelling is often in the incisive papilla area in smaller lesions but
can be in the mid-palate in larger lesions and may be buccal and mid-palatal in very large lesions. Radiographically it usually presents as a bilateral, well-circumscribed, round, ovoid
or heart-shaped radiolucency. Tooth displacement or diverging ofthe roots ofthe central incisors is commonly identified. Depending on the size and clinical symptoms, treatment ranges
tiom no treatment to surgical curettage ifthe cyst is infected or interlering with a prosthetic
appliance. Recurence is rare and prognosis is good. Note: The cyst of th palatine papilla
is identical to that of its intrabony counterpart.
The median mandibular cyst, like the globulomaxillary cyst, was once considered a fissural
c.vst. Ho$'ever, embryonic evidence rcfutes thrs (there are no epithelial lined processes).It is
a \ ery rare cyst that appears in the midline ofmandible. Most of them are periapical, lateral
periodontal cysts or odontogenic keratocysts.

Remember: The nasolabial cyst has been called, inaccurately, nasoalyeolar cyst. This lar
ter designation is inappropdate because the entity is not a true cyst ofthe maxilla. Rather, it
represents a soft tissue cyst without involvement ofthe alveolus, hence the preference for the
designation "nasolabial cyst." Note: Because this cyst is extraosseous, it is not likely to be
seen on a radiograph.

-\lbright's syndrome (also cqlled McCtne-Albright s-vtdrcrre/ is the most severe form
of poll-ostotic librous dysplasia. It affects young people (males andJbmales equally). It
is characterized by Caf-auJait spots on the skin, and endocrine abnormalities (the most
conunon ol u,hich is precocious sexual development infemales). The extent to which each
ofthese problems exist in those with the syndrome is quite variable. The hallmark of Albright's syndrome is premature puberty in the female. Early sexual development in the
nrale is less comnon than the female.

Clinical features:
. Early childhood
. \{ultiple, slow-growing, painless expansile bone lesions confined to the craniofacial
area or throughout the skeleton

. endocrine manifestations; in females often sexual precocit-v


.lrregular shaped Cafe-aulait spots on the torso and sometimes intraorally
. Disfiguring
. Increased level ofserum alkaline phosphatase

*** Pathologic fractures

are frequently associated with this syndrome.

There is no specific treatment for this syndrome. Drugs that inhibit estrogen production,
such as testolactone. have been tded with some success.

Important: An additional complication is the malignant transformation potential of


both the polyostotic (nainly) and monostotic fibrous dysplasia into osteosarcomas.

An &year-old chlld has complained of pain on the left sido ofhis head for 5
abnomal findlngs on physical examination. A prnoramlr
radiograph reveals multiple radiolucent lesions on the left side ofthe moxillo.
The lesions give ahe appearance of teeth thot rre "floating in space." Tbe
lesions are sharply circumscribed, with a punched-out appearance.
Which of the following is the most likely di.gnosis?

weeks. There are no

Langerhans cell disease

. Hyperparathyroidism
. Cherubism
. Paget's

disease

92
Cop)rishr O 201I -2012 - Dental Decks

. Is a common lesion and appears more frequently than does the peripheral giant cell granuloma

. Is found predominantly in children

and young adults

. Affects males more than females

.Is

present almost exclusively in the small bones

ofthe hands and feet

93
CoDright A 20ll-2012 - Dental Decks

ldiopathic histiocytosis or Langerhans cell disease, also formcrly known

as

histiocylosis X,

is a disorder charactcrized by a prolifemtion of cells exhibiting phenotypic charactcristics of


Langerhans cells, The clinical manifestations of this process range from solitary or multiplc
bone lesions to disseminated visccral, skin, and bonc lcsions.

Traditionally, idiopathic histiocytosis has been used to encotrlpass thrcc disorders: eosinophilic granuloma, Hand-Schullcr-Christian syndromc, and Lellcrcr-Si$,e disease.
. Eosinophilic granuloma (cltronic loculized Jbnn) has refened to patients with soJitary or rnultiplc bonc lcsions only
. Hand-Schuller-Christian syndrome (chronic listelnitvl?d lbrn.) has represented a specific
clinical triad of lytic bone lesions, exophthalmos, and diabetes insipidus. Many ol thesc paticnts
also cxhibit lymphadenopathy, dermatitis, splenornegaly, or hepatomcgaly.
. Letterer-Silve disease (.rcute dissemitvted fonn) has been charactcrizcd by a rapidly progrcssir e. usually latal, clinical coursc. Widcsprcad organ, bonc, and skin involvement by the prolifLrdti\e lrocc:'s rn Inl'anls has been the comnton presenlalion.

ldiopathic histiocytosis

is generally regarded as a condition

ofchildren and young adults. Oral

initial presentation in all fonns ofthe disorder. Tenderness, pain and srvelling
arc liequcnt patient complaints. Loosening ofthe teeth in thc arca ofthe affected alvcolar bone is
changes may bc the

conlnlon occulrcncc. Thc gingival tissues are frequently inflamed, hypcrplastic, and ulceraled.The
ja\\ s mal exhibit solitary or multiple radiolucent lesions. The lesions frcqucntly ali'ect the alveolar bonc. rcsu)ting in the appearance of teeth that arc "floating in space." Bone lcsions rvitb a
.h:rrpiy- circumscribcd, punched-out appearance may also occu. in the central aspects of the
:randiblc or naxilla.
a

\licroscopicall!, eosinophils
rnuc

lci

are mixed with thc tumor Langcrhans cells. some

ofwhich

arc

mult-

d.

Prognosis is Very good when the disease is localized, however, the disseminated form is usually
:...te1.

The central giant cell granuloma is a benign process that occurs almost etclusively within the jaw
bones. The tumor typically prescnts as a solitary, radioluccnt lesion ofthc mandible or maxilla- It is an
uncommon lesion and occurs less frequently lhan does the peripheral giant cell granuloma. It is
loL)nd prcdominantly in children and young adults. Fcmalcs are atltcted lnorc frcquently than males.
Lcsions occurmorc frequenlly in thc mandible than in themaxilla. These lcsions tend to involvc thcjaws
entarior to thc molar teeth. Thc ccntral giant coll graiuloma typically produces a painlcss cxpansion or

.\\ elling ofthc affected jaw


(-entral giant cell granuloma is classified into aggressive and non-aggressive types; the aggressrvc

lre

tends to occur in younger patients and is known to cause disfiguration, especially after surgcry.

Radiographically, it consists ofa multilocular or, less frcqucntly. unilocular radioluccncy ofbonc. The
r::rgins of the Iesion are relatively well demarcatcd. often presenting a scallopcd border. Roots of the
i3.th nlay br displaced and. less commonly, rcsorbed. See picture #,ll in booklet

Important: Thc lcsion is composed ola proliferation ofspindled libroblasts in a slroma contain ing \ ari.rb.c anlounts of collagen. N{ultinucleated giant cells are ptesent lhtoughout the conneclivc tissuc

\otei Thc differential diagnosis includes


.l !t. and aneurysmal bone cyst.

amcloblastoma. odontogenic mlxoma, odontogenic kerato-

Thc usual treatment is surgcry however. treatment altematives to surgery have emcrged with success_
1u1 results ranging from stcroid injections to calcitonin injcctions or nasal spray to interferon alpha-2a
injections . \\,hich are administered 2-J times pcr rvcck for several months.

.
\otei'
. --' .

L The microscopic appearance ofccntral giant ccll granulona is vimrally identical to ihe
giant cell lesion /B/?rtri /rrrol/ associated lvith hyperparath] roidism. However. blood
tests \r'ill shotv an increase in serum calcium and alkaline phosphatase and a decrcasc in
serum phosphorus in hypcrparathyroidism.
2. The giant cell tumor of bone may present with similar clinical and mjcroscopic features.
However, the Siant cell tunor is regardcd as rare in thejaws in comparison to the cenffal giant
ccll granuloma. They occur most frequently in long bones. See picture #96 in booklet

ORAL PATHOLOGY

N-O Tirm

A 21-year-old male patient is home from colleg and came to you becaus his
"bite seems off " A quick physical assessment seems to indicate that
his chin is deviated to the right. Taking a panoramic x-ray and
comparing to the previous panoramic radiographs you have in his
chart, you notice that the lft condylar neck seems to have elongated.
What condition does this patient most likely have?

. Condylar

agenesis

. Condylar hyperplasia
. Condylar hypoplasia

. Hemifacial microsorria

94
Coplaight

aq

2011-:01l - DenitlDecls

N-O Thm

ORAL PATHOLOGY

A 6-year-old boy is a patient in your practice who has been hospitalized


multiple times for broken bones. During routine lab tests, it was noted that
his alkaline phosphatase levels were quite high. Dentally, you notice multiple
impacted teeth. If this patient has a form offibrous dysplasia,
which radiographic feature may you lind?

. The lesions are usually radiolucent, well-circumscribed, and may have a "cotton wool"
appearance

. The lesions

are usually a saucer-shaped radiolucency

. The lesions

are usually radiopaque, not well-circumscribed, and may have a "groundglass" appearance

. The lesions are usually well-demarcated unilocular or multilocular radiolucencies


95
Copyrighr'-C 2011

20ll

- Denlal Decks

Congenital and developmental anon.ralies ofthe temporomandibularjoint, although relatively rare, are important to identiry early to reestablish normal midface growth centers.
The more common entities include condylar agenesis, condylar hypoplasia, and condy-

lar hyperplasia.
Condylar agenesis is the absence of all or porlions of the coronoid process, condylar
process, ramus and mandibular body. Other first and second arch abnormalities are commonly seen. Early treatment is indicated to limit the degree of deformity, with the primary objective being to re-establish the condylar growth center. This is best done with a
costochondral graft with or without orthodontic surgery and facial plasric augmentation.
Condylar hypoplasia may be congenital, but is usually the result of trauma or infection.
The most common facial deformity is shortness of the mandible with deviation of the
chin towards the affected side. Treatment ofthe child involves the placement ofa costochondral graft. In the adult, treatment involves either shortening of the normal side or
lengthening ofthe involved side. Both result in an acceptable cosmetic and functional result. Onhodontic therapy is necessary in all cases to establish proper occlusion.
Condylar hyperplasia is an idiopathic disease characterized by a progressive, unilateral
o\ergro\rth olthe mandible. The chin is deviated towards the unaffected side. Presentation is common in the second decade. Radiographic findings are usually a normal
condlle but an elongated neck- Treatment depends on whether the condyle is still growing. lts growth is occurring, condylectomy is the treatment- If growth has ceased, orthognathic surgery is performed.

Fibrous dysplasia is an idiopathic condition in which normal mcdullary bone is gradually replaced by
an abnormal fibrous connective tissue proliferation. The mesenchymal tissue contains varying amounts

of ostcoid and osseous material that prcsumably arises through mctaplasia.

The discase most commonly presents as an asymptomatic, slow enlargement of thc involved
bcrne.

It may involve one bone or several boncs concomitantly.

Forms of fibrous dysplasia:

. \Ionostotic: is the most common, comprising 70o% ofcases. There is an equal male to fcmalc ratio,
and this fclrm is the most likely to quiesce at puberty. A t!?ical monostotic lcsion will involve thc
femur tibia. or ribs, uith 25% occuring in the bones ofthe skull. See picture #49 in booklet
. Pol] ostotic fibrous dysplasia applies to cases in rvhich nrore than one bone exhibits evidcncc ofthc
disorder. It is relatively uncommon; however, many patients have lesions of the skull, facial bones,
t r.ia$ s. as a componcnt ofthe condition. Note: Alkaline phosphatase may bc elevated in up to 30%
oipatients $ith polyostotic fibrous dysplasia, and a dramatic dse may herald malignant degeneration.
Calcium and phosphorus tcnd to be normal.

\otes-

l. Craniofacial librous dysplasia is a form offibrous dysplasia charactcrized by maxillary


lesions that extend to involve the maxillary sinus, zygoma, sphcnoid bone. and floor ofthe
orbit.

Itfccune-Albright syndrome is a designation that has been applied to patients with


polyostotic fibrous dysplasia, cutaneous melanotic pigmentation fcafe au lait macules), and
endocrine abnormalities (most commonlr precocious sexual development infenales).
3. Jaffe-Lichtenstein syndrome has been used to describe patients with multiple bone lesions offibrous dysplasia and skin pigmentations.
4. Osteoblastoma is an uncommon primary lesion of bone that occasionally arises in the
rnaxilla or mandible. It is a benign process that may exhibit a seemingly rapid onset and
cause pain. The mandible is the most liequent head and neck sitc. Young adult malcs arc
most commonly affected. Radiographically, is a well-circumscribed lesion that varies fiom
radiolucent to radiopaque. May have a "sun-ray" paftcm ofnew bone production.

. In the retromolar pad region

. On the lingual surface ofthe mandible, most often in the premolar region
. On the lingual surface ofthe mandible, inferior to the mylohyoid ridge
. Along the midline ofthe hard palate

96

CopriglrtO 20ll-2012

- Dnral

D4ks

All of the following are clinical features of the ossifying


fibroms EXCEPT one. Which on is the EXCEPTION!

. Slow growing expansile lesion


. More often in maxilla
. Asymptomatic
. Common in young adults around 35 years ofage

97
Cop}.right O 201 l-2012 -

De

al Decks

Tori and exostoses are nodular protuberances of mature bone whose precise designation
depends on anatomic location. These lesions are of little clinical significance; they are
non-neoplastic and rarely are a source ofdiscomfon. The precise etiology ofthese lesions
remains obscure, although evidence has been presented to suggest that the torus may be
an inherited condition. Note: The etiology ofexostoses is also unklown. It has been suggested that the bony growths represent a reaction to increased or abnormal occlusal
stresses ofthe teeth (i.e., bruxism) in the involved areas.
Mandibular tori (also called torus mandibularis) are bony. exophytic growths that occur
along the lingual surface ofthe mandible superior to the mylohyoid ridge.

Vandibular tori may occur singly, however, there is a marked tendency toward trilateral
occurrence, and the lesion is not necessarily confined to the premolar region. Unlike
palatal tori. the mandibular tori are more readily demonstrated radiographically.

forr huoxillary

and mandibulor) are ofno pathological significance and rarely are they
erf clinical significance r.vhile the normal teeth are still present. If, howeyer, a complete
denture needs to be made, they should be carefully removed.

***

This is false; these lesions, with rarc cxccptions, arise in tooth-bearing regions ofthc
iau s. most ofien in the mandibular premolar-molar area. See picture #89 in booklet
Thc ossifling libroma is a bcnign fibro-osseous lesion of the jaws that is considercd by
nran\ in\estigators to dcvclop fiom undiffercntiated cells ofthe periodontal ligament. The
:imilaritics between this lesion and thc ccmcntifying fibroma are numerous. Both tumors
occur in similar age groups and locations and manifest comparablc clinical characteristics.
The ossif-ing fibroma is a slow-growing, expansile lesion that is usually asymptomatic when
d:scorered. Thcsc lcsions, with rare exceptions, arise in tooth-bcaring regions ofthejaws,
most oflen in the mandibular premolar-molar area. They are uncommon lesions that tend
ro occur in fcmales during the third and fourth decades of life.
The most important radiographic feature ofthis lesion is the well-circumscribed radirrluccncy \r'ith a sharply defined border Thcy prcscnt a variable appearance dcpcnding on
thc maturation or thc amount olcalcification present. As thc lcsion matures, it eventually
becomes a relatively uniform radiopaque mass.
The tumor consists ofa collagenous stroma that contains varying numbers ofunifbrm spindled or stellate cells. Irregular trabeculae of woven immature bone are most consistently
rored in thse tumors. The treatment ofossifying fibroma is most often accomplished by surgical removal utilizing curettage or enucleation. Recurrence is uncommon.

.\ r ariant of ossifoing fibroma, thejuvenile ossifying fibroma,

has been described in children


and teens. This rare lesion behaves in a more aggressive fashion than does the ossifliing fibroma, and it may require more extensive therapy when encountered.

A l?-year-old patient ofyours comes in for a routine examination. A head and


neck examinrtion reveals multiple cysts of the skin. Her panoramic exarns
have always shown multiple impactd teeth and today shows multiple
radiopacities ofthe jaws especially at the angle of the mandible. You
suspect Gardneris syndrome. What complication should she most
be concerned with whn consulting her physician?

. Odontomas
. Osteomas

. Adenocarcinoma
. Epidermoid cysts
. GI polyposis
9E
Copyfl ghr

a., 201

1,201 2 - Dental Decks

ORAL PATHOLOGY

N-O Tirm

A l0-year-old boy comes with ber mother to the dental office because of a
painless swelling ofhis maxilla. Radiographic exam reveals an irregularly
shaped radiopoque mass with a ground glass appearance. No other bulges
have been noted by the mother. A biopsy reveals fibrous tissue in the bone.
What is the most likely diagnosis?

. Monostotic fibrous dysplasia


. Polyostotic fibrous dysplasia
. Albright's syndrome

. Jaffe syndrome

99
Copyrighr,(] 20l

l'2011

Dental Decks

Gardner's syndrome is an autosomal dominant disorder and is characterized by intestinal


polyposis, multiple osteomas, fibromas ofthe skin, epidermal and trichilemrnal cysts, impacted pemanent and supernumerary teeth, and odontomas. The most serious complication of Gardner's syndrome is the multiple polyps that affect the large intestine. The
inevitable outcome ofthis disease is invasive colorectal ancr.

Clinical features:
. Onset early puberty
. Polyps ofthe colon ultimately change into adenocarcinoma by the fourth decade of
life
. Abnormality ofthe retina ofthe eye
. Development olmultiple epidermal cysts usually on face, scalp, and extremities
The oral findings ofGardner's syndrome include:

. Vultiple impacted and supernumerary teeth


. \4ultiple jaw osteomas which give a "cotton-wool" appearance to the jaws. These
osteomas appear as dense, well-circumscribed radiopacities. Osteomas most olten dev-

eiop first rvithin the angle ofthe mandible

. \4ultiple odontomas

***

\\rhen Gardner's syndrome is suspected based on oral findings, the patient should
be referred to a gastroenterologist for consultation. Note: Multiple desmoid tumors
t.tibt onatosis) and epidermoid cysts ofthe skin are also characteristic of the disease.
Remember: Multiple impacted and supemumerary teeth are also seen in Cleidocranial
dl splasia.

Fibrous dysplasia is an idiopathic condition in which normal medulJary bone is gradually rcplaccd by
an abnormal fiborus connective tissue prolifcration. The mesenchymal tissuc contains \ arllng amounts
ofosleoid and osseous mateial that presumably arises through metaplasia.

Thc discase most commonly presents as an asymptomatic. slo$' enlargcmcnt

of the

involved

bone. I! may jnvolve one bone or scvcral bones concomitantly.

. \Ionostotic

fibrous dysplasia: is the most common, comprising 707o of cases. There is an equal
male to female ratio, and this form is the most likcly to quiesce at puberty. A typical monostotic lesion Nill involve the fcmur, tibia or ribs, with 2570 occu.ring in the bones ofthe skull flle mdiil/a r.r
.atnnnlv involved). A panorex will show a radiopaque mass with inegular bordcrs that has a "ground
glass" appearance. See picture #,19 in booklet
. Poltostotic fibrous dysplasia applies to cases in which more than one bone exhibits evidence ofthe
disLrrder. It is rclatively uncommon; horlcver, many patients have lesions of the skull, facial boncs,
or ii\\ s. as a component ofthe condition. Note: Alkaline phosphatase may be elevated in up to 307o
oipalicnts $ ith polyostotic fibrous dysplasia, and a dramatic risc may herald malignant degeneration.
Cal.rum and phosphorus tcnd to be normal.
Important: Fibrous dysplasia has a variable radiographic appearance that ranges from a radiolucent lesron to a densely radiopaquc mass. The classic ptesentation has been describcd as radiopaque with numerc'us bony nabeculae imparting a "ground-glasss" appearance. An important distinguishing feature
L)ifibrous dysplasia is the poorly delined radiographic and clinical margins ofthe lesion The process
iecms lo blend into the surrounding normal bone without cvidence of a circumscribcd border.

\lalignant degeneration occurs in less than l7o ofcases offibrous dysplasia. Malignancies are almost exciusi\,ely osleosarcoma. For unknown reasons, monostotic and craniofacial Iesions have the greatest potential formalignant dcgeneration, and radiation therapy has been found to incrcase the risk by 400-fold.
The differential diagnosis of fibrous dysplasia of the jaws includes the ossilying fibroma, howeyer, radiographically thc ossifying fibroma has a well-circumscribed appearsnce.

\ote:

Remember: N{cCunc-Albright syndrome is a designation that has bccn applied to patients with polyostotic fibrous dysplasia, cutaneous melanotic pigmcntation fcate .r lait macules), and endocrine abnorrnaTlties (rnost cotwtronly precocious sexual

devlopuent in.lbmales).

ORAL PATHOLOGY

Odont Cyst

A new patient walks into your oflice. Your initial physical assessment reveals
that his eyes are set wide and that he has multiple lesions of the skin. When
shaking his hand, you notice that the skin ofhis palm is very thick and has
palmer pitting. When doing a health history, he reveals that he sees a
neurologist and that he has some calci{ied structures "in his brain."
A panoramic radiograph may likely reveal:

. Osteomas

. Odontogenic keratocysts
. Odontomas
. Dentigerous cysts

100
Copynghr i,e

20ll'2011-

Denral Decks

ORALPATHOLOGY

Odont Cvst

A mother brings her 2-year-old boy into the dental oflice because of a "swelling"
on his alveolar ridge. Your exam reveals a smooth-surfaced bluish lesion with
fluctuance where tooth #K will be erupting. The most likely diagnosis ofthis is:

. Dentigerous cyst
. Eruption cyst

. Hematoma
. Hemangioma

10'l
Coprrighl !, 2011-2011 - Denlal Decks

This patient has nevoid basal cell carcinoma syndrome, which is an autosomal dominant disorder characterized by oral, systemic and skeletal anomalies. It is an inherited
group ofdefects which involve abnormalities ofthe skin, eyes, nelvous system, endocrine,
glands and bones. The condition is characterized by an unusual facial appearance and a
predisposition for skin cancer. It is also krown as the basal cell nevus-bifed rib syndrome,
the basal cell nevus syndrome and the Gorlin and Goltz syndrome. Note: Recently, mutations in the human homologue ofDrosophila patched fPICIl), a tumor suppressor gene
were identified as the underlying genetic event in this syndrome.
Possible abnormalities include:

. Cutaneous anomalies: including multiple basal cell carcinomas, other benign dermal cysts and tumors, palmer pitting, palmer and planlar keratosis and dennal calcinosis.

. Dntaf and osseous anomalies: including odontogenic keratocysts (o.ften multiple)


mild n.randibular prognathism, rib anomalies (often bifid), and venebral anomalies.
. Ophrhalmologic abnormalities: including hypertelorism with wide nasal bridge an
congenital blindness.
. Neurologic anomalies: including mental retardation, dural calcification, agenesis of
corpus callosum and congenital hy&ocephalus.
. Sexual abnormalities: including hypogonadism in males and ovarian tumors in females.

Radiographic features:

. Calcification ofthe falx cerebri bbrain


. Presence of odontogenic keratocysts

An eruption cyst is essentially a soft tissue variant ofthe dentigerous cyst. It is invariably
associated with an erupting tooth (asaa lly prim^ry but o(casionally a permanent tooth).
The effects are mostly limited to the overlying gingival tissues rather than bone.
Clinically, the lesion usually appears as a smooth-surfaced, reddish-pink or bluish-black,
tluctuant. Iocalized swelling on the alveolar ridge over the crown ofan erupting primary
or permanent molar tooth. The intense bluish coloq which is often characteristic, is due
ro an accumulation of blood. Due to this appearance, it may be mistaken for a hemangioma or hematoma.

\o

treatment is necessary

as the cyst often ruptures spontaneously. In a few

rare cases,

incision or even the removal ofthe overlying tissue may be necessitated by pain or tendemess associated with lhe lesion.

Odont Cvst

Upon viewing a panorex ofa l4-year-old patient, you see a well-de{ined


multilocular radiolucency with scalloping around the roots on the left side
ofthe mandible rpical to the canine and first premolrr. No clinical symptoms
are present. Teeth are not carious and respond normally to vitality tests.
Mdical history is unremarkable. Upon opening into the area, no fluid or
tissue is evident. What is the most probable diagnosis?

. Dentigerous cyst
. Traumatic (simple) bone cyst

. Primordial cyst
. Residual cyst

. Stalne /s/a/i., bone defect


102
Copyflghr O

20ll 201:

Dental Decks

ORAL PATHOLOGY

Odont Cvst

A healthy l9-year-old patient presents to your olTice for a routine exam. Taking
a panoramic radiograph, you see a well-corticated, unilocular radiolucency
surrounding the crown of impacted tooth #17. The lesion is asymptomatic.
What is the most likely diagnosis?

. OKC
. Dentigerous cyst

. Cystic ameloblastoma
. Central ossifying fibroma

103
Copyaghr,e

20ll

:01:

Denral Decks

Remember: This cyst may be completely devoid ofsolid or liquid material. lt occurc most frequently in younger persons rlith no sex predilection. The usual location is in the mandible bctween the canine and ramus. The regional teeth are vital.

Clinical features:
. Children and adolescents. usually belbre the
. Usually asynlptomatic
. Prinarily seen in mandiblc

. Slight ilun) conieal

age

of20

bonc crpansion

. Associated with vital teeth, no displaced teeth


The dentigerous cyst (or lbllicular c.vst) contains a crown ofan unerupted tooth or denlal anonralv such as an odontoma. Enlarged dentigerous cysls can cause Drarked displaccmcnt oftccth. Prcssure of accumulatcd fluid usually displaces the tooth in an apical direction.
See

picture #50 in booklet

The primordial cyst diflers from the periodontal and dentigerous cysts in that it conlains no calcrlied structures. These cysts are lined by stratified squanous epithelium and may be either unilocular. multilocular or multiple.
The residual cyst rcl'ers to a situation in which a tooth associated with a radicular cyst is cxtractcd
but the c)st is lcft undisturbed, it persists within thejaw and this lcsion is called a residual cyst.
\ote: \bu must curette the socket ofa tooth with a radicular cyst aftcr extraction. See pictures #

86 and #87 in booklet

Ihe st^Ine (stqtic)

bone defect is radiolucency ofthe mandible due to invagination ofthc lingual


.urlace of the of the jaw. lt is locatcd in thc posterior mandible below the mandibular canal.

\ote:

Thc focal osteoporotic bone marrow defect is a radiolucency in the jaw that conlains
11 is olten seen in an extraction site.

henratopoietic bonc marrow.

Clinical features:

.lt

usually contains a crown ofan unerupted tooth


. Usually not clinically visible rvithout radiographs
. .\\\ nrptomalic, occasionally pain or swclling
. Lsually involve uneruptcd mandibular third molars. olher frequenl sites includc ma\illary canines.

marillary third molars and mandibular second premolars


Rrdiographic feature: \\'ell-circumscribed, unilocular radiolucency around crown ol'looth.
=50 in booklet

See

picture

Remember:

The lateral periodontal cyst may bc dcfincd as a non-kciatinized, non-inflammatory dc\ clopmcntal cyst occurring adjacent o. lateral to thc root ofa tooth. See picture #91 in booklet

:.

The fissural cysts ltthich are also called developmental .,l.irtr) are non-dental in oiigin, thcy include nasoah colar. median palatal, and nasopalatine cysis.
-1.

Th!- primordial cyst contains no calcified structures.

Thc traumatic (simple) bone cyst n].ay contain blood, 11uid, dcbris or be completely empt-l.',
Comrnonly found in young persons, in the mandible between the caninc and ramus.

-1.

The residual cyst is often found in edentulous areas. This cyst refers to a situation in which a tooth
* ith a radicuiar cyst associated with it was extracted. and the socket wasn't curelted. The radicular clst persists in thejaw as a residual cyst.
The gingival cyst ofthe newborn has also been designaled as the dental lamina cyst ofthe newborn or Bohn's noduels. Such cysts appear typically as multiple nodulcs along the alveolar ridge
in neonales. They are due to cystification ofrcsts ofthe dcntal lamina. In the vast majority ofcascs.
these cvsts degeneratc and involute or rupture into the oral cavity. Note: Similar epithelial inclusional cysts may occur along the midline ofthe palate fpala/i/te q'sts ofthe nerborn or Epstein's
2earl9. Thcse are ofdevelopmental origin but are not dcrivcd from odontogenic cpithelium. No

treatlnent is necessarv.

. Henwig's epithelial root sheath


. The reduced enamel epithelium

. Remnants ofthe dental lamina


. A pre-existing osteoma

10,t
CopFiShr O

201 I

'2012 - Dnral Decks

. Residual cyst

. Traumatic bone cyst


. Primordial cyst
. Periodontal cyst

105
Copynght O 201l-2012 - Denhl Deck

Keratocysts dillcr fiom olher odonlogenic cysts in their nricroscopic appearancc and clinical behavior They may resemblc periodontal, primordial or follicular cysts. Usually they cannot be distinguished radiographically. See picture #88 in booklet

Clinical features:
. Widc age range. peak

occurrence in 2nd and 3rd decades


[,csions found in children are often reflective of nrultiple odontogenic keratocysts as a componenl ol'the nevoid basal cell carcinoma syndrome
. More common in males than fcmalcs
. The chrefsite ofinvolvcmcnt is thc mandible, in approximatcly a 2 to 1 ratio
. In thc nrandiblc, most occur within the posterior portion ofthe body and rarnus rcgion
. Typically asymptomatic

Radiographic features:

. well-demarcated area ofradioluccncy with


. Unilocular or multilocular
\licroscopicallyi
. The lininging epithclium

a sca)lopcd, radiopaqr,re

margin

is thin and parakcratinizcd

. The basal layer is palisadcd u,ith prominent, polarized, and intensely staining nuclei ofuniform
dramctcr

. The luurcn may contain large amounts ofkeratin debris or clear fluid similar to serum transudalc
. The parakeratotic type forms 85 10 95% ofall odontogenic keratocysts; thc balance is made
op of the .'rthoker.ltinized vaiant. lmportant: The orthokeratinzed variant is less aggressive,
\\ ith a rnuch lorver rate ofrccurrence and is not syndrome-associatcd.

Important: The most remarkable

feature of keratocysts is their great tendency toward recur-

rence.

A primordial cvst arises from cystic changes in a developing tooth bud before the formation ofenamel and dentin matrix. Since the primordial cyst arises from a tooth bud, the
tooth \\'ill be missing from the dental arch unless the cyst arose from the tooth bud of a
supenrumerary tooth. The mandibular third and fourth molar regions are the most
common locations for a primordial cyst. lt is usually found in children and young adults

l0 ard 30 years of age. Radiographically, the primordial cyst is a circular radrtrlucencv * ith a radiopaque border and lound at the site where the tooth failed to debetri .-en

\elrrp. \lany investigators have reported that most prirrordial cysts have the same
aharacteristic l'eatures as those of odontogenic keratocysts. However, until conclusive
prtrtri is established, primordial cysts and odontogenic keratocysts are considered separate
iit trtles.

\\ h.n a tooth having a radicular cyst at its apex is extracted, the radicular cyst is left
behind in bone and is now called a residual cyst. A residual cyst can also rise from remn3nts ofthe epithelial rests after tlre extraction ofa tooth. This cyst occurs in older indir iduals. the average age is 50 years. The radiographic appearance is that of a circular
radiolucency sunounded by a radiopaque border and occurring in an edentulous area. A
residual cyst can easily be misdiagnosed as a primordial cyst. The latter arises in lieu of
a looth r\hereas a residual cyst arises in relation to an extracted tooth.

. Lateral periodontal cyst


. Dentigerous cyst

. Odontogenic keratocyst

106
CoplriSht O 201

1,201 2

- Dental Decks

. Based on symptoms
. Radiographically

. Histologically
. By an electric pulp tester

107

Copright O2011,2012

- Denral Decls

Dendgerous

{follicular cyt)

.
.
.
.

Odontogenic
kmtocysl

Lateral
Periodontal cyst

\otcs,

Radiogmphic Chrracteristics

Clinical Characteristics

Cyst

Children and teenagers


Mandibular third molar and maxillary canine
area 00%o) in nandible)
Associated with impact or unerupted teeth
Sccond most common odontogenic cyst
Usually occurs between the ages of 10-30
Often associated with an impacted tooth
50o/o mandibular third molar area
Over 307o recurrence rate
95% mandibular canine-premolar area
Apposition with rooi ofvifal tooth
Usually symptomless

Well-defined usually
unilocular RL associated with
the crown of !n unerupted
tooth
Well-circumscribed RL with
smooth margins lnd thin
radiopaque borden

Well-defined, round or
teardropshaped RL with an
opaque margin along lateral
surface of tooth

L The lateral Dcriodontal cvst mav be deflncd as a non-keratinizcd. non-inflammatorv developmental cyst occurring adjacent or latenl to the root ofa tooth.
2. Thc gingival cyst ofadulthood is the sofl-tissue oounterpart ofthis lesion.
3. The origin ofthe lateral periodontal cyst is related to proliferation ofthc rests ofdental

lamina.
4. The calcifying odontogelic cyst (COC./ is believed to be dcrived from odontogenic ep-

ithelial remnants within thc gingiva or within thc mandible or maxilla. Ghost cell keratinization is the characteristic microscopic fe.ture of the COC. Radiographical)y, the
COC may prescnt as a unilocular or multilocular radiolucency with discrctc. wcll-demarcated margins.

within

a "salt and peppcr"

the radiolucency, ghost cells may undergo calcification and appear as

typc of pattcm. Note: This cyst has a cutaneous countcrpart known

as

calcifu ing epithelioma of Malherbe or pilomatrixoma.


5. The glandular odontogenic cyst, or thc sialo-odontogenic cyst is rare cyst that occurs
most often within the mandible. This lesion is locallv asgressivc and recurence is common.

Lesion
Dental gaanuloma
(h is one ofthe mosl

Apex oftooth

coamon ofall sequelae

o{p lpiti,
Radicula. cyst
l..llso called apical

periodontal & periapical

fls,

It is the most common


odontogenic cyst

Clinical and
Radiogriphic Characteristics

Usual
Location

Apex oftooth

.
.
.
.
.
.
r
.

Asymptomatic
Circumscribed radiolucency at
apex of tooth
Tooth is non-vital
May be sensitive to percussion

Treatment
Root canal rcatment
or extraction of
involved tooth

Root canal treatment


Asymptomatic
Circumscdbed ndiolucency at with apicoectomy or
extraction wilh
apex of tooth
curettage ofsocket
Tooth i: non-vital
May be sensitrve to percussion

\ecrotic pulp causes periapical inflammation:


. lt acute. a periaprcal abscess forms
. If chronic, a dental granuloma forms
cyst derives its epithelial lining from prolitbration of small odonrogenic epithelial residres (rest of Malcssez) within the periodontal ligament.

\ote: This inflammatory

ORAL PATHOLOGY

Odont T[rm

A 37-year-old patient comes into your office with the complaint of a slowlygrowing, painless swelling of his lower left jaw. A panoramic shows a
multilocular radiolucency with well delined and sclerotic margins along
the left mandibular molar-ramus area. All teeth test as vital. A triopsy is
conducted and shows odontogenic epithelium. What is the most likely
diagnosis for the most aggressive type of this turnor?

SoIid (multicystic or pol!"cystic)

. Unicystic
. Extraosseous

(p

rip heral)

108
Copyrighr'a 201 l-2011 - Dental Decks

Odont T[m

ORAL PATHOLOGY

A 30-year-old patient coms into your oflice complaining ofa painless swelling
of his lower left jaw. A panoramic radiograph shows a welltircumscribed

multilocular radiolucency with a 'honeycombed" pattern at the location


ofthe lower left molars. The teeth have been displaced. The pathology
report calls this an odontogenic mlroma, This tumor:

. Is composed of large polyhedral, neoplastic, epithelial cells

. Is composed ofneoplastic epithelium

and mesenchyme

. Arises from follicular connective tissue resembling pulp tissue

. ls composed of spindle-shaped

mesenchymal cells and aggregates of multinucleated

giant cells

109

coplrighr ill 20ll-201: - Denral Decks

Ameloblastoma consisrs entirely ofodontogenic epithelium, which at sites shows the differentiation ofthc familiar, histologic layers ofthe enamel org.n. It is most often seen in the mandibular
molar-ramus area. Clinically, it is the most aggressive odontogenic tumor. Important: The ameloblastoma is the most common epithelial odontogenic tumor.

Clinical features:
. No sex or racial predilection
. Mean age is commonly between 35 and 45 years
. 2 biologic-microscopic subtypes:

if

treatcd consen'ativcly.
I ) Solid or multicystic: is more aggressive and high recunencc rate
2) Unicystic: is less aggessive and less likely to recu. A histologic variant ofthis type is the plexiform unicystic ameloblastoma.

. Slow growing

and painlcss swclling

. Locally aggrcssivc
. Capable ofcausing large facial deformities

\oter

Teeth are vital

Rarely, extraosseous peripheral ameloblastomas arc found in tlre gingiva and buccal mucosa.
a benign non-aggressive course.

The) exhibit

Radiographic characteristics: Multilocular or unilocular radioluccncy with well defincd and slerotic
margins. Notei In the mandible it appears similar to thc central giant cell granulona. See picture #51
in booklet
Treatment: Each casc should be considered on its own basis. The solid hnullicristic or polvcl'stic) le.ion requires surgical excision. Rescction should be reserved for larger lesions. Unicystic Iesions usu
alh require only enucleation. Thcy should not bc ovcr trcatcd. Note: You nccd to know thc
.rmeloblastoma ertremely well for thc cxam!l!

\olei }lalignant

behavior by amcloblastoma is rarely encountered. These lesions occur in a younger age


group (JOr/ and appcar in the mandible more frequently than the maxilla. Malignant lesions have been
ll\ rded into nvo subtvpes: the malignant ameloblastoma and ameloblastic carcinoma.

The odontogenic myxoma is mesenchymal in nature and origin. mimicking microscopicall! the dental pulp or follicular connective tissue. The mean age in which this lesion appears is J0. Radiographically. this lesion is always radiolucent, although the pattem
mav be quite variable. It may appear as a well-circumscribed lesion or as a diffuse lesion.
h rs often multilocular. frequently with a "honeycombed" pattem- Important: Cortical exltnsion (rother than perforation) and root displacement (rqther lhan resorplion) are the
rule. Surgical excision is the treatment ofchoice. These lesions exhibit some aggressiveness and have a moderate recurrence rate. See picture #90 in booklet

\ote: \\:hen relatively

large amounts ofcollagen are evident, the term


used to designate the odontogenic myxoma.

Ihe central odontogenic fibroma is a rare lesion that is regarded

"myofibroma" is

as the central counter-

part to the peripheral odontogenic fibroma. It is seen in all age groups, and is found in both
rhe n.randible and maxilla. It appears as a well-defined radiolucent lesion that is usually
multilocular, often causing cortical expansion. It is treated with surgical excision and recurrence is very uncommon.

l. The calcifying epithelial odontogenic. (pindborg) ttrmor is composed oflarge


polyhedral,neoplastic,epithelialcells.
\otcr
. t,,.,.r,rr' 2. The ameloblastic fibro-odontoma and ameloblastic fibroma are composed
of neoplastic epithelium and mesenchyme.
3.The central giant cell granuloma is composed ofa proliferation ofspindled
fibroblasts in a stroma containing variable amounts ofcollagen. Multinuclated
giant cells are present throughout the connective tissue stroma.

. In

the mandible than in the maxilla. and more often Dosterior than in the anterior

reglons

. In the mandible

than in the maxilla, and more often anterior than in the posterior

reglons

. In

the maxilla than in the mandible. and more often Dosterior than in the anterior

regions

. In the maxilla than in the mandible,

and more often anterior than in the posterior

regl0ns

'fi0
Coplrighr O

201 1-201 2

- Dnral Decks

. A predilection for persons younger than 20 years old

. A predilection for middle-aged caucasian men


. A predilection for middle-aged black women
. No age, racial or sex predilection

111
Coptrighr

201 l

'2012 - Dntal Decks

The cementoblastorna, also known as the true cementoma, is a rare benign neoplasm of
cementoblast origin. It occurs predominantly in the second and third decades, typically before 25 years ofage. It is more often seen in the mandible than in the maxilla, and more
often posterior than in the anterior regions. It is intimately associated with the root of a
tooth, and the tooth remains vital. It may cause cortical expansion and, occasionally, lowgrade intermittent pain. Radiographically, this is a well-circumscribed radiopaque lesion
that replaces the root ofthe tooth. It is usually surrounded by a radiolucent ring. Because
ofthe intimate association ofthis lesion with the tooth root, this lesion cannot be removed
without sacrificing the tooth. There is no recurrence.

Note: To distinguish this lesion from condensing osteitis


the root outline.

in CO you can distinguish

represents a reactlve
This lesion also known as periapical cemento-osseous dysplasta
response of periunusual
an
to
be
appears
lesion
,uth". thun a neoplastic proiess. This
phenomenon that occurs aI
apical bone to some local factor This is a relatively common

affected more than are men'


the apex ofvital teeth. Women, especially black women' are
the anterior peiiopp"urc in *iOat e age (around i0 ,earsl and the mandible' especially
often' the apices
More
riapical region, is farrnor" .o.-only uffected than is the maxilla
vital' As the
are
the teeth
of trvo or r.riore teeth are affected. There are no symptoms and
a mixed or mottled patcondition progresses or matures, the lucent lesion develops into
The final stage appears as a solid opaque mass often surrounded

,. i"t. repair
thin iucent ring. No treatment is required for this condition'
by a "ttl.g

i"*

\ote: A rare condition called florid

osseous dys

plasit (FoD) appears to be an exuber-

ant fonrl ofperiapical cemental dysplasia'

the age of 40 There are


The cementiffing fibroma occurs chiefly in adults around
Iesion may cause tooth
The
fr.iii".tio* ior ine uody of the mandible antl for females'
it is well-circumscribed and may aprr.ro. ement or cortical expansion. Radiographically,
lt is treated with
pea, ..tatively radiolucent, lucent with opaque foci, or diffusely opaque
can be considered simcu..etag. o. e^cirion. Recurrence is very rare Note: This lesion
ilar or identical to the ossifying fibroma'

. TuberosibT area
. Maxillary anterior area

. Mandibular premolar area


. Molar

ramus area

112
Coplright O

20ll-2012

Denral Decks

Th smeloblrstic libromr and ameloblastic librordontoma eppeer to be


variations of tbe same process. These neoplasms occur predominantly in:

s-,

. Adults with a mean age of 40

. Elderly people with a mean


. Young adults with

. Children

age

of

75

mean age of25

and young adults

.113

Copyright O2011,2012 - Dentat Deck

The mandible is affected twice as often as is the maxilla. There is a predilection for the
molar-ramus region. although any site may be affected.
sheets of large polygonal epNote: A unique microscopic pattem typifies the CEOT
v,ith
of
amyloid
that
have
concentric calcified deposits
ithelial cells are usually seen
areas
([, ie s e g an g ri ngs ) tl]roughout.

Teens; females:

anteriorjaws; in
association wilh the
crowns ofimpacted

Well-circumscribed
unilocular RL lesion; may
have small opaque foci

teeth

Adults f,/0

l"dr'ti

molar-ramus region;

law expansron
Adolt qhears);
anterior maxilla and
posterior mandible;
no symptoms
Females over 60;
mandible and maxilla

Resection ofaffcctcd
Unilocular or multilocular
RL lesior! may have opaque area; recurrence common
if inadcquately treated
foci

Well-circumscribed, oft en
scmilunar RL lcsion
associated with the roots
teeth

of
Resection of affected
areai locally aggrcssivc;
metastases lo lung and
regional lymph nodes

E\cept for the presence of an odontoma, the 1wo lesions are tlte same. The mean age is
lround 12 years, and the upper age limit rnay extend as high as 40 years. The mandibuiar molar-ramus area is the favored location for the two lesions. Radiographically, these
lesions are rvell circumscribed and are usually surounded by a sclerotic margin. They
mal be either unilocular or multilocular and may be associated wiih the crown of an impacred toofi. Important: An opaque focus appears within the ameloblastic fibro-odontona o\\ ing to the presence ofan odontoma. This lesion therefore presents as a cornbined
lucent-opaque lesion; the ameloblastic fibroma is completely lucent radiographically.

\ote:

These lesions are treated conservatively by curettage or excision. Recurrence is

rare.

The other tumor of mixed, (epithelial and mesenchynal) origin is the odontoma. These
;alcilied iesions take one or two general configurations. They may appear as multiple
miniature or rudimentary teeth, in which case they are known as compound odontomas, or they may appear as amorphous conglomerations of hard tissue, in which
case thel are known as complex odontomas. As a group they are the most common odontogenic tumors.

How would you refer to the group of small radiopacities between the
rnandibular canine and first premolar on the periapical x-ray below?

'114
Cop)rrghr

ae

:0ll :012

Dertal Decks

ORAL PATHOLOGY

A 4s-year-old African Amerlcan female presents to your oflice for a routine


exam. Periapicals of the mandibular incisors show multiple radioopacities
with radiolucent rims. Teeth #23 through #26 test as vital. There is no pain
on percussion or palpation. Tfeatment for these lesions should be:

. Do nothing (obserue)

. RCT treatment lor

teeth #23 through #26

. Surgical excision of lesions


. None ofthe above

115
Cop)'right C 201 l-2011 - Dental Decks

Complex

Children and young adultsi


tendency to occur in the
posreriofja\r's

\{icroscopic
Characteristics

Radiogrxphic
Charactedslics

Clinical l'eatures

Tumoa

Amorphous opaque

Nomlal appealng

masses iYpically in a
tooth-beanng ara, between roots or over tne

enamel. dcntin. cementum. and pulp


may be sccn.

Treatment
aud Prcgnosis
Enuclealion;

impacted tooth

Compound Children and young adults;


tendency to occur in thc
anteriorjaws

Multiple small toothllke structures


typically in

tooth-

bearing area, berween


roots or over the qown
ofan impacted tooth

Normal-appearing
enamcl. dentin.
cementum. and PulP
may bc seen.

Enucleation;
docs not rccur

Remember: A compound odontoma looks like a tooth, and a complex odontoma does
not lit is a disorganized arrangement oJ tubular dentin, enqnrel' and thin la1'ers of cenemunl. See picture #52 in booklet for picture ofcomplex odontoma
as ameloblastic ondontoma has been described. This is essentially an ameloblastoma in which there is focal differentiation into an odontoma.

l.'ote: A rare variant known

Periapical cemental dysplasia, also known

as

periapical cemento-osseous dysplasia (and

lornerly knov'n as cementoma) represents a reactive rather than a neoplastic process. The
tenr "cernentoma" is a misnomer as thc opacities are not cementum but bone. While they
appear lo arise lrom the teeth, the lesions apparently arise within the bone instead. It appears to be an unusual response ofthe periapical bone to some local factor (br example,
It dunntic occIusion or infbction).
Clinical features:
. Occurs at the apex ofvital teeth

. A pre dilection fbr middle-aged black women


. As] mptomatic, usually multiple, small periapical

areas ofradiolucency in the mandibu-

lar incisorarea. See picture #53 in booklet

\ote:

Depending on stage, it may appear mixed radiolucent and radiopaque or totally ra-

dropaque.

Important: Age. gender, Iocation, radiographic appearance, and tooth vitality considered together are diagnostic of this condition.
Three stages:
L Osteolytic stage; radiolucency appears on radiograph.
Cementoblastic stage: beginning of calcification in the radiolucent area (mixed radiolucent and rqdiopaque appearance).
3. llature stage: radiopacity appears on radiograph with a thin radiolucent line around

l.

area.

\ote:

No treatment is required for cementomas. Once this stage is reached, the lesion stabilizes and causes no complications.

ORALPATHOLOGY

PigLesofOralCav

A S-year-old boy presents with his mother for his lirst dental exam.
Your exam reveals a normally developing dentition but you notice
multiple "freckles" on his lower lip and on the buccal mucosa.
What condition should vou be concerned about?

. Gorlin-goltz syndrome
. Gardner's syndrome
. Peutz-Jeghers syndrome
. Cleidocranial dysplasia

115

Coplriglrr o

ORAL

20ll-l0l:

- Denral Decks

PATHOLOGY

Pig Les of Oral Cav

What most likely caused the discoloration of the


gingiva as seen in the picture blow?

117
Copyrighr C

20ll'l0l:

Denral Decks

Peutz-Jeghers syndrome (also called hereditat! itesti al polyposis syndrome) is an unusual


condition which is ofinterest to the dentist because of the oral findings. The pigmentations
usually appear at an early age, often during the first decade of life and at this time are restricted to the oral region. Intraorally, these pigmentations may be located anywhere on the
mucosa, but are most frequently seen on the buccd mucosa, gingiva and hard palate. The
mucosal surlace ofthe lower lip is almost invariably involved. These spots or macules, while

of variable intensity. may mnge through shades of brown, blue and black. During the succeeding decades of the patient's life, pigmentations may arise elsewhere on the skin, especially on the extremities. In addition, intestinal polyposis is seen, most commonly in the
small intestine (jejunum) and may produce signs and symptoms of abdominal pain, rectal
bleeding, and diarrhea. It should be noted that the pigmentations ofPeutz-Jeghers syndrome may occur without demonstrable evidence of polyps and, also that multiple pollps
may be encountered without pigmentations. See picture #35 in booklet

Important: It is significant to note that although the oral pigmentations per

se are harm-

less. their presence is important in that they indicate a necessity for investigating the pos-

sible presence of multiple polyposis which may prove harmful. There is a strong tendency

for these multiple polyps of the colon to undergo malignant chang.

\ote: It

appears to be caused by a mutation ofa gene known as


I 9 that encodes a multifunctional serine-threonine kinase.

LKBI ofchromosome

has been mistaken for a


mclanin-pigmented lesion. The most common looations for amalgam tattoos are the gingiva. buccal mucosa and alveolar mucosa,

.\n amalgam trttoo is the most common oral pigmented lcsion The tattoo

Trauma and chemicals are frequent causes

oforal lesions:

occurs when patients placc the tablet against an aching tooth, allowing the check
'\n aspirin burn
or lip to hold it in position while it dissolves slowly. Within a few minutes a buming sensation ofthc
mucosa will bc noted and thc surface becomes blanched or whitened in appearancc The caustic acrion ofthe drug causes necrosis ofthe oral mucosa, with subsequent sloughing ofthe necrotic epiiheliurn. \ote: Aftempt to locate amalgam on x_ray if tattoo is suspcctcd; ifyou can not locate any
amalgam. a biopsy may be needed to rule out a melanocytic neoplasm See pictur #54 in booklet
Remember: Chcmical bums may also be caused by hydrogen peroxidc. silver njtratc, phcnol' etc.
. The use ofthe heary mctal bismuth is still common in treating certain dermatologic disorders as well
as larious other diseases. Bismuth pigmcntation appears as a "bismuth line," a thin' blue-black line
in the marginal gingiva which is sometimes confined to the gingival papilla Note: Lead ingestion also
l.avcs dark marks on the gingiva.
. Traumatic ulcer: very common; may mimic oral cancer and chronic infectious ulcers
. Fo.al (frictionat) keratosisi common whitc lesion caused by chronic friction on the mucosa.
Important: Differcntiated from idiopathic leukoplakia becausc cause is known
. Linea alba: a type offrictional keratosis that appcars as a linear whitc line in buccal mucosa
. Smoking-associated melanosis: caused by a chemical in tobacco smoke that stimulates melanin production. Note: If thc pigmentation is localized, an ulceration is present or thc lesion is elevalcd, a
biops.v is necessary to exclude other pigmented conditions feg, nevi, melanoma). Although smoker's
mclanosis is an abnormal deposition ofmclanin, the lesion itsclf is not associated with an increascd
risk ofmclanoma or carcinoma.
. Nlelanotic macufei most common mclanocytic lesion. Gingiva a commonlocatton (called an orul
nelonotic macule - See picture #55 in booklet/ and the lips, most frequently on the lower lip (called
a labial melanotic macrle). Can be associated with Peutz-Jegher's syndromc
. Drug-induced pigmentation: Most commonly associated with minocycline' chloroquine' cyclophosphamide, and azidothymidine (AZT).

. Buccal mucosa
. Tongue

. Hard palate
. Alveolar mucosa

118

Copright O 201l-201: - Dental Decks

All ofthe following conditions demonstrate pigmentation of th intraoral


mucous membranes .EXCfPl one. Which one is the EXCEPTION2

. Addison's disease

. Albright's syndrome

. Cushing's syndrome
. Peutz-Jeghers syndrome

't19
CoD,right C 201 I -20

12

- Denral Decks

Moles (nevi) are small, usually dark, skin growths that develop frotn pigmenlproducing
cells in the skin (called melanocltes). While nevi are fairly common on the skin, intraorally they are quite uncommon. When present, they are usually on the hard palate, but
can also be seen on the gingiva and lips. Congenital nevi (commonlt knou,r as birthmarks) are usually large (greqter thqn I 0 cm) and with the passage of time, may change
ftom flat, pale tan macules to elevated, vemrcous, hairy lesions. Approximately 150/o occur
on the skin ofthe head and neck. Congenital nevi have a higher incidence of malignant
transformation (as opposed to acquiretl nevi).

Acquired nevi are microscopically classified into five subtypes:

l. Intramucosal nevus: most common in oral cavity


2. Blue nevus
3. Compound nevus: rare in oral cavity
,1. Junctional nevus: rare in oral cavity
5.Intradermal nevus: is the most common lesion of skin. known as the common mole
\ot: Acquired nevi
ertraorally.

are much more common than congenital nevi both

intraorally and

Important: The B-K mole syndrome and the dysplastic nevus syndrome are both characterized by having numerous large, pigrrented atypical nevi which have a high risk for
der eloping malignant melanoma.

Addison's disease (qlso called chronic adrcnocortical itsuflicienq,) results from hypofunction ofthe adrenal cortex. lt is chamcterized by bronzing ofthe entire skin. Oral signs consist
of diffuse pigmentation ofthe gingiva, tongue, hard palate and buccal mucosa. Although curaneous pigmentation will most likely disappear following therapy, pigmentation ofthe oral
tissues tends to persist.

-{lbright's s1'ndrome (also celled McCune-Albright syndronre) is a severe form of


pollostotic fibrous dysplasia, involving nearly all bones in the skeleton. In addition to
the bone lesions there are brown patches ofcutaneous pigmentation (called caf6-au-lait
spots, and endocrine dysfunction, especially precocious puberty in girls.
Important: There is an increased incidence of osteosarcoma seen with polyostotic fibrous dysplasia.

Peutz-Jeghers syndrome (also known as Hereditary Intestinal Pol!-posis Syndrome) rs


an inherited disorder that is characterized by having multiple intestinal polyps and inrraoral melanin pigmentations. These pigmentations usually appear during the first
decade of life and at this time are restricted to the oral region. They may be located anyu here on the mucosa, but are most ftequently seen on the buccal mucosa, gingiva and
hard palate. The mucosal surface of the lower lip is almost invariably involved.

\ote: Cushing's syndrome

is a hormonal disorder caused by prolonged exposure

ofthe

body's tissues to high levels ofthe hormone cortisol. It is relatively rare and most commonly affects adults aged 20-50. The symptoms vary but most people have upper body
obesity, rounded face, increased fat around the neck ("buflblo hunp"), and thinning arms
and legs.

year oro
patient pnesenrs
presents ro
A /ru
40 yeer
physician wrtn
otd pauena
to nrs
his pnysrcran
with comptaints
muscle
complaints ol
of musce
werkness and loss of appetite. II has noticed a loss of weight and also
his skin hss started to "bronze,' His labs show lowered blood glucose and
sodlum and an increased potasslum. One condition likely to be crusing this is:

,'

t[rt

Peutz-Jeghers syndrome

. Cushing's syndrome
. Addison's disease

. Albright's syndrome

120
CoplriShr

@ 201

1,2012 - Dental Decks

Focal melanosis is a common circumstanc in which brownish areas


of pigmentation occur in the oral cayity; once properly diagnosed:

. Surgical excision
. Radiation

is required

is required

. No treatment is necessary
. Antibiotics are required

121

Copliight C 20ll-2012 - Dental Dects

Addison's disease occurs when the adrenal glands do not produce enough cortisol /a g/ricocorticoid). Cortisol's most important function is to help the body respond to stress. The
failure to produce adequate levels ofcortisol can occur for different reasons. The problcm
may be due to a disorder ofthe adrenal glands themselves (prinary adrcnal irrsulJiciencv)
or to inadequate secretion ofACTH by the pituitary gland f.secondary adrcnal insullicienc,").
The symptoms ofAddison's disease usually begin gradually.
These include:

. Muscle weakness
. Loss ofappctitc
. Weight loss
. Skin changes with areas of hyperpigmentation covering cxposed and nonexposed
parts ofthe body. This darkcning ofthc skin is most visiblc on scars, skin folds, pressure
points such as elbows, knccs, knucklcs, and tocs as wcll as thc oral mucous membranes
is diffuse pigmentation ofthe gingiva, tongue, hard palate. and buccal mucosa.

-there
. Nausea,

vomiting and diarrhea

. Low blood pressure


*** Clinical features do not begin to appear until at least 9070 of glandular tissuc

has

been destroyed.

Laboratory tests show:


. Lou'blood concentrations ofsodium and glucose

.
.

Increased serum potassium


Dccreascd urinary output ofcertain steroids

Important: The main concern when performing dental procedurcs on a patient with Addjson's diseasc is that the adrenal cortex has no capacity to put out cxtra cortisol as a response to strcss.

Focal melanosis is a disorder of increased melanin pigmentation that develops without


preceding inflammatory disease.
nelanrn (especially in the skin).

lt

is a condition characterized by abnormal deposits

of

f tiologl': Developmental
Location(s): Any mucosal site; gingiva a comrnon location (cqlled an oral melqnotic
picture #55 in booklet) and the lips, most frequently on the lower lip
nracrile
rc?lled a-See
labial melanotic macule)

Clinical Features: Brownish coloration to the oral mucous rlembrane


Radiographic Features: None
\Iicroscopic Features: Melanin pigment within rnelanocytes
Complications: None
Treatment: None
Prognosis: Excellent
Pathogenesis: Increased numbers olmelanocytes locally

Important: Melanosis may be observed in adrenal insufficiency (Addison s disease),


-{CTH-producing tumors, malignant tumors metastatic to the pituitary gland, or metastatic malisnant melanoma.

. Intradermal news
. Compound nevus
. Junctional news
. Blue news

. Intramucosal nevus

122
Coplright O 20ll-2012, Dental Deck

. Well-defined unilocular or multilocular radiolucency with scalloping around the roots

. Not well-circumscribed radiopaque lesion which may have a "ground-glass"

appear-

ance

Saucer-shaped radiopaque lesion

. Not well-defined multilocular radiolucency with a "pear-shaped" appearance between


the maxillary central incisors

123
CopJright O 20ll-2012 - Dental Decks

Most pigmented skin tumors are composed of ner.us cells and are a result of a developmental anomaly ofmelanocytes; they are rare in the oral cavity. The initial, flat, raised
lesion can become nodular, with an increase in consistency. Spontaneous involution may
occur and malignant transformation is a rare complication. When found intraorally they
most frequently occur on the hard palate (See picture #58 in booklet) but can also be
seen on the gingiva and lips. lntramucosal nevi are the most common variety found in the
oral cavity followed by blue nevi. Compound and junctional nevi are very rare.
Subtypes of Acquired Nevi:
.Intradermal nevus: most common lesion olskin. Known as the common mole.
Nevus cells lie exclusively within the dermis. See picture #57 in booklet
. Junctional nevus: nevus cells are located at the inteface between the epithelium and
lamina propria. They are flat and not detected by palpation. Some regard as premalignant, may undergo transformation into malignant melanoma.

picture #59 in booklet


. Compound nevus: nevus cells are located at the epithelium/lamina propria interface
and also deep in the dermis. They are raised and solid. See picture #60 in booklet
. Blue nevus: congenital, painless; color based on the deep cutaneous or subcutaneous/
submucosal deposits of melanin. See picture #56 in booklet
. Intramucosal nevus: nevus cells are located in the connective tissue or lamina propria ofthe oral mucosa. Under palpation these nevi appear solid and are slightly raised
See

or er the surface

ofthe mucosa.

Important: Ifa pigmented lesion shows ulceration, an increase in size, darkening in color,
erc.. a biopsy should be performed--+his may indicate transfomation into a malignant
melanoma.

The traumatic bone cyst is an asymptomatic intmosseous empty cavity ofyoung patients located
p:imanll' *,ithin the mandible, lined by a thin loose connective tissue membrane.
TrarLmalic bone cysts are non-cysts fre/ened lo as pseudocysts) and have many names, which
include the simple bone cyst, hemorrhagic bone cyst, unicameral bone cyst, extravasation

t'one cyst. idiopathic bone cyst and solitary bone cyst.

Clinical features:
. Children and adolescents, usually before the age of20
. \lore conmon in males
. Usually asymptomatic, may produce enlargement ofthejaw
. Conmonl,"- found in posterior mandible *** See picture #61 in booklet
. Regional teeth are vital teeth
The treatment of traumatic cysts is relatively easy. lt consists of opening the lsion, curettage
and closure. The resultant blood clot soon undergoes organization and the bone defect is soon reoarred.

is a closed compartment that has a connective tissue lining (iro epithelial linilg,r ofvaried thickness. It may contain blood, serosanguineous fluid, debris composed chiefly
ofa blood clot, or it may be completely devoid ofsolid rnaterial.

\ote: This cyst

The following are also not true cysts

. Latent bone cyst

. Lingual mandibular concavity


. Aneurysmal bone cyst

-they

are called "pseudocysts":

ORAL PATHOLOGY

Pseudocyst

All ofthe following statements are true concerning the aneurysmal


bone cyst.EXCEPI one. Which one is the EXCEPTIOM

. It is an uncommon expansile

osteolytic lesion of bone consisting of a proliferation


vascular tissue that forms a lining around blood filled cystic lesions

. Most aneurysmal bone cysts occur in patients under 20 years of


mon after the age ol30
. It commonly involves

the

age, and

of

it is uncom-

jaws

. The lesions are usually tender or painiul, particularly upon motion ofthe bone affected
. Upon entering the lesion surgically, excessive bleeding is encountered

124
Copvnglu

e 2011-:012 '

Dental Decks

ORAL PATHOLOGY

R-B Les

A patient presnts with an asymptomatic, elongated, erythematous


patch of atrophic mucosa of the mid-dorsal surface of the tongue due
to a chronic C. albicans infection. The most likely diagnosis is:

. Thyroglossal duct cyst


. Lymphangioma
. Hemangioma
. Median rhomboid glossitis

.125

Copyiight O 20ll-1012 - Dental Decks

***

This is false; it commonly involves the proximal humerus, femur, tibia and pelvis.
is not as common in the jaws, but it does appear here as well.

It

The aneurysmal bone cyst (ABC) is classified as a pseudocyst because it appears radiographically as a cysrlike lesion but microscopically exhibits no epithelial lining. It represents a benign lesion ofbone that may arise in the mandible, maxilla, or other bones.

It is typically seen in patients under 30 years old, with

a slight female predilection. When

the mandible and maxilla are involved, the more posterior regions are affected, chiefly the
molar areas. Pain is described in approximately half the cases, and a firm non-pulsating
swelling is a frequent clinical sign. On auscultation, a bruit is not head, and on firm pal-

pation, crepitus may be noted.


The radiographic picture ofthe lesion is often distinctive. Usually a unilocular radiolucency is found with slightly irregular margins. The bone is expanded and appears cystic
with a "honeycomb" or "soap bubble" appearance. Teeth may be displaced with or
without concomitant extemal root resorption. See picture #62 in booklet

Histologically, the lesion consists ofa fibrous connective tissue stroma containing many
cavemous or sinusoidal blood-filled spaces. Fibroblasts and macrophages (histiocytes)
line the sinusoids. Multinucleated giant cells, similar to those ofa central giant cell granuloma, are dispersed throughout.
Surgical curettage or excision is the treatment ofchoice, with little chance ofrecunence.

\ote:

The differential diagnosis should include ameloblastoma, central giant cell granuloma and a central vascular lesion fthis can be ruled out uoon auscultation since a bruit
is often heard).

This entity was once thought to be a congenital abnormality related to the persistence of
the tuberculum impar, however, it is now believed that this condition is a permanent end
result of a chronic Candida albicans infection. Diabetics, immunosuppressed patients
and patients on long-term antibiotic therapy are more susceptible to this condition.
as a smooth, denuded, beef,, red lesion devoid offiliform papillae. The most common location is the midline ofthe dorsum ofthe
tongue, just anterior to the circumvallate papillae. It is generally asymptomatic. Generally

Median rhomboid glossitis usually presents

no treatment is necessary, however, topical or systemic antif,rngal drugs to manage the pre-

disposing factors may be helpful.

Median rhomboid glossitis

. Children

Teenagers

. Post-menopausal women
. Middle-aged males

126
CoDrighr O 201l-2012 - Dental Decks

. Wegener's granulomatosis

. Rendu-Osler-Weber
. Sturge-Weber
. Juvenile nasopharyngeal angiofibroma

127
Cop)'righr O 201 I -2012 - Dental

Deck

Burning mouth syndrome (BMS) is a relatively common condition that is characterized by


a complaint of an abnormal sensation of the lining of the mouth that most patients describe as feeling like their mouth has been scalded. Usually this sensation develops in the
ftont part of the mouth, typically affecting the inner surfaces of the lips, the roof of the
mouth and the sides and tip of the tongue. In some patients, only the tongue will be affected, however, any combination ofthese sites may be seen. Some patients may have a
decreased taste ability or altered taste sensation (bitter or salt!). Other patients may feel
that their mouths are dry or sticky. In all cases, howeveq the lining of the mouth clinically appears normal. Note: BMS is a "diagnosis of exclusion."
The etiology is unknown. There are a few common diseases that should be tested lbr, such
as anemia. diabetes and oral yeast infections. Buming mouth syndrome is diagnosed by
doing blood tests and cultures to make certain that one of the other problems mentioned
previously is not present. Ifthose tests are all negative, and ifthe lining ofthe mouth appears normal, then the diagnosis ofburning mouth syndrome is made.

Unfortunately, no one has developed a medically proven treatment for buming mouth
slndrome. For about halfofthe affected patients, the condition will resolve after a period
of tin.re. but no one can predict how long that will be for a particular individual. For the
most part. this problem is a nuisance, and it is a frustrating situation for both patients and
doctors. Note: Some individuals find relieffrom axiolytics, antidepressants, or low doses
ofcapsaicin, applied 3 or 4 times topically on the area(s) where the pain is localized. appear to be quickly effective in alleviating the pain in BMS.

Rendu-Osler-Weber syndrome, also known as hereditary hemorrhagic telangiectasia, is a congcnrtal hcrcditary form of hemangioma. It is characterized by numerous spiderlike telangiectases on the face, neck. chest. lips, gingiva, buccal mucosa and tonguc. One ofthe earliest signs ol'
rhe disease is epistaxis (ho:tehleeds). Arteriovenous fistula, cspecially ofthe lungs and liver. are a
\ ariablc conlponent. Blceding from the tclangicctases may be recunent, lifethreatening and in;rease in sevcrity with aging. The onset often is in childhood. Both sexes are affected equally

Rememberi
. Encephalotrigeminal angiomatosis fslurge-lleber disease) is an uncommon congcnltal syndronrc ofunknown etiology (somelines classdied as a variant.forn ofhemangioma). It consists
of a l'acial lcsion, known as the port-wine stain, rvhich is distributed ovcr the trigeminal nerve
accompanicd by a similar vascular disorder ofthe underlying mr;ninges and cerebral cortex lt
usuall! occurs unilaterally.
. Juyenile nasopharyngeal angiofibroma is a rare, benign neoplasm that ncarly always aflects
adolescent males. It characteristically produces a mass in the nasopharynx that leads to obstruction or epistaxis. Treatment is surgery. Recurrences are conmon.
egener's granulomatosis is a disease involving granulomatous inflarnmation, necrosis and vascuLtis that most frcquently targets thc uppcr respiratory tract, lower rcspiratory tract, and kidneys.

\\

The diagnosis ofWegener's granulomatosis is established most securely by biopsy specimcns showing the triad ofvasculitis, granulomas. and large arcas ofnecrosis (lototvn a.s geographic necrosis)
admixed with acute and chronic intlammatory cells. Also CANCAs (otlttueulophil c)'toplashic
antibodies) are highly spccific for Wegener's granulomatosis. The treatment includes prednisone and

cyclophosphamide.

Note: Midline granuloma is a unifocal destructive process, generally in the midline of the
orofacial region, that does not affect other organ systems. lt clinically mimics the lesions of
Wegener's granulomatosis. Most cases represent peripheral T-cell lymphomas of the upper
respiratory tract or m ovth (perforation of the lwrd palate na!- be seen). There ls a good prognosis when treated early with radiation.

A clinical term delined as a red patch that cannot be clinically


or pathologically diagnosed as any other condition is callsd:

. Leukoedema
. Psoriasis
. Erythroplakia
. White sponge nevus

124
Cop)righr ()

20ll

20ll

Denral Decks

ORAL PATHOLOGY

R-B Les

The arrow in the picture trelow is pointing to a fast-growing reactive


proliferation ofendothelial cells which is commonly found on the
gingivo and usually forms in response to chronic irritation.
The most likely diagnosis is:

129
Copy.ighr e

20ll 201?

Denral Decks

"erythroplakia'r like the term "lerkoplakia" has no histologic connotation. It


is a clinical diagnosis. Almost all true erythroplakias exhibit a microscopic picture of
epithelial dysplasia, carcinoma in situ, or invasive squamous cell carcinoma.
\ote: Biopsy is mandatory.
The term

Remmber: Carcinoma in situ exhibits all of the histologic characteristics of malignancy (pleomorphism, hyperchromatism, abnonnal niloses, onaplasia, e/c.), but does not
shorr invasiveness or extension into adjacent structures.
Enthroplakias may be located anywhere in the mouth, but are most likely to be found in
rhe mandibular mucobuccal fold, oropharynx and floor of the mouth. There is no sex
predilection and patients over 60 years old are most commonly affected.

This lesion represents an overexuberant connective tissue reaction to a known stimuIus or injury. The term "pyogenic granuloma" is somewhat ofa misnomer in that it is not
pus producing, as "pyogenic" implies. It is, however, a "tumor" ofgranulation tissue, as
"granuloma" implies.
Pl ogenic granulomas may also occur on the lower lip, tongue and the buccal mucosa.
Thel rarel.v occur on other areas ofthe oral mucous membrane. It is generally believed
lhar rhe pyogenic granuloma arises as a result of some minor trauma to the tissues fceneutdtion of q crotvrt, calculus, etc,),which provides a pathway for the invasion olnonspecitic t-vpes of microorganisms.

soft pedunculated broad-based growths that have a


smooth red surface. This red appearance is due to the presence ofhyperplastic granulation tissue, which contains many capillaries. They are often ulcerated, bleed easily and
ma1 have a raspberry-like appearance. Treatment consists ofexcision. They are benign,
but rnav occasionally recur.
Pr ogenic granulomas present as

\ote:

Pregnant patients are prone to these lesions (sorreliz es called "pregntncy tumor").

Peripheral giant cell granulomas are seen xclusively in:

. Buccal mucosa
. Alveolar mucosa

. Bone
. Gingiva

't

30

coplright o 20ll-2012 'DmialDects

ORAL PATHOLOGY

The picture below shows a benign, soft, moderately well-circumscribed


prinless mass which is deep red or blueish red in coloration.
The most likely diagnosis is:

131
Coplyighi C

201

1,2012 - Dentnl Decks

The peripheral giant cell granuloma has an unknown etiology, with some dispute as to
whether this lesion represents a reactive or neoplastic process. However, most authorities
believe the peripheral giant cell granuloma is a reactiv lesion. Local irritation due to
dental plaque or calculus, periodontal disease, poor dental restorations, ill-fitting dental
appliances, or dental extractions has been suggested to contribute to the development of
the lesion. They are seen exclusively in the gingiva, usually in the area between the first
molars and the incisors. See picture #63 in booklet

Peripheral giant cell granulomas typically present as red to blue broad-based masses.
They usually bleed easily and may occur at any age and tend to be seen more frequently
in females than in males.
When this process occurs on the edentulous ridge, a superficial, cup-shaped radiolucency
may be seen. Histologic sections are diagnostic and are identical to those of a central
giant cell granuloma. The lesion is composed of a proliferation of spindled fibroblasts
in a stroma containing variable amounts ofcollagen. Multinucleated giant cells are present throughout the connective tissue stroma. Conservative excision is typically curative,
although the lesion must be completely removed to prevent recurrence.
Generally, this lesion is clinically indistinguishable from a pyogenic granuloma.
-{lthough the peripheral giant cell granuloma is more likely to cause bone resorption than
rs the pvogenic granuloma, the differences are otherwise slight. Biopsy will provide detinitir e results.

\ote:

Hcmangiomas are benign vascular tumors composed ofcells that nonnally line the blood vessels
rentlothelial cells). They arc the most conmon tumor ofchildhood, occurring in up to l0% ofin1ants. Henangiomas may be visible at birth or may not be recognized until the first few weeks or
cr en months of lif'e. ln general, however, most hemangiomas becomc evident by 2 to 3 weeks 01'
age. -\lthough they comrnonly occur on the head or ncck (60%). they can occur in any region of
rhc bodl ). Some lesions are small and hardly visible, whereas others are large and rcadily obscrv:b1c'. \lost hemangiomas appear as single tumors, though l57o present with more than one lesion.

\otes

1 . Congenital hemangioma (slrawberry revr.r'./: local proliferation of capillaries; exhibit a rapid growth phase followed several years latcr by an involution phase; persist-

ent lesions are excised.

2. \'ascular malformation: a pcrsistcnt malfonnation of blood vessels; do not involutc; exhibits a thrill (palpate a pulse) andbrdt thear a pulse). A typc ofvascular nallomation is known as Sturge-Weber disease (encephalotrigeminul angiomatosis)
w hich consists of a facial lcsion. known as the port-rvine stain, which is distributcd
over thc trigeminal nervc accompanied by a similar vascular disorder ofthe underlying
meninges and cerebral co ex. lt usually occurs unilaterally.
3. Remember: Rendu-Osler-weber syndrome, also known as hereditary henlorrhagic
telangiectasia, is a congenital hcreditary lbrm ofhemangioma. 11 is charactcrizcd by numerous spider-like telangiectases on thc face, neck, chest. lips, gingiva, buccal mucosa and tongue. One of the earliest signs of the disease is epistaxis (nosebleeds).
Arteriovenous listula, especially ofthe lungs and liver, are a variable component. Bleeding from thc telangiectases may bc rcculrent, lil'e{hreatening and increase in sevcrity
rvith aging. The onsct oftcn is in childhood. Both sexes arc aflected equally.
4. lmportant: How to distinguish between a hemangionra and a hcmatoma
-hemangioma will blanch on diascopy. hematomas do not blanch.

. Uncommon and represerfiz4yo ofhead and neck neoplasms

. Common

and represent 75-80% ofhead and neck neoplasms

. Uncommon and represent

. Common

25-30olo

of head and neck neoplasms

and represent 95-98% ofhead and neck neoplasms

132
Coplright

@ 201 I

-2012 - Denral Decks

. White sponge nevus


. Lichen planus
. Necrotizing sialometaplasia
. Focal hyperkeratosis

133
Coplright O 201l-2012 - Denral Decks

Salivary gland tumors may be broadly categorized into benign neoplasms, tumorJike conditions, and malignant neoplasms. The glands are divided into major and minor salivary gland
categories. The major salivary glands are the parotid, the submandibular. and the sublingual
glands. The minor glands are dispersed throughout the upper oral-digestive submucosa
(i.e., palate, lip, pha4nx, nasopharl,rrx, latynx, paraphatyngeal space).

Benign Salivary Gland Tumors:

. Pleornorphic adenoma lbenign

mLxed tumor./ is the most common tumor of major and

minor salivary glands. It is a mixture of epithelial and mesenchymal elements (pleomorphi.).85yo occnr in the parotid gland. Within the oral cavity, the majority are found in the
palate. Pathologically, it is characterized by slow growth and few symptoms.
. Monomorphic adenomas: The term "monornorphic adenoma" refem to a group ofrare
salivary tumo$ that includes the basal cell, canalicular, sebaceous, myoepitheliomas and oncocytomas. Ofthese, the basal cell adenoma is the most common.

. Warthin's ttrnor

(papillary' .!-stadenoma lymphomatosnn) is almost exclusively

parotid neoplasm. It is a glandular and cystic tumor lined by

a bllayered (inner columnar

otlcoc)lic qnd outer ba.sal) epithelium with a lymphoid stroma.


. Benign lymphoepithelial lesions include a wide range of cystic changes that share the
common denominator in atypical lymphoid hyperplasia. Hyperplasia refers to a proliferation ofcells that is non-neoplastic. These changes are found often in patients infected with
H

I\i

-\.'ot

L Clinical features of benign salivary gland tumors: mucosa is normal, painless,


nodular, movable, fifm. and slow-growing.
2. The most common site ofintraoral minor salivary gland neoplasms is the palat.
3. The most common site of intraoral major salivary gland neoplasms is the
parotid.

\ecrotizing sialometaplasia is a recognized lesion of the minor saliYary glands characterized by necrosis ofthe glandular parenchyma with associated squamous metaplasia
and hyperplasia ofthe ductal epithelium.
Important: The initiating event ofnecrotizing sialometaplasia is believed to be related to
ischemia, secondary to alteration of local blood supply. lnfarction of the salivary gland
tbllou s. presumably due to compromise ofthe vascular supply.
It is usually observed in adult rnales and presents as an asymptomatic, necrotic. ulcerated
area inr,olving the palatal mucosa. The hard palate appears to be the most common site.

Histologicall!', there is lobular necrosis of the glandular parenchyma, with squamous


metaplasia and hlperplasia of the ductal epithelium. Both clinically and histologicallS
the lesion may sin.rulate a malignancy and in the past, the condition has been misdiagnosed as a squamous cell carcinoma or mucoepidermoid carcinoma.

Following biopsy and the establishment ofthe diagnosis, further treatment generally is not
recommended since healing usually occurs within 6-10 weeks.

. Measles

. Mumps
. Rubella

. Chicken pox

1U
Copyright O 20ll-2012 " Dent.l Decks

. Parotid gland tissue


. Submandibular gland tissue
. Sublingual gland
.

tissue

All of the above

l3s
coplright

20r I -20 12 - Dental Decks

Mumps is an acute viral illness. The mumps virus is a paramyxovirus in the same group
parainfluenza virus. The virus is acquired by respiratory droplets. It replicates in the
nasopharynx and regional lymph nodes.
as the

Important: Mr.rmps is the most frequent diagnosis of acute viral sialadenitis, mostly in
the parotid gland.

Clinical:
. 907o ofthe

cases occur before l4 years of age


. A major sign is sudden salivary gland swelling without purulent discharge from the
duct
*** Parotid
involvement and bilateral in two-thirds of the cases.
-90o%and anorexia
. \4iltl ferer. malaise

. Most cases are self-limiting

Complications:

. Orchitis (inflqmmation of the testis) and epididymitis


males. *** Important: May cause sterility
. CNS

systeln

can occur in post-pubertal

rneningitis and encephalitis

. Dealness. myocarditis, pancreatitis, oophoritis

and pyelonephritis

The serum amylase may be elevated during the acute phase. Prevention with a live
attenuated vaccine is 95% effective for at least five years. However, in non-inoculated
indir iduals, it is still a cause ofacute non-suppurative saliyary adenitis.

The stafne bone cavity or cyst is a developmental anomaly represented by a bone conitl usually containing submandibular gland tissue. The posterior mandible region,
pafiicularly at the angle and below the mandibular canal is the most common location. The
sraric bone cyst is usually discovered incidentally on dental radiographs. It is asymptomatic and is not a true cyst. rather an anatomic depression in the lingual aspect of the
bodl of the mandible where normal salivary gland tissue rests. The radiographs show a
small, circular, corticated radiolucency below the level of the mandibular canal. Histologicalll', nonr.ral submandibular salivary gland tissue is found and no treatment is required except routine radiographic follow-up. Note: Many other terms have been used to
describe this entity, including submandibular salivary gland depression, lingual salivary
eland deoression. and Stafne bone cvst.
car

Stafne bone cyst in characteristic location inferior to the mandibular canal

ORAL PATHOLOGY

SG Thm

A 53-year-old woman comes into the dental clinic with bilaterally


enlarged parotid glands. It was discovered that she had recently
ben to the African continnt and had contracted tuberculosis.
What is the name of the autoimmune disease associatd with
enlarged salivary glands in association with a secondary disease?

Sjogren's syndrome

. Mikulicz's disease
. Gorlin-Goltz syndrome
. Pierre-robin syndrome
. Apert syndrorne
136
Copyrighr

aq

20ll l0ll - Dcnbl

Decks

ORAL PATHOLOGY

SGTirm

A 33-year-old patient comes into your office for a routine maintenance appt.
While doing an intraoral exam, the hygienist discovers a bluish lesion ofthe
lower lip. The patient relates a history of biting this area last week whn
he had a sinus infection. What is the most likely diagnosis ofthis lesion?

. Ranula

. lnfectious sialadenitis
. Maxillary sinus retention cyst
. Mucocele

137
CoDrighr

aq

20ll-2011- D.ntal

Decks

The term "benign lvmphoepithelial lesion" (also called Mikulicz's disease) is manifested essentially as a progressive, asymptomatic enlargement of the parotid and
submandibular glands. It is initially unilateral, but over time, it becomes bilateral. The
etiology is unknoun, however, there is increasing evidence that both Mikulicz's disease
and Sjogren's syndrome are both actually autoirnmune diseases in which the patient's
o\\ n salivaD gland tissue becomes anligenic.
The benign lymphoepithelial lesion is rare. It occurs most frequently in middle-aged
rvomen. Histologically', there is replacement of the gland parenchyma by lymphocytic
inliltrate rvithin which there are scattered epimyoepithelial islands. This is tl.re histologic
comerstone lor the diagnosis of the benign lymphoepithelial lesion.

differential diagnosis ofchronic bilateral enlargement ofthe salivary glands as a result of benign lymphoepithelial lesion must be separated for similar findings seen in sarcoidosis. lymphoma, gout. leukemia, diabetes mellitus, chronic alcohol abuse, and, rarely
The

hlpenension.

Important: Most of these lesions remain benign. however. malignant transformation


of the epimyoepithelial islands has been demonstrated.

\lucuse\tralasationphenomenon,commonlykno\\nbytheclinical

term, "mucocele", is scparrtcd from

mucus retention c!-st bccause each has a disdncrivc pathogenesis and microscopy. Scveral clinical features
rl.,r .il.a:ale thcsc lcsions.
. \lucus ertravasation phenomenon: is considered to be related to rnechanical lrauma to thc minor sali\ 3a
e\cretory duct. Although the lower lip is thc most frcqucnt sitc. the buccal mucosa, vcntral sur_
-!lalrd
ficr- Lrf thc rongue (\'here the glanls of Blondin Nuhn arc locdterll. lloor ol thc rnouth, and reiromolar
:r!iLrr rrc ofien atfccted. These lesions are usually painless and smoolh surl'accd, \\'ith an o\eral1 bluish hue
!rf rranslucenc) associated \lith a lnore superficial localion. The trcrtment ofchoice firr mucus extravasr:r!r: r!hcnonrenon is surgical cxcision rvith rcmoval ofthe associatcd gllnd.
. \luaur retention cyst: is regarded as a cyst becausc it is lincd by an epithelium
unlike mucus exi::r\ 3sairon phcnomcnon. r.hich contains mucus pool surrounded by granulation tissue.This cyst resuhs
tirm obtnu.lion of sali\ary flo\l', The mucus letention typc mucocclc is less common than the extravasa:ri:] \rrc Thc rcrcntion t-vpe usually appcars aftcr 50 ycars ofagc, and it rarely prcscnts in the lorver lip.
I:r.iead it is found in the palate, cheek, floor ofihe mouth. as wcll as in the nraxillary sinus. Clinical prcs.::]iron i\ characicrized by an asympbmatic s$elling, oflen without previous injury. The Icsions vary
l:L.nr I to l0 mm and. on palpation. are rnobile, non-tender and lhe overlying mucosa is intact and ofrormal color. Thc trca{ment ofthe mucus rctcntion cyst inchldcs surgical excisron *'ith rcmoval ofthe asso;ra:ed gland.
as a generic term (hclbrc nicroscopic didguo.ri.r) to
refer to both the mucus retention cyst and the mucus extravasation phenomenon.
L The maxillary sinus retntion cyst and pseudoclsts are commonly discovcrcd incidentally,
\ote3 ofren being ofgrcatcr curiosity than clinical significance on panoramic radiographs. May repre. sent b lockage o f sinu s sal ivary gland, or focal fluid accumulation of sinus mucosa. These lesions

lmportant: "}Iucoccle" is uscd in the clinical setting

'

arc asymptomatic and require no treatment.


:. Infectious sialadenitis: int'ections ofthe salivaF"' glands
tc al.

ftav

be acute or chronic.

viral or bac-

. \'iral:
- Nfumps is the most frequent diagnosis ofacute viral sialadenitis, mosdy in parotid.
- Cytomegalovirus infections are chronic; usually seeD in immunosuppressed patjents
. Bacterial: the most commonly isolated organisms arc Staphylococus aurcus. Strcptococcus. viridans. II. intluenzae. and E. coli.

What is the most probable diagnosis for a lesion that presents


as a translucent, bluish, well-rounded, smooth-surfaced bulge
that protrudes from one side ofthe floor ofthe mouth?

. Adenoid carcinoma

Squamous cell carcinoma

. A ranula

. A lymphangioma

138
Cop)aighr

iar

201l-2012 - Dental Decks

ORAL PATHOLOGY

SG Thm

A patient comes to your ollice complaining of pain when eating and even
sometimes when thinking about food. Your intraoral exam reveals a small,

hard swelling in the floor ofthe mouth. A mandibular occlusal radiograph


shows a per-sized radiopacity with 'onion-skin" thickening lingual to the
right mandibular border. Name the likely diagnosis:

Sialometaplasia

Sialadenitis

. Sialolith
. Sialosis

139

Coplriglit C 2011-2011 - Denral Dects

Ranula, is

a clinical term that is used to designate a mucocele that occurs specifically in


the floor ofthe mouth (see picture below). Pathogenetically and microscopically, it may
represent either mucus extravasation phenomenon or mucus retention cyst. The ranula is
associated with the duct system ofthe sublingual salivary glands, and, less commonly, the
submandibular glands.

The ranula is usually caused by an obstruction produced by either a salivary stone or soft
organic substance. It usually presents as a painless, fluctuant, unilateral, soft tissue mass
in the floor ofthe mouth. It t)?ically has a bluish appearance that has been compared to
a frog's belly, hence the term "ranula."

Of diagnostic significance is a history of increased size just before or during a meal,


and a decrease in size between meals. The treatment is surgical, either through complete
excision or by removing the roof ofthe cysl. If it persists, excision ofthe gland may be
needed.

A sialofith is a stone (salivary calculus) within a salivary gland or duct. The formation of
a sialolith is called sialolithiasis and occurs as a result of precipitation of calcium and
phosphate salts around a nidus ofmucous or bacterial debris. Sialoliths occur as single or
multiple stones and can cause swelling and pain. The pain is experienced during salivary
stimulation and is intensified at mealtimes.
The best radiographic projection for visualizing sialoliths in the submandibular duct and
gland is the standard mandibular occlusal view. Occasionally, sialoliths are seen incidentally on periapical radiographs, in which case they may be misdiagnosed as osteosclerosis.

The rate of occurrence in submandibular gland and duct is much higher than in the
parotid or sublingual areas. This is thought to be due to the tenacity ofthe submandibular saliva and the long and irregular shape of the duct. See picture #64 in booklet
The treatnent ofchoice is almost invariably surgical

extirpation ofthe sialolith. Stones

located in the glandular parench;.rna usually require removal ofthe gland as well.

Of the neoplasms rffecting the mNjor or minor


glands;

the

is the most common.

. Basal cell adenoma

Sebaceous adenoma

. Pleomorphic adenoma
. Ductal papilloma

140
Coplright O 201 I -20 l2 - Dnral Decks

. Sarcoidosis
. Mikulicz's disease

Sj<igren's syndrome

. Hypothyroidism

. Diabetes mellitus
. Malnutritior/Starvation
. Dehydration

. Cystic fibrosis

141
CopJaight @ 201l-2012 - Dnral Decks

Pleomorphic adenoma (benign mixed tumor, is the most common tumor of major and
minor salivary glands. It is a mixture of epithelial and mesench).,rnal elements (pleomorphic).Int\e parorid, 90% occur in the superficial lobe and most conmonly are seen in the
tail ofthe gland. Within the oral cavity, the majority are found in the palate. Pathologically, it is characterized by slow growth and few symptoms. See pictures #65 and #66 in
trooklet
The gross pathologic appearance of a pleomorphic adenoma is a smooth or lobulated.
well-encapsulated tumor that is clearly demarcated from the surrounding normal salivary
gland. They are tlpically solid tumors and may have areas ofgelationous myxoid stroma.
Cystic degeneration or tumor infarction and nectosis are rarely seen except in large, longstanding lesions. Microscopically, these tumors are composed of varying propoftions of
gland-like epithelium and mesenchymal stromas. The treatment ofchoice is surgical excision with a generous margin ofnormal tissue. Inadequate initial removal ofthe mixed
tumor in major glands may result in recurrence. Approximately 25% ofbenign mixed tumors rvill undergo malignant transformation illesions are untreated for an extended length

of tirne.
Sebaceous adenomas are rare lesions composed predominantly of sebaceous gland-derir ed cells; they are well differentiated in the benign forms. The parotid gland is the site
of chief involvement. Parotidectomy is the treatment ofchoice when lesions arise in this
sland.

Ductal papillomas are a rare group ofbenign papillary neoplasms ofthe large excretory
duct. There are three types major subtypes of ductal papillomas of salivary gland origin
thar are about equally rare: the intraductal papilloma, the inverted ductal papilloma, and
the sialadenoma papilliferum. The treatment for all three is conservative simple local excls1(]n.

There are many conditions associated with parotid gland enlargement, this may be unilateral
or bilateral.
. Sjtigren's syndrome: is an autoimmune disorder affecting lacrimal and salivary glands
\\ hich causes decreased moisture in glands. Dry mouth, tooth decay, mouth sores, enlarged
salir ary glands, sialoliths, and recunent salivary gland infections are possible symptoms.
The syndrome also et'fects moisture in the eyes, which might cause chronic eye infections,
comeal ulcers and vision loss.
. Sarcoidosis: unknown cause; believed to be alteration in cellular immune function and in\ ol\'ement of some allergen. Most often involves the lungs; can affect the parotid gland.
Granulomatous fmacrophage infiltates) inflammation causes organ nodularity and loss of
parenchyma. Note: Sarcoidosis is also characterized by distinctive laboratory abnormalities,
including hyperglobulinemia, an elevated serum angiotensin converting enzyme level and
hypercalcemia. Glucocorticoids remain the mainstay oftherapy when treatment is required.
although other anti-inflammatory agents are being used increasingly often.
. \\'arthin's tumor: also called papillary cystadenoma lymphomatosum
. Infections: for example, mumps, actinomycosis, tuberculosis
. Benign lymphoepithelial lesion: also called Mikuticz's disease

. l\4alnutdtion/Starvation
. Dehydration
. Cystic fibrosis and diabetes mellitus

\letabolic conditions associated with bilateral parotid gland enlargement include: chronic alcoholism, dictary deficiencies, diabetes mcllitus, hypertcnsion, hyperlipidemia and Sjdgren's syndrome.

The

lcinic cell crrcinoma

is derived from serous acinsr cells

and is fouRd almost exclusively in the:

Submandibular gland

. Parotid gland
. Minor glands of the palate

. Sublingual gland

142
Coplrighr O

201 1-201 2

, Dnial Decks

. Adenoid cystic carcinoma


. Mucoepidermoid carcinoma
. Acinic cell carcinoma
. Polymorphous low-grade adenocarcrnoma

1/f,3

Copt'righr O

201 I

'2012 - Dental Decks

\cinic cell carcinoma is

the second most common parotid malignancy and the second most conlmc\n
salivary gland ma\gnancy (nucoepidermoid carcinoma is lhe nost conmonlbr both). CrL,"
^:i:rlnc
appearance demonstrates a mass ihat is well-circumscribed but lacks a true caps[le. Microscopic appearance has been categorizcd as solid, microcystic, papillary cystic and follicular. Tumor cells are dark
staining and have granular or honeycomb cytoplasm. The surrounding sffoma often demonstrates a lymphoid infiltrate. Treatment of acinic cell carcinoma includcs surgical cxcision. This tumor is gcncrally
regarded as a low-grade malignancy.

not olheheise specirted) ofthe salivary glands are rare but aggressive tuAdenocarcinomas (NOS
present in the parotid glands. Minor saiivary glands on the palate, lip
mors. About halfofthese trmors
and tongue are the next most commonly affbctcd sitcs. Adenocarcinoma is different from other salivary
gland neoplasms in that as many as 2570 ofpatients will complain ofpain or facial weakness at presentalion. Gross pathology reveals a firm mass with irregular bordcrs and infiltration into surroundin-e tissue. It is generally a solid tumor wjthorit any cystic spaces- These malignancies can demonstrate a wide
range ofgrowth pattems and, for lhis rcason, can bc somewhat difficult to classily. However' all adcnocarcinomas have in common thc formalion ofglandular structures and they are described as grades
I. II or lll based upon thc degrcc of ccllular differcntiation. Grade I lesions havc wcll-formcd ductal
itruclures \\hile Grade III lesions have a more solid growth pattern with few glandular chamcteristics.
Bccausc thcsc are more aggressive tumors, treatment for adcnocarcinoma is more aggressive. Complete
local excision is the mainstay oftherapy.

\lalignant Mixed Tumor

rcpresents thre separate entities that are histologically distinct; These in-

clude the:

. Carcinoma ex-mixed tumor

ofthe three salivary ncoplasms. It occurs when


ofa preexisting pleomorphic adenoma.
. Carcinosaacoma or true malignant mixed tumor: thc mctastatic lesions contain both the stromal
is the most oommon

a carcinoma develops from the epithelial component

end epithelial elements. This is difl'erent from the carcinoma ex-mixed tumor in which only the ep'rl_alial clcmcnti are prc5cnt in mclaslasii.

. \Ietrstasizing mixed tumor


r eloos metastatic deoosits

refers to an otherwise benign acting pleomorphic adenoma that de-

oftumor

\tucoepidermoid carcinoma is the most common malignant salivary gland neoplasm in both minor and
rNaior glands. It devclops rnost commonly in thc major salivary glands, mosl oftcn the parotid (45-70'/o)
Tha sccond most common sitc ofoccurrence is the palate //8Zr). Thcse tumors grow slorvlY and preslr:: as painless masses in most cases. Nlicroscopically, thcsc fumors are charactcrized by the presence
..: t\\ o populations ofcells the mucus cells and the epidermoid cells. the proponion of rvhich helps to
i3ine thc grade ofthc txmor. They arc uncncapsulated or poorly encapsulated and intlltrate srirround_

::r!lirsuefreelt.'High-grade"and"lowgradc"formsexisteachwithadifiercntprognosiskoodforloy'

!,ttti.

poor.fot high-g^rrle/. Surgical cxcision is the ffeatment. See picture #67 in booklet

.\denoid clstic carcinoma /'1CC/ accounts for approximately 23% ofall salivary gland carcinomas. App:o\iDrarcl) 50 to 709/0 occur in the minor salivary glands ofthe palate- In the maior salivary glands. the
prrotrd gland is nrost often affected. Microscopic appearance is described as cribriform, rubular or solid
Th. cribrifonn pattem is the most common and most easily rccognizable. It is often referred lo as the
_s\riss cheese" pattem. Tumor cells arc armnged in ncsts around cylindrical spaces that may contain a
nucrnous or hyalinized material. Cells lhat are arranged in layers and form ductal struclures character_
lzi rhc i.rbular pattem. Thc solid pattem contains sheets offumor cc]Js u'ith lo intent'ning spaces.
See

picture #68 in booklet

.\CC is an unusual tumor It is slow-growing but relentless. It tcnds to be locally invasive and infiltrate
rhe 'iheaths" or coatings surrounding nerve fibers (per-lneurol spaces). ACC often rccuN years later at
ihe site \\here thc fumor firct arose or it mny metastasize. Unlike most carcinomas, it seldom metastasizes to nearby lFnph nodcs but rather to distant sites. The lung is thc most common site ofmetastasis,
\\ rth rhc liver second. Bone metastases indicate a poor prognosis.
PofJmorphous low-grade adenocarcinom (PLGA) is the second most common malignancy in the
minor salivary glands and occurs most frequently in the palatc. lip and buccal mucosa. The microscopic
appearance ofthcsc tumors is what gjves them their name. Any ofa variety ofgrowth pattems (solid, tu'
hula4 trubecular, glandular tribriforn, cysti., can be seen within the same lcsion or among different lesions. PLCA displays a tendency for perinerual and perivascular invasion, howevcr it typically follows
an indolent course. Treatment consists ofconscrvativc yct complete local excision. Distant metastasis is
not common,

. Lupus Erythematosus

'

Sjogren's disease

. Sarcoidosis
. Crohn's disease

Coplrighr

201

141
l'2012 - Dental Dcks

. Malignant
. Mixed ftenign and malignant)
. Carcinoma in situ
. Benign

'145

CopltiSht O 20ll-2012 - Dental DcIs

Sjiigren's syndrome is a chronic lymphocyte-mediated autoimmune disorder aIi'ecting the lacrimal and salias well as other organ systcms. Thc paticnts most commonly afTected arc post-mcnopausal women
who prescnt with dry eycs. dry mouth and, in about 50% of the cases, enlargemen! ofthe parotid and submandibular gl^nds bilarerally. Primary Sjdgren's syndrome consists ofxeroslomia and kcratoconjunctivilis
vary glands

stcc (dry eres), and possibly salivary gland enlargement. Scondary Sjcjgren's syndrome has the same xerostoma and keratoconjunctivilis sicca but is also associated with anothcr autoimmune disordr such as R-{,
SLE, etc. There is dense inflammatory infiltrate $ ith destruction of glandular tissue- Treatmenr is palliative.
Note: Biopsy ofthe labial or palatal salivary glands may bc helpful in establishing the diagnosis, along with
sialograms, salivarj" flow rate tests , the Schimer test ldeterniles \lrcther the e),e procluces enough tears to
*eep it moist)
work. Important: Only halfofSjdgrcn paticnts have anti-SSA and/or anti-SSB /SJd^ndblood
grcn\ Svndronte
A dnd B,) antibodies in their blood. Other tests which may be abnormal include the white
blood cell count //ot . total gamna globulin level /rr'll. blood C3 and C4 complemen! levels /hx C4l. sedimentation rate frigrl aud rheumatoid factor lporltlr?r.

Important: The decrease

ir1

Important: The histological


nign l)

m ph

salivation may cause rampant caries reminiscent ofradiation cariqs.


f'eafures

ofthe salivary gland lesions in both Sjitgren's syndrome and the "be-

opithfial lesion" lalso calle.l

Milrll.:

./rs?.rr., are idcntical.

ponant to remenlber that malignant lymphomrs and "pseudolymphomas" (also cdlled at picdl benign lvehoid h.r!)erpld.rld) develop in some patients who have been diagnosed *ith Sjcigren's syndrome. This
It is

mandarcl clo$ follo$-up oflhe parienl!.


Lupus er]thematosus is a chronic autoimmune disease that occurs in tivo forms:
. Discoid lupus erythem^tos\s (DLE)| is I chronic skin condiiion of sorcs with inflamrnation and scarring fa\oring the facc, can, scalp, and/or oral mvcosl (i.e., buccdl nucosa. gingivd, rcr illion). Thcse lciions delelop as a red, inflamed patch with a scaling ilnd crusty appearance. Oral lesions mimic erosive
fichen planus. Trcated with cortisone or other drldgs (i.e., Pldqucnil. Aralen, or Quinacrine).
. St stemic lupus erythematosus (SIrr: is a chronic inflammatory connectivc tissuc disorder that involvcs
muhiplc organs including the kidneys. heaft,joints, skin, mucous membranes, and blood vessel walls. Abutt.rll)-shapcd rash /ma1dr rdrrl on the t'ace that covers the cheeks and bridge ofthe nose is a c)assic sign\ole: \umercus autoantibodics direcled against nuclcar and cytoplasmic antigcns are fbund in SLE palients.
Oral lesions are generally similar to thosc secn in DLE. Scrologic rcsis (ANA [aubantibodie.\J test and LE
icll re-vr are Dositive in Datients with SLE.

.\pproximatel,v 8 out of l0 salivary tumors diagnosed are in a parotid gland. One in 10 diagnosed is in
l submandibular gland. Thc remaining l0% are diagnosed in othcr salivary glands. In gneral, glands
morc likcl! to show tumor growth are also glands least likcly to show malignant nrmor growlh Thus.
elihr.ugh tumors ofthe sublingual glands arc rarc, almost all ofthem are malignant. In contrast, only
rl.rlrt Iion ofparotid gland tumors are mahgnant.

Clinical leatures of malignant salivary gland tumors:


- Jlucosa is ulcerated - Firm - Painful - Nodulas

Fixed

Rapid growth

\lafignant Safivary Gl^ndTumors (b eflt)t

\denoca rcinoma,

OS lnot olherv,ise specified), arc rare but aggressive tumors About half of

rh.se tumors present in the parotid glands. Adenocarcinoma is different fiom othcr salivary gland neofla5ms in that as many as 259lo ofpaticnts rvill complain ofpain or frcial rreakness ai presentatlon.
{denoid c!,stic carcinoma: approximateiy 50 to 7oyo occur in the minor salivary glands of the
talate. In the major salivary glands, the parotid gland is most often affected. Microscoplc appearance
is described as cribriform, tubular or solid. The cribriform pattern is the most common and most
carily recogrizable. It is often refened to as the "swiss cheese" pattem.
. \lucoepidermoid carcinoma: is the most common malignant salivary gland neoPlasm in both

'

minor and major glands. lt develops most commonly in the major salivary glands. nost otien the
patotid G5-70a.,'). Microscopically, these tumors arc characterized by the prescnce of two populations
ofcells thc mucns cells and the epidermoid cells, the proportion ofwhich helps to define lhe grade
of thc tumor
. Acinic cell carcinoma: Acinic cell carcinoma is the second most common parotid malignancy and
thc second most common pediatric salivary gland malignancy ih&coep[dermoid carcinona is lhe osl
t'onnnon lor hoth).This tumor is gcncrally regarded as a lort-grade malignancy.
.llalignant mixed tumors: represents three scparate entities that are histologically distinct; These
includc thc carcinoma ex-mixcd tumor, carcinosarcoma or true malignant mixed tulnor, and thc metaslasizing mixed tumor.
. Polymorphous low-grade adenocarciliotn (PLGA) is the second most common malignancy in
the minor salivary glands and occurs most frequently in the palate, lip and buccal mucosa. The microscopic appeamnce ofthesc tumors is what gives them their name.

ORAL PATHOLOGY

Oncocytomas are
tumors that constitute about
of benign epithelial salivary gland neoplasms.

. Common; 50o/o
. Common; 75%

. Rare; 2%
. Rare;

l5olo

'146

Copyright C

20ll-l0l:

- Denkl Dects

ORAL PATHOLOGY

A 65-year-old patient comes to your oflice complaining of a slowly-growing


enlargement ofthe jaw. You palpate the angle of his right ramus and find
an encapsulated mass that is non-tender and firm. Your oral pathologist
defines it as a glandular and cystic tumor lined by a bilxyered (inner
columnar oncoc!,tic and outer basal) epithelium with a lymphoid stroma.
Name this second most common benign neoplasm of the parotids.

. Pleomorphic adenoma
. Warthin's tumor
. Fibroadenoma
. Monomorphic adenoma

147

CoDrigllr t) 20ll-201? - Dental Decks

The term "monomorphic adenoma" refcrs to a group ofrare salivary tumors that includes thc basal cell
adenomas, canalicular adenomas. scbaccous adenomas, myoepithcliomas and oncocytomas.

Oncocytomas arc rare tumors that constitute only 2% of benign epithelial salivary gland neoplasms
Thc majorjty ofthese tumors affcct lhc parottd gland 178%). The clinical prcscntation ofoncocyomas
is esscntially identical to other bcnign salivary tumors a slowly growing, nontender mass. typically
in the superficial lobe
cytes.

ofthe parotid. Microscopically, thcrc arc shccts,

nests or cords

ofuniform onco-

Basal cell adenomas are also rare tumors that constitute only J'lo ofbcnign epithelial salivary gland !umors and gpically occrrrs in the 6th decade oflife. It seems to occur morc frequently arnong Caucasians
than African Amcricans. The majority occur in the parotid gland where thcy present as a slowly cnlarging firm mass. They are well-encapsulated, smooth tumors on lross inspection and arc divided into four
subtypes based on their microscopic appearance solid, trabecular, tubular and membranous

Canalicular adenomas are also rarc ftrmors (1.596) that most commonly involve thc minor salivary
glands of the npper lip (74yu) or brc.al mrcosnl ( 12a;).It peaks in thc 7th decade of life and, like tlle
basal cell adcnoma, is morc common in whites than blacks Clinically it presents as a nonpainful submucosal nodule. On gross pathologic examination, canalicular adenomas may or may not possess a cap_
sulc and it is not unusual for therc to bc multifocal glowth. Microscopically there are cords ofsingle-layer
columnar or cuboidal cells forming duct-like structures in a background of fibrous stroma
Thc rarc m!oepithelioma accounts for less than one perccnt ofall salivary gland ncoplasms. Most arise
\! irhin the parotid gland and, less frequcntly, in the submandibular gland and intraoral minor salivary
glands. Three paftems ofmicroscopic appearance have been describcd. The spindle cell pattern is the
mosl common ovcrall and is typical for parotid myoepitheliomas. The plasmacytoid pattern is less
common but the most frequently encountered pattern in palate nlmors. Tle third pattern demonstrates
3 combination ofthc spindle and plasmacytoid cells and is uncommon.

\ot: Sialoscintigraphy

is a simplc and non-invasive procedurc that can usually scparate benign enti-

like \\'arthin's tumor and oncocytoma ofthe salivary glands from maligrant tumors, and signifiaanrlv affcct the course oftrcatment.
rres

c! ameforWafthin's tumorl ls almost exparotid neoplasm. lt is a glandular and cystic tumor lined by a bilayered (inner
.olunindt on.oc)lic and outer basal) epithelium with a lymphoid stroma.
Papillar)- cystadnoma lymphomatosvm (afq
cltLsir elv a

Clinical features: The vast majofity ofpatients are over 50 years ofage, with a 5:l male predc'minance. Approximately l0To to l5Yo of the tumors are bilateral. The tumor most often
arises in the lorver pole ofthe parotid and presents clinically as a non-tender' slowly enlarging. flmr to fluctuant nodule oyer the angle or ramus ofthe mandible.

Histologic features: The tumor is encapsulated and co[tposed ofcystic spaces containing an
.r)sinophilic coagulum into which extend papillary projections of the lining epithelium. The
epithelium consists ofa double row ofcells with eosinophilic, granular cytoplasm, a luminal
lay er oftall columnar cells and a basal layer ofround, cuboidal or polygonal cells. lnterspersed
among the cystic spaces are aggregates of lymphoid tissue, some with germinal centers.

Treatment and prognosis: Surgery is the treatment ofchoice and recurrence is uncommon.
\lalignant vadants ofthe tumor have been reported but are mre.

\ote: Pleomorphic adenomas are the most common parotid tumor lt grows slowly and is
benign. A pleomorphic adenoma begins as a painless lump at the back ofthe jaw just below
rhe earlobe. These are more common in women.

A 40-year-old female presented with a subcutaneous nodule on


the right latcral surface of her tong\e (afiort is poinrtng tu lesion).
The nodule was non-tender, soft, movable and had been slowly
growing for about 2 years. The nodule was excised. Its eut surface
was yellow and lobulated. What is the clinical signilicance ofthe
nodule being movable and slowly growing?

148

CopriSlrr O

20ll ?01?

Denral Decks

ORAL PATHOLOGY

A developmental dfect characterized by an overgrowth of tissus


NORMAL to the organ in which it arises is a:

. Teratoma
. Choristoma

. Hamartoma
. None ofthe above

't

49

Copyrighl e 201l-2012 - Denral Decks

Gcncral charactcristics of benign neoplasms:

.
.
.
.

Well-diffcrcntiated
Slow growth
EncapsulatedAVell circumscribcd
Localized

Gcncral charactcristics of malignant neoplasms:

. Invasion

. Immovable
. Rapid growth
. Metastasis
. Not well-differentated (or qnaplqstic)

Important:
. Paresthesia is suggestiv of metastatic

disease

. Metastasis is the most important characteristic that distinguishcs malignancy from benign
Radiographically, a benign neoplasm in bone may be diffcrcntiated ftom a malignant neoplasm in the following rvays:

ln a benign lesion, the cortex tends to remain intact but may be thinncd and thc pat

involved expanded
ln a benign lesion, the margins are usually defined and demarcated lrom surrounding

bone

***

Te

The nodule in the picture on thc front ofcard is a lipoma.

rms used in oncology:

. Oncology: the study ofneoplasms


. \eoplasm: an uncontrolled new growth oftissue
. Tumor: a Iocalized swelling, may or may not be a true neoplasm
. Hlperplasia: an increase in the size of a tissue or organ due to an increase in the
nurnber of component cells
. H]'pertrophy: an increase in the size of a tissue or organ due to an increase in the
size of component cells
. Cancer: a general term for all malignant neoplasms
. Carcinoma: a malignant epithelial neoplasm
. Sarcoma: a malignant mesenchymal (connective /i.tsae,,) neoplasm
. Hamartoma: a developmental defect characterized by an overgrowth oftissues normal to the organ in which it arises
. Teratoma: a neoplasm composed of multiple tissues foreign to the organ in which
it arises; may be benign or malignant

. Metaplastic

. Hwerplastic
. Araplastic

. Dysplastic

150
Cop)r'ghr O

201

l-2012 - Dntal Dcks

Metaplasia most commonly occurs by rephcement

of

. Cuboidal cells; columnar cells


. Columnar cells; stratified

-bv

squamous epithelium

. Columlar cells; pseudostratified colurnnar cells

. Cuboidal cells; stratified squamous epithelium

151

Cop)rigltt O 20ll-2012 - Denral Decks

Lack of differentiation, or anaplasia, is considered a hallmark of malignant transformation. It is marked by a number ofmorphologic and functional changes. Both the cells
and the nuclei characteristically display pleomorphisrn variation in size and shape.
Characteristically the nuclei contain an abundance of DNA and are extremely dark staining (hyperchromatlc). The nuclei are disproportionately large for the cell, and the nuI :l instead of the normal 1 :4 or 1 :6. The nuclear
shape is usually extremely variable, and the chromatin is often coarsely clumped and distributed along the nuclear membrane. Large nucleoli are usually present in these nuclei.

clar-cytoplasmic ratio may approach

Histological grading of malignant neoplasms:

. An

attempt to estimate the aggressiveness of degree of malignancy of a rnalignant


neoplasm based upon the degree of differentiation of the component cells and the
number of mitoses.

Grade

l. Well-differentiated

Grade 2. Moderately well-differentiated


Grade 3. Poorly undifferentiated
Grade 4. Undifferentiated

.Applicable mainly to squamous cell carcinomas. Most pathologist use three grades and
prefer to designate squamous cell carcinomas as well differentiated, moderately well-differenriared or Doorlv differenliarea.

\Ietaplasia is the process whereby one cell type changes to another cell ty?e in response
to stress and generally assists the host to adapt to the stress. The most common type of
epithelial rnetaplasia involves rplacement of columnar cells by stratified squamous epithelium.

Dlspfastic cells exhibit considerable pleomorphism (vqriation in size and shape) and often
possess deeply stained (hyperchromatic) nuclei, which are abnonnally large for the size ofthe
cell. It is associated with chronic irritation ofa tissue by a chernical agent, such as cigarehe
smoke or by chronic inflammatory irritation, such as chronic cervicitis. The tissue appears
some\\ hat structureless and disorganized and may consist ofatypical cells without invasion.

Epithelium exhibits acanthosis 6rrr.ll r-t an abnorual thickening ofprickle cell layer).
Important: When dysplastic changes are marked and involve the entire thickness ofthe epirhelium. the lesion is considered a pre-invasive neoplasm and is refened to as carcinoma in
sttu.
*** \{ild to moderate changes that do not involve the entire thickness ofepithelium may be
relersible, and with removal of the putative inciting causes, the epithelium may revert to

normal
Histologic features of malignancy:

. Anaplasia . Hyperchromatism ' Pleomorphism '

Abnormal mitosis

The host response to a malignancy is best reflected by lymphocytic infiltration at the edge of
a tumor The most characteristic feature ofa malignancy as opposed to an inflammatory Iesion is that a malignancy will grow after removal ofthe causative agent. The most important characteristic ofmalignant neoplasms, which distinguishes them from benign neoplaslns,
is their

abilitv to metastasiz.

. Recurrent apthous minor


. Recurrent aphtous major

. Recurrent herpetiform aphthous


. Recurrent herpetic stomatitis

'152
CoplriShr O 20ll-2012 ' Dental Declc

. Stomatitis
. Lesions ofthe eye
. Genital lesions

. Maculopapular

rash

153
Copyrighr O

20lt 2012, DentalDecks

*** Recunent aphthous ulcers are commonly referred to as "canker sores" by 1ay persons- In the literafure othcr terms include aphthous stomatitis, rccurrent aphthous stomatitis, rccurrent ulcerative stomatitis or ulcerative stomatitis.

Three classifications:
1. Recurrent aphthous

minor:

807o occur on non-keratinized movable mucosa. See

picture #92 in

booklet
2. Rccurrcnt aphthous major: heal often with scarring, more common in HIV patients. occur on the
soft palate, tonsillar fauces, labial and buccal mucosa, and tongue. See picture #93 in booklet
3. Recurrent herpetiform: occur in up to 100 at a time, occur on any mucosal surface, heal without
scarring. See picture #94 in booklet
*** All three classifications present painful recurrent ulcers.

Importanti Vcsiclcs do not precede the formation ofthese painful recurrent ulcers. They appear on wet
tnot |ennilion) nonkeratinizcd oml mucosa (i.e., not hanl palate).
Remmber: Herpetic lesions are preceded by vesicles and are more likely to be found on keratinized
tissue. See picture #95 in booklet

\ote:
hcet

Systemic diseases in which aphthous ulcers are seen include: Crohn's disease /rarnor aphthae),Besyndrome ftnln or aphthae), Celiac sprue (minor aphthae). and AIDS (maior aphlhae).

These ulce6 appear to be associated with stress. The stress factors may include:

.
.
.
.
.
.

Bacterial infection
Trn[fia li.e., self-i ticted, oral surgen, procedures, routine dental procedures)
Endocrine conditions (i.e., a lemales menstrual period)
AIIergic factors fi.e., ceittain foods or drugs)
Immunologic abnomalities
lr,Jn- Viramin B or folic acid deficiencres
*** The calse is unknown; however, evidence supports they are related to the focal immune dysfunction where T-lymphocytcs play a major role

Stevens-Johnson syndrome fS-/S/ is an immune-complex-mediated h)?ersensitivity complex that is a severe expression of er''thema multiforme. It is now known also as erythema
multiforme major ln SJS, the systemic symptoms are sever and the lesions are xtensive,
inr olling multiple body areas (especiqlb, the mtrcous membra es).

Tl?ical
.
.
.
.

s\ mptoms are as follows:


Cough productive of a thick purulent sputum
Headache

\lalaise
Arthralgia

The tvpical " bull's-eye-shaped" target lesions are present. These lesions are considered
parhognomonic.The classical triad ofthis SJS consists ofeye lesions, genital lesions and
stomatitis. lmportant: The lesions ol SJS are severe and often vesicular or bullous,
r ith hemonhage after denudation. Note: Blindness can occur due to secondary infection.
Treatment of SJS is primarily supportive and symptomatic:
. Vanage oral lesions with mouthwashes
. Topical anesthetics are useful in reducing pain and allowing the patient to take in flurds

. Areas ofdenuded skin must be covered with compresses ofsaline


. Underlying diseases and secondary infections must be identified and treated
. Offending drugs must be stopped
Drugs and malignancis are most often implicated as the etiology in adults and the elderly. Pediatric cases are related more often to infections than to malignancy or a reaction
to a drus.

ORAL PATHOLOGY

Ulc Cond

The lesion below is a. smnll (2mm-Smm in diameter), whitish sore with a red
border. The patient strtes that it usually begins as a reddish area with a
trurning or tingling sensation. The most likely diagnosis is:

154
Copyrighr

lil20ll

1012

'

Denlal Deck!

ORAL PATHOLOGY

Ulc Cond

nrythema multiforme (EM) is an acute self-limited eruption characterized


by a distinctive clinical eruption, the hallmark of which is the:

. Chronic desquamative gingivitis


. Petechial hemorrhage
. Iris or target lesion
. Mucocutaneous

rash

155
Coplrrght O

20ll

l0l:

Denral Decks

There are three forms or classifrcations ofrecunent aDhthous stomatitis fuhich is also
called recurent aphthous ulcers, canker sores, etc.)

l.Recurrent aphthous minor: is the most common form of the disease and the
one referred to by the lay public as the "canker sore." The lesions occur somewhat
more frequently in women than in rnen. Minor aphthae have a propensity for movable
mucosa that is situated over minor salivary gland tissue. The lesions begin as a single
or multiple superficial erosion covered by a gray membrane. The lesion is typically
very painful. The lesions vary in size from 2-3 mm to over l0 mm in diameter They
generally persist for 7- l0 days and heal gradually with little or no evidence of scarring.

2. Recurrent aphthous major: is characterized by the occurrence of large


(5-20 nm) painful ulcers, usually one to ten in number. These ulcers occur
at frequent intervals and rrany patients with this disease are seldom free from the presence ofat least one ulcer Unlike the minor aphthous ulcer, these lesions persist for up
to six weeks and leave a scar upon healing.

3. Recurrent herpetiform: the prominent feature of the disease is the numerous,


pinhead-sized, gray-white erosions that enlarge, coalesce, and become irregular ulcers.
Ulcers occur in clusters of 10 to 100.
Remember: Vesicular lesions do not precede the formation of ulcers in all of the
abor e. This is a distinctive diagnostic feature.
In healing ofan ulcer, the epithelium that eventually will cover the defect is derived
liom intact epithelium al the ulcer margin.

\ote:

Erlthema multiforme frMl is a hypersensitivity reaction that occurs in mild and scvere forrns lt
produccs tissue reactions ccntered on the superficial vcsscls of lhe skin and mucous menbranes.
Pre.ipilating iactors includc infcctions such as herpes simplex, mycoplasma pneumonia and histoplasmosis. dmgs, radiation therapy. etc. lt occurs primarily in children and young adults. The dia!:no5is oferythema multilorme is primarily based on the classic skin lesion appearance.
Clinical features; 3 forms:
. E\1 minorl prirnarily involves thc skin with only

2570 in oral mucosa. A low-gradc fcvcr, gcneral malaise and headache usually precede thc appearance ofthe lesions by 4 to 7 days. Assoclared * ith secondary herpes simplex hypersensitivity. Oral lesions appear as .cd macules, papules
or \ esicles that may become erodcd and painful. Note: These lesions are covercd by a yellowish-\t hitc membnne aftcr rupturing. Focal or diffuse arcas oferytherna follow and the skin "targct" or "bulls eye" lcsions appear See picture #69 in booklet

. Chronic ENI Minor: mildcst foml

and lesions are small in sizc and shortcr in duration. Oral


lesions rary from focal erosions similar to aphthous ulcers to more dilluse areas of erythema or
erosions that are painful.

. Elu Nfajor lterers-Johnson Syndrome): acute form that involvcs the skin and mucous Inelnbrane. Large bullae form on the mucous membranes and skin. A positive Nikolsky's sign is common. The bullae collapsc which produces a whitish pscudomembrane on the tnucosa and dark
red crusted lesions on the dry skin surfaces (lips and eyes very common).

Note: Behcet's syndrome is a rare disorder that causcs chronic inflammation in blood vessels
rhroughout the body. Manifestations include oral and genital aphthous-type ulcers. conjunctivitis,
uveitis, arthritis, headache, and other CNS symptoms. Treatment with corticosteroids and other immunosupprcssive drugs prcvents serious complications, such as blindness.

. Coccidioidomycosis
. Histoplasmosis
. Tuberculosis

. Actinomycosis

Scarlet fever

156

Copfight O 2011,2012, Dntal Decks

. Syphilis
. Gonorrhea

. Chlamydia
. Tuberculosis

't57
Cop)ri8hr O

201 I

'2012 - Dertal Dcks

Actinomycosis is a subacute{o-chronic bacterialinfection caused byActinom)ces israellii, an anaerobic, graln-posilive filamentous bacterium. The infection is not a contagious discasc and can not be spread fron person lo pcrson.
Inlcctions appear after trauma! surgery or previous infection.

Clinical fcatures:
. \4ost infections occur in thc lhorax. abdomen and head and neck
called cervicofacial actinomycosis /i,
fection n'picall.v occurs i puti?ntsIi!hpoorlental hryiene or lilloti gorc| surgen)
. oftcn prescnts as a swelling ofthe nandible and usually is painful
. Once in the tissue, Actinomyces forms an abscess, prcducing a hard. red-to-reddish-pu4)le Iump, olien on the

"lump) jalr" *** See picture #70 in booklet


Exudatc from thc draining sinus tracts often contain small, clinically visiblc ycllow-green calcified structures
(sulfur grunules)
arc adually colonics ofinficting organisms

jaw

-\\hich
*+* Treatnrent for aclinomycosis

is

long-term penicillin.

Histoplasmosis is a chronic lung infection caused by inhalalion ofspores of Histoplasma capsulatum. It is endcmic ro rhcAmerican Mid\r'esr. The lassic oral manifestation ol histoplasmosis infection is a chronic nonhealing ulcer. Note: The lesions are usually covercd bv a non-specifrc gray membranc and are induratcd.

Clinical features:

. Fe\er.

malaisc, cough, and dyspnea

. Ccrvicll lymphsdenopathy is common


. In chronic fonns, dissemination to the skin and oral mucous membrancs may be the first sign ofinfec-

"**

Treatmenr for histoplasmosis is an)photericin B, itraconazolc or ketoconazole for 6-12 months.

Coccidioidomlcosis (also ktlo\n as Mller lever) results fron inhaling the spores olCoccidioides species
'C,iLtJi)id!\ i,tmilis or Coccidioicles posada';ir. Most infections in the United States arc acquired within the
l|: ,,: reuions ofcndcmiciq ofsouthern Arizona, centfitl or othcr arcas ofClalifornia. southcm Nc$'Mcxico,
::rc \.i: Tc\as. Symptoms are similar to histoplasmosis and it is usually treated $ iih amphotericin B.

\otc:

Scarlet

feier

is a systemic infection causcd by Strcptococcus pyogenes. It

i!

characterized by pharyn-

irirs. l!!cr. malaise, strawberry tongue (hos d :||hile coating with red, inflaned fungiforn pupillae) and ^
ikr:r :ash Important: The p!'rogenic (et_rlhrogeni() e\?toni,'? causes the rash ofscarlet fevcr and systemic
:.,\ ta ihock slndrome.

Srphilis is an inf'cctious. contagious venereal discase that is caused by the spirochetc treponema

pallidum. It is one of the

less common scxually transrnitted infections /S11/. The incubation pe-

nod \aries some$.hat bctween 10 and 90 days. Thc sylnptonls ofsyphilis may pass unrecognizcd,
!rr Dra\ be nlisintcrpleted and at times there arc no initial symptoms at all. Whcn prcscnt. thc clasirJai s\mptoms ofsyphilis lnanifest themsclves in threc stages as lbllows:

L Primarr': the first symptom is a non-painlil chancre that generally appears 2-6 weeks alier
e\l-r.)sure. lt usually is fbund on thc part ofthe body exposed to thc partner's ulcer. such as thc
tL'ni\. thc \ul\a, or the vagina. lt can also develop on thc cervix, tonguc. lips. or othcr parts of
lhe brrd) The chancrc disappears within a l'erv wccks whethcr or not a penon is trcated. Ifnot
lrcatcd during thc primary stagc. about one third ofpeoplc will progress to chlonic stages.

i.

Secondarl: is a highly infectious stage; it occurs 6 weeks aftcr non-treatment of primary


:r philis. \\'idely dissetrinated spirochetes cause mucous lnerlbranes 10 cxhibit a reddish brown
nraculopapular cutaneous rash and ulcers that are covered with a nucoid cxudate ku/led mu.ot,r pdtches). Condylomata lat^ (\'hich are elevated bntad-based pluquesl are also seen on
ikln and mucosal sr-rrl'aces. lf left untrcatcd, thesc symptoms will rcsolve on their oq,n but thc
iniictious microbc remains behind. It is at this point that syphilis passes into its latent phase.
This \ilent period ntay last fbr many ycars and permits the infcction to evolve wi{hout any ob\ rous cxtemal symptoms. At this point. the only nrcthod ofdctecting the prcscnce ofsyphilis is
\ i r., blood lc.l spccilic lor syphrlis.
.l. Tertiary: occurs in inf'ected persons many years alier non-trcatment ofsecondary syphitis. Thc
gumma (*,lrich is a .fotal nodular nlasr) typifies this stage. lt most cornmonly occurs on lhe
palate and tonguc. The bacteria damage the hcart, eyes, brain, nervous svstem, bones.joints, or
almosl any othcr part ofthe body. Note: Headache, stit'fneck, and f'ever are symptorrs of neur-

os)philis.
Remember: Conorrhca is a sexually transmitted disease caused by Neisseria gononhoeae. An oral
manifestation of gonorrhea is oral pharyngitis.

158
CoplriShr

201|,2012 - Dntal Deck

. Coccidioiodomycosis
. Mucormycosis

. Aspirgillosis
. Zygomycosis

'|

59

Cop)'righr O 201 I -2012 - Dental Dcks

Congenital syphilis is caused by infcction with thc spirochctc Treponema pallidum during the
fetal period. Expectant mothers who have syphilis can transmit the disease through the placenta to
the unbom infant. Nearly halfofall infants infected with syphilis during gestation die shortly beforc or after birth.
The severity ofcongenital syphilis depends upon l) The time in which the organisn]s pass the placental barier (protected up to I6th week), 2) The mother's stage of syphilis and, 3) The immunologic rcsponsc ofthe fctus. Iftrcated by the 4th or 5th month, 957n show no manilestationsi ifnot
feated, fetal sepsis may result in stillbirth or visceral and mucocutaneous manifestations.

l. Symptoms in the newborn


.lrritability
. Failure to thrive
. Bloody discharge from the nose
. Saddle nose, lrontal bossing and short maxilla

. Later rash: coppcr-colorcd, vcsicles on the palms and soles


. Early rash: small blisters or a flat bumpy rash on the lbce. palms

and sole

2. Symptoms in older infant and young child

. Bone pain
. Joint swelling
. Abnormal teeth fHrbhinson's incisors)

Saber shins r'borre abnormalitT-

. Gray, mucous-like patches on

qfthe lower leg)


andy\tlya (cond),lona lata)

the anus

. Visual loss. CN VIII nerve deafness and intentitital keratitis


. Scaning of the skin around earlier lesions of the mouth, genitalia, and ants (called rhagctdesl

Remember: Hutchinson's triad

combination of Hutchinson's teeth, interslitial keratosis,


and deafness due to lesions ofthe eighth cranial nerve. Note: "Screwdriver" incisors and "Mulberry molars," dcntal defects seen in congenital syphilis are caused by direct invasion of tooth
germs by Treponema organisms.

-the

The gencra most commonly rcsponsible for mucormycosis usually are Mucor or Rhizopus. Orbitorhrno-cerebral mucormycosis, the most common typc, genemlly occurs in conjunction rvith sinus
rrr nasal involvement. Mucormycosis also may aflect other parts ofthc body, including the lungs,
GI tract. or skin.

Diabetic patients arc predisposed to mucormycosis because of thc decreased ability of thcir neurrophrls 10 phagocytize and adhcre to endothelial walls. Furthermore, the acidosis and hyperglycemia provide an excellent environrnent for the fungus to grow.
Othcr patrent. at risk include thc follou ing:
. Patients on chronic antibiotics, steroids, or c),totoxic therapy
. Palienls with chronic renal failure or liver problems

. Parients with transplants

Palients with cancet HtV, malnutrition or acidosis

Important: In the head and neck, most lesions appear

as destructive ulcerations

in the paranasal

sinuses or nasal cavity.

The use of systemic amphotericin B is important in treating mucomrycosis; its use, along with incrcased awareness ofthe disease, has decrcascd thc mortality

Rhizopus is the principal cause ofzygomycosis, which occurs primarily in patients suffcring from
diabetic ketoacidosis (thinocerebral diseasel, malnutrition, scvcrc bums, or who are itnmunocompromrsed.

Aspergillosis is a group ofillnesses caused by mold. In some people, the spores trigger an allergic
reaction. Other people develop mild to serious lung infections. The most serious form of as-

pergillosis

invasive aspergillosis occu6 when the infection sprcads to blood vessels and beyond, into the lungs to othcr organs. Note: Aspergillosis is the second most frequently seen f'ungal
infection ofthe f'ace and mouth in patients receiving chemotherapy. It is second only to candida in
its fiequcncy. Oral lesions appear as necrotic ulcerations with gray pscudornembranes ofthe ginsiva and hard palate.

ptint comes with hr mother for a routine appointment. The


mother states that her d.ughter just started not feling well and had a mild
fever earlier in the day. Th daughter has ben having troubl swallowing.
An intraoral exam reveals multiple 1-2 mm vesiculopapular lesions ofthe
nasopharynx and soft palate. Your working diagnosis is:

A 4-year-old

. Herpangina
. Hand-foot-and-mouth

disease

. Herpes simplex infection


. Pemphigus lrlgaris

'!

Coplrigbr

iO

60

20ll-2012 , Denul Decks

ORAL PATHOLOGY

Which type ofherpes virus is associated with the lesion on the lower lip?

. HSV- l
. HSV-2
. HSV-3

. HSV-4

161

CopFighr C

201

l-201 2 - Denral Decks

Herpangina refers to a stomatitis (inflammotion ofthe mouth) caused by a strain olthe


Coxsackie- virus. It is differentiated in clinical practice from Type I Herpes infection (the
cold sore virus) by the fact that the ulcerations ofherpangina are localized to the posterior soft palate and nasopharynx. Remember: Herpes Type I lesions are found typically more forward in the mouth on the tongue, gingiva, buccal mucosa and appear as
vesicles (smal/, cleqr blisters that ulcerate and crust) around the mouth and on the lips.

Clinical features:
. Most commonly affects infants and young children
. Typically occurs during the summer
. Typically spreads via the fecal-oral route or via the respiratory droplets
. Symptoms are mild and short in duration (no more than I v'eek)
. Sore throat and difficulty swallowing
. Mild fever
. Small vesicular or punctuate lesions with white base on posterior soft palate near
uvula and anterior fauces ofthe tonsils *** See picture #71 in booklet
Important: The disease usually runs its course in less than a week. The treatment is palliative.

\ote: Hand-foot-and-mouth disease, is a highly contagious systemic infection caused


A of limited duration in which vesicular eruptions occur on the palms of

coxsackievirus

hands. soles offeet and mucosa

oropharyngeal

are

*** Remember:

ofthe anterior part ofthe mouth. It is uncommon in the

a (unlike herpangina).

The majority

oforal herpes cases

are caused by

HSV-l

and the majority

of

genital herpes cases are caused by HSV-2.


Oral and para-oral presentations of Herpes Simplex Type

include;

. Herpes fabialis (also called fever blisters or cold sores) is an extremely common disease
caused by the herpes simplex virus Type I . charactedzed by an eruption of small and usuaLll painful blisters on the skin ofthe lips, mouth, gingiva, or the skin around the mouth.
\ote: The reason most patients suffering from recunent herpes labialis rarely give a history
of having had actne herpetic gingivostomatitis is that the pdmary infection rvas subclinical.

. -{cute herpetic gingiyostomatitis (also known as primary herpeti. gingivostomqtltis)


generall-v'. affects children under the age ofthree and young adults. There are prodromal
slmptoms (,fever, malaise, irritabiliry, headache, clysphagia, vomiting, b,mphadenopdth)')

-2 days prior to local lesions. Then small, yellowish vesicles form, which rupture quickly.
resulting in shalloq round, discrete ulce$ with an eDthematous halo. It affects both the free
and attached mucosa. A generalized marginal gingivitis may prccede the ulcers. Treatment
includes fluid intake, good oral hygiene and gentle debridement ofthe mouth. ln healthy individuals the lesions heal spontaneously in 7- l4 days without a scar.
l

. Recurrent /secord4ry, herpetic stomatitis generally occurs in adult patients and is triggered by trauma, fatigue, immunosuppression, stress, allergy or sunlight, which causes the
release (or rcdctivalioz) ofthe latent HSV-l virus in the trigeminal ganglion. This reactivation causes a recurrent infection f.e., told soles) on the lips (that is bound to periosteuml, hard palate, attached gingiva and alveolar ridge. Site-specificity is a characteristic
manifestation. Note: Lesion on the finger is called herps whitlow.

. Cytomegalovirus

. Epstein-Barr virus
. Herpes simplex virus I
. Herpes simplex virus 2

162
Cop)righr O 201l-2012 - Dental Decks

. Lipschutz bodies
. Civatte bodies
. Lisch nodules
. Reed-Stemberg cells

163
Coplright O20ll-2012, Denhl Decks

Primary herpes is most common in children and young adults. Patients develop fever. irritability, regional lymphadenopathy and headache. Within days. the gingiva becomes intensely inflamed. Any pan of the oral mucosa and lips may become involved. Vesicles
then form and rupture a short time later to leave shallow ulcers covered with a gray membrane and surrounded by a red halo. These ulcers are very painful. These ulcers will heal
on their own within 7 to 14 days.

After recovery from primary HSV infection, the virus is not cleared from the body, but,
rather. it lies dormant in a non-replicating state, in the sensory nervous system (specilicall,r, the trigeminal ganglion/. Periodically, Iatency reactivates. allowing the virus to
retum to the skin or mucous membranes, where it causes a recurrent inlection. Cold sores
are a manifestation ofrecurrent herpes simplex virus infection around the mouth. The
lr.rost common site is on the lips. Some factors that are often associated with a recurrent
outbreak are: sunbum, fatigue, emotional upset, trauma, upper respiratory tract infection
or menstruation. Often a day belore the formation of vesicles there will be a tingling or
itching ofthe skin or mucosa. Vesicles ulcerate and resolve the same as in the primary ini-ection.

Histofogically, the cytopathic effect (CPE) take the form ofballooning degeneration of
rhe epithelial cells with loss of cohesion to adjacent cells. The nuclei are olten multiple

*ith

margination ofthe chromatin around the intra-nuclear inclusions called Lipschultz


bodies. These changes can be seen in scrapings taken from an unroofed vesicle (tlrcse
.rcrapings are callerl a Tzanck smear).

The Tzanck smear is done by smearing cells taken from a fresh blister or ulcer onto a
mrcroscope slide. The cells are stained with a special stain, such as Wright's stain, and
then examined under a microscope for characteristic changes caused by a herpesvims.
Herpes causes giant cells with multiple nuclei. The shape ofeach nucleus appears molded
ro t'it together u ith those adjacent. The background ofthe cell looks like ground glass and
contains snall dark spots called inclusion bodies (Lipshutz bodies).
Orher tests that can be used to diagnose herpetic lesions include:

Fluorescent staining: cells show positive fluorescence when stained with fluor-

escent labeled HSV immune serum and globulin. This procedure is used to distinguish
benr een herpes zoster and heryes simplex.

.Isolation in tissue culture


. Antibody titers (anti HSV ontibod!- titers)i is a test for complement fixing or neutralizing antibody in acute and convalescent sera as well as on tissue sections (this begins irt one v:eek arul peaks in three veeks).
Biopsied material will show an intraepithelial cleft covered by an exudates offibrin and
polyrnorphonuclear leukocytes. The epithe|um willexhibit degenerative cells, which include bizarre giant cells and cells with displaced chromatin with perinuclear halos and
inclusions.
The treatment for herpes is

primarily supportive:

. Analgesics
. Topical anesthelics prior to eating
. Maintain electrolyte balance
. Antiviral agents

. Herpes zoster
. Herpangina
. Recurrent herpes
. Chicken pox

tu
Copynghl O 201l-2012 Dntal Decks

. Manifested

as

ANUG

. Associated with HIV

. Subclinical
. Characterized by severe lymphadenopathy

and acute dermatitis

165

CoprrightO 20l l-2012 - De al Deks

The varicella zoster virtrs (VZV) is a member ofthe herpes virus group. It causes the disease chickenpox (varicella) and shingles (herpes zoster). The virus is very contagious and
may be spread by direct contact or droplets.

Chickenpox is primarily a disease of childhood, which peaks at school-age in the winter


and spring. lt is characterized by the appearance on skin and mucous membranes ofsuccessive crops of typical pruritic vesicular lesions that are easily broken and become
scabbed. lt is generally accompanied by mild constitutional symptorns (/everr malaise).lt
is most contagious one day before the onset of the rash and until all the vesicles have
crusted. It is relatively benign in children, but adult infection may be complicated by pneumonia and encephalitis. Note: ZIG (Zoster Immune Globulin) reaches morbidity in highrisk children.
Shingles (herpes zosle, is the result ofreactivation ofa latent varicella-zoster virus that
is believed to reside in the sacral ganglia from a childhood case ofchickenpox. The virus
reaches the sensory ganglia ofthe spinal and cranial newes, producing an inflammatory
response. It is characterized by painful vesicles that occur on the skin or mucosal surfaces
along the distribution ofa sensory nerve in a distinctive unilateral pattern.

\ote:

The histology for both chickenpox and shingles shows the same cytopathic effect

as seen in herpes simDlex.

Herpes simplex is one ofthe most common viral diseases affecting man. The primary infection. rvhich is known as primary herpetic gingivostomatitis, is most common in
r oung children /rin der live yeors old). It usually occurs in a child who has had no contact
u ith the Type I herpes simplex virus, and who therefore has no neutralizing antibodies.
h mal also affect young adults //J-25). Nearly all primary rnfections (90o/o) are ofthe subclinicaf type (they moy only have flulike symptoms) and one or two mild sores in the
mouth lvhich go unnoticed by the parents.
In other children, the primary infection may be manifested by acute s)'rnptoms, which is
knorvn as acute herpetic gingivostomatitis. These symptoms include feveq initability,
cen ical lymphadenopathy, fiery red gingival tissues and small yellowish vesicles that
mprure and result in painful ulcers on the free and attached mucosa. The most serious potential problem in a child with this infection is dehydration due to the child not wanting
io eat or ddnk because ofthe pain. See picture #72 in booklet
The treatment is supportive and aimed toward the reliel of the acute symptoms so that
fluid and nutritional intake can be maintained. Primary herpetic gingivostomatitis usually runs a course of l2-20 days, and the ulcers heal without scaning.

\ote: Corticosteriods are contraindicated in patients with

herpes simplex infections.

v-B Dis

After the initial primary attack during the early childhood period,
the herpes simplex virus remains inactive most comrnonly in the:

. Geniculate ganglion

. Ciliary ganglion
. Trigeminal ganglion

. Pterygopalatine ganglion

166
Coplright O

20ll 1012

Denral Decks

ORAL PATHOLOGY

V-B Dis

A 49-year-old patient ofAshkenazi heritage presents to your oflice complalnlng


of "blisters in her mouth.' Your intraoral exam shows ulcers present on multiple
areas of mucosa. She also related to you that while getting out ofthe car earlier,
her skin ofher arm rubbed against the car door and tore. Known as Nikosky's
sign, this phenomenon is associated with which disease?

. Herpes zoster

. Lupus erythematosus
. Lichen Planus
. Pemphigus

t67
Copyright,e 201

l-:012

Dental Decks

The inactive herpes simplex virus resides in sensory nerve ganglia (mo.st commotll!', the
trigeminal gangliol, but will often reappear later as the familiar "cold sore," usually orr
the outside of the lips. This disease is referred to as "recurrent herpes labialis." Emotional stress, trauma and excessive exposure to sunlight have been implicated as factors
for the appearance of the recurrent herpetic lesions on the lip. Acyclovir 57o ointment
(Zovirax) as well as valacyclovir, and famciclovir have been successful in reducing the du-

ration and severity ofthese sores.

Remcmber:
Herpes Simpfex Type I (prinary; herpelic gingirostomLttiti.t, recuftent herpes labiis transmitted by direct contact. It affects the lips. face, skin and oral mucosa.
2. Herpes Simplex Type ll (herpe.s geuitalis/ is spread by sexual contact. It affects
I.

dlrt

the mucosa

ofthe genital and anal regions.

\ote: Cenital

herpes may have serious consequences in pregnant women because the


r irus can be tnnsmitted to the infant during vaginal delivery. The virus can cause damage to the infant's central nerr,'ous system and/or eyes.

Remember: The primary infection of herpes simplex can range from subclinical
IQs\tnptonqtic, v'hich is most common) to severe systemic infections.

Penrphigus (an autoimmune disorrler) is a term used to describe blistering ofthe skin caused
b1 binding ofantibodies to the surface ofthe cells ofthe outer layer ofthe skin, the epidermis. In pemphigus yulgaris, the most common form ofpemphigus. antibodies are directed
against rhe desmosomal adhesion molecule Dsg3, resulting in severe mucosal erosions and
eprdermal blistering in areas such as the mouth. As a result, patients develop severe oral ulcerations. and may also have inflammation or erosions of the lining of the eye and eyelids
I conitol(tiya), the nasal mucosa, or the genital mucosa. Half of the patients also develop b)isrers or erosions ofthe skin, often in the head and neck area. It seldom occurs before the age
oi j0 rususlly benteeu 30 and 50) and occurs more frequently in Jewish people.
There are four types ofPemphigus:
L Pemphigus vulga s: most common
L Pemphigus vegetans

3. Pemphigus foliaceus
4. Pemphigus erythematosus

first manifestation ofthe disease. Intact bullae are rarely seen in the
oral cavit,v. instead, large areas ofulceration and erosions are often seen that are covered by a
Oral lesions

are often the

u hite or blood+inged exudates. Sometimes, areas of epithelium will slide ofl simply by rubbing of an apparently unaffected area (this is 1e7-lt?ed Nikolsky's sign/. This disease is often

fatal without therapy, which includes high-dose systemic steroids or chemotherapy


drp1e, methotrxate/.

/br

e-r-

Important histological features: The vesicles and bullae are fbrmed entirely intraepithelially,
just above the basal layer of cells (calletl suprabasilar veslcleJi. There is intercellular edema
and loss ofintercellular bridges with loss ofcohesiveness. This is called acantholysis. Clumps
of cells are often found floating free in the vesicle space (tieve cells are called Tzanck cells).

. Candidiasis
. Hairy leukoplakia

. Desquamative gingivitis
. Hemorrhagic mass

168
Copt{ight O 20ll-2012 - Denhl Decks

. Human papillomavrrus (HPV)


. Adenovirus

. Epstein-Barr virus

. Parvovirus

169

Cop'-i8hI O

201 I

'2012 - Dental Decks

Benign mucous membrane pemphigoid (BMMP) is an autoimmune or "self-allergy" disease in which a patient's own circulating antibodies become altered so that they attack the
fibrous attachment of the skin and membrane epithelium to the underlying connective tissues. Women are more commonly affected than men and the disease is usually diagnosed
between the ages of 40-60 years. The typical lesion is a small or large, clear-fluid blister
which breaks fairly rapidly in the mouth to leave a flat white, somewhat tender ulcer with
a thin red line around it. The gingivae are especially likely to be involved, resulting in
sloughing during eating or tooth brushing ("desquamative gingivitis"). Systemic steroid
therapy has provided adequate management of BMMP Note: Conjunctival involvement
may lead to blindness.

Remembr: Nikolsky's sign, which is an indication ofpemphigus vulgaris, nay also be


lound in BMMP The sign occurs when apparently normal epithelium may be separated
at the basal layer and rubbed offwhen pressed with a sliding motion.

While similar in its clinical presentation to pemphigus vulgaris, it is much less severe
and the involved antibodies attack the attachment fibrils (Type VII collagen) ofthe basement membrane antigens fi.e., Laminin 5 and BP 180), rather than the desmosomal attachments (desmoglein 3 [Dsg3]

) between epithelial cells,

as occurs in pemphigus.

Important: Histologically, the major difference between BMMP and pemphigus r,ulgaris
is that the vesicles in BMMP are subepidermal and there is no evidence of acantholysis
in pemphigus vulgaris there is acantholysis and a suprabasilar vesicle,

This is the same virus that causes genital wans which lcads to dysplasia and cervical cancer. Some esti
mates ofthe prevalence of IIPV infection in the population range as high as 79%. Warts may arise on
anv skin surface, bnt occur most commonly at acrd (peripheral) sites. Although treatment may remove
rhe \\'art. the virus remains latent within the skin cells. However, treatrnent may diminish spread ofHPV
in the skin ofthe infected patient and possibly to uninfccted contacts.

. Papillomas:
. Benign epithelial prolifefttions (pedu culated or sessile) of \ftle significance.
. \trruca vulgaris (also called squamous papillona) has an incubation period from about six
\\'eeks to a ycar. Although it is primarily a lcsion ofthe skin, it may occur in the oral cavity, particularly on the lips and palate. Clinically, it is a sessile, soft, cauliflower-like lesion. Ifexcised, they
usually do not recur, but autoinoculation is possible. Note: Intraorally, that is how most cases dcvclop. Much more prevalent in HIV positive patients
. Condyfoma acuminatum (ge ilalwa s): Caused by HPV subtypes 6 and l1; oral lesions acquired
b) oral-genital contact; broad-based verruciform lesion.

. Focal epithelial hyperpl^si^ (Heck\ dise4se,): Most common in ethnic groups fi.e., Nat[re Artelicans ond Centtul Anlencdrt. Multiple small, dome-shaped warts on oral mucosa. Caused by HPV
subttpes 13 and 32.

Important: Sexually transmitted, high-risk HPVS include types 16, 18, 31, and 33. Tlese are the HPV
t] pes associated with cancer.

Verrucr vulgaris on the tongue

V-PLes

A 45-year-old female walks into your olfice complaining ofa "wart" on her
gums that has been thre for yars. Your exan reveals an asymptomatic,
well-circumscribed, slightly raised, papillomatous lesion on the buccal
gingiva of tooth #5. A likely diagnosis of this is:

. Fibrosarcoma
. Neurosatcoma

. Lipoma
. Verucifonn xanthoma

170
CopyriSlrr O

20ll

?01: - Denral Decks

ORAL PATHOLOGY

V-PLes

A 63-year-old completely edentulous patient comes into your office because


her dntures hrve "finally gottn too bothersome to wear.' Eer health history
consists of COPD and cigarette 40-year pack history. Your intraoral exam
reveals a noxious odor and an ill-litting upper denture. When you remove the
upper denture you note multiple red, papillary projections ofthe hard palate.
Your patint states she does not remoye her dentures at night or tretween meals.
After reviewing denture hygiene instructions, you give her the diagnosis of:

. Epulis fissuratum
. Papillary hyperplasia

. Nicotinic stomatitis
. Kaposi's sarcoma
171

CopriSht O 201l'?0ll ' Denlal Decks

i.s 45 t t'ars). Thc usual intraoral locations are the gingiva and alveolar mucosa, but any oral mucosal site may bc involvcd. Thc lesion appears as a rvcll-circurnscribed, slightly elevated mass with a papillomatous or vcrrucous surfacc.Thc
level of keratinization of the surface will influence its color. which ranses fiom \1hit(r or red.

This unique Iesion occurs in middle-aged individuals /alera.ge a.ge

Well-circunNcribed, lobulated, soil


yellowish mass; vessels visible over
suiface; buccal mucosa, floor of
mouth. long! common srtes

Rarely encountercd in soft tissues of


the head and neck. Yellowish submucosal mass. Slow-growing malignancy.
Adnlts (average age is ,l5r; gingiva
alveolar mucosa are most common
sites; well-circumscribed, slightly
ftised, papillomatous or verrucous
surface

When in bone,lesion
ma), arise fiom periostcum. endosteum or
PDL. When in sofl

Surgery or radiationi
prognosis is lair to good.

&

Rare sofl tissue and bony malignancy

Wide surgical excision;

ofthe head and neck. Young adults are reculTence not common
most commonly affected. Infi ltrative
neoplasm that is locally deslructive.

trssue. fibroblasts.

Eithcr from a
pre-existing lesion of
neurofibronlatosis or
dl: ,,o'r,. Thought to
be from Schwann cell.

Etiolo$

f,nrity
De.nire-induced fi brous
hlpcrplasia
, I t'll a|""at ory
b,peryla si a,
d?nhrc h\perplasia, attd

Rare malignancy; Pain or paresthesia


may accompany lesion in bone. ln sofl
tissues, it appgars as an expansile mass
that is usually asymptomatic

lll-fitting prosihesis

Widc surgical excision;


Recurrence is conmon,
metastases are fiequently
sccn. Prognosis is fair io
good

Location
Common lesion
that occuis in the

Clinical
Characteristics
I'ainless folds

offi'

overextended denture

'Epulis lissvtatum")

flange conects

Poor oral hyglene &


_Palatrl papillomatosis!') ill fitlinf prosthesis
r

Hard palate

Treatment

lrd

Prognosis

Surgical excision
prosthesis with
possible remaking

flange
relining dnturs

Papillar,v hyperplasia

\otes

Surgical cxcision

papillary projections.

l. There is no malignant potential to any ofthe librous hyperplasias.


fibroma is a reactive hyperplastic mass that occun in the gingiva and may be derived from connective tissue ofthe submucosa or the PDL. It
2. The peripheral

prcsents as well-demarcated focal mass with either a sessile or pedunculated base.


It is similar in color to the surrounding connective !issue. [t may be ulcemted. The
treatment for a peripheral fibroma is local excision. Recurrence is rare.
Note: Other variant forms ofthe peripheral fibroma include the peripheral ossiiying fibroma and the peripheral odontogenic fibroma.
3. Focal fibrous hvperplasia is hyperplasia oforal mucosa. Tt is also called tmumatic fibroma, irritation fibroma, and hyperplastic scar. lt is a reactive lesion
caused usually by chronic tmrrma to oral mucous membrane. See picture #23 in
booklet. The giant cell fibroma is a focal fibrous hyperplasia in which connective
tissue cells, many ofwhich are multinucleated, assume a stellate shape.

The picture below shows a benign epithelial neoplasrn which appears as a


pdunculated, whitish cauliflower-like mass on the lower lip.
The most likely diagnosis is:

172

coprighl

a(,

201 1-2011

- Dental Decks

ORAL PATHOLOGY

A 54-year-old African American female presents to your clinic for an initial


exam. She has a history of hypertension controlled with beta-blockers but no
other contributory health lindings, Your intral oral exam reveals a bilateral

lilmy opalescence ofthe buccsl mucosa. When stretching out hr cheeks,


this white hue disappears. Your most likely diagnosis is:

. Squamous cell carcinoma


. Fordyce granulation
. Leukoedema
. Leukoplakia

173
Cop)right C 20ll-2012 - Dental Decks

The paprlloma is a benign exophytic papillary growth of stratified squamous epithelium.


Note: The common wart, or verruca vulgaris, is a lrequent tumor of skin analogous to
the oral papilloma.

Clinical features:
. Adults
. Anywhere on the oral mucosa
. Sessile or pedunculated exophytic growth
. Papillary ftauliflowerJike) appearance
. Long duration
. May show considerable keratin and in some instances appear white clinically.
has little keratin on the surface it will appear pink.

If it

Treatmenl and Prognosis: Conservative excision: recurrence is rare.


Rememtrer: A fibroma is a benign neoplasm of connective tissue origin.

The appearance varies from a filmy opalescence of the mucosa in the early stages to a
more definite grayish-white cast with a coarsely wrinkled surface in the later stages. The
lesions usually occur bilaterally and are most noticeable along the occlusal line in the bicuspid and molar region. Diagnostically, one can stretch the tissue and the white essentiallv disappears. Important: Leukoplakia would not disappear when stretched.

Important point: Leukoedema appears to be simply a variant ofnormal mucosa and no


treatment is necessary; merely diagnosis. See picture #73 in booklet
The differential diagnosis should include:

. Leukoplakia
. Snuffpouch

(SP)

. Frictional keratosis

. \Vhite sponge nevus


. Hereditary benign intraepithelial dyskeratosis
Histologically, in leukoedema, the epithelium is parakeratotic and acanthotic, with marked
intracellular edema of spinous cells. Note: The white appearance ofleukoedema is caused
bv s ater within the spinous cells causing the light to reflect back as whitish.

Remember: Snuff pouch (SP) is a form of hyperkeratosis with various degrees ofclinical manifestation (1.e., white mucosal change). SP develops on those mucosal sites where
the tobacco is held. The causal agents ofSP are considered to be the nitrosamines and hydrocarbons contained in tobacco. Prolonged use ofthis habit may conduce to the devel-

opment of a squamous cell carcinoma due to the carcinogenic potential


components. See picture #77 in booklet

of

those

ORAL PATHOLOGY

WLes

A 67-year-old Caucasian male comes into your oflice for a routine check-up, He
relates to you that he just got back from Florida where he goes for the fall and
winter months. He enjoys taking his boat out with his wife. Your extraoral
exam shows chapped lips but his lower lip also presents with grayish-white
plaques. There is marked loss ofelasticity of the vermillion border.
Which of the following would you make your diagnosis:

. Actinic keratosis
. Actinic cheilitis
. Actinic dermatitis

Solar lentigo

174
Copyright O 20ll-2012 ' Denlal Decks

WLes

ORAL PATHOLOGY

An incisional biopsy is indicated for which of the following lesions?

. A.5 cm papillary fibroma ofthe gingiva


. A 2 cm exostosis ofthe hard palate
. A 2 cm area ofFordyce's disease ofthe cheek
. A 3 cm hemangioma ofthe tongue
. A 3 cm area of leukoplakia ofthe soft palate

175
Copyrighl

(l20ll ?012

DentalDecks

Actintc (solar) cheilitis, a variant oforal leukoplakia, is considered to be the labial counterpart of solar (actinic) kerctosts (a precursor of SCC of the skin). The lips appear dry
mottled, and opalescent with slightly elevated white or gray plaques and that cannot be
stripped off Isolated areas ofhyperkeratotic callus may also be evident as well as loss of
elasticity and definition ofthe vermilion border. See picture #74 in booklet
The short-term effects of exposure to UV light (especioll.v UVB, 2900 to 3200 wn) are
transient, but the cumulative long-term effects produce irreversible damage (actinic cheili/lr, usually to the lower lip ofexposed individuals.

Other clinical signs include:

. Eq.thematous or hemorrhagic areas


. Parallel marked folds
. An unobtrusive "chapped lip" appearance
. The junction of vermilion and skin becomes indistinct.

Malignant change is manifested clinically by areas of more diffuse cheilitis and ulcerations ofrelatively long duration. Although degenerative changes have been observed predominantly in men after the age 40, the condition now is increasingly recognized in
) ounger men.

Important: This condition is considered premalignant and may lead to squanous cell
carcinoma. It should be treated accordinslv.

Leukoplakia is a premalignant lesion. This means that ifleft untreated, some ofthe lesions progress
carcinoma. [t is bccause ofthis chance ofmalignant translbrmation that all leukoplakias should

r(r

be biopsied, Note: Leukoplakia is a clinical diagnosis.

. Idiopathic leukoplakia: refers

to white/opaque oral mucosa lesions that do not rub off and

are not clinically diagnostic for any other whitc lesions. The causc is unknown, however, tobacco
iitation (especialll pipel and alcohol may be contributing facto.s. [t is more common in older
men. lt is a slowly developing change in a mucous membrane characterized by thickened, white,
1irmly attached patches that arc slightly raised and sharply circumscribed. Lesions ofthe floor

of the mouth and base ofthe tongue are most aggressive. Most display no dysplasia but can
de\ elop into rnalignancy (5'% to I5o/A.ln all cases, leukoplakia must be completely excised
since diagnosis cannot be made clinically. See picture #75 in booklet

. Proliferative verrucous leukoplakia: it

is a high-risk form of leukoplakia The cause is unknorvn, although somc are associated with human papillomavirus 16 and 18. Lesions arc recuffent or persistent and usually multiple. Lesions may start llat but progress to broad-based'
v artllke (vermcforn) lesions. There is a high risk ofmalignant transformrtion to verrucous
carcinoma or squamous cell carcinoma.

Explanation oflesions on front ofcard:

. Papillary fibroma:

is a benign neoplasm ofconnective tissue ongin


Exostosis of the hard palate: lori (p!:tlatal or nandibulat): most cotr..rnon exophytic lesions,

slou-grou rng benign Lnots ofbone


. Fordyce's disease (or granules): ectopic sebaceous glands in the buccal mucosa and/or lip
They are present in over 757o ofadults. They usually appear as yellow, sometimes yellow white
submucosal clusters that are essentially normal. See picture #76 in booklet
Note: When mechanical irritation produces

white lesion it is called frictional keratosis.

ORAL PATHOLOGY

You are conducting a routine exam on a 54-year-old patient with


diabetes mellitus t'?e 2 and a 20-year pack history of smoking.
You see a white patch on left alveolar edentulous ridge. The lesion
cannot b wiped off and the patient denies a history of trauma
or allergies. Which ofthe following would be your diagnosis?

. Squamous cell carcinoma

. Lichen planus
. Erythroplakia
. Leukoplakia

175
Copyright

(i 201l':01:

- Dental Decks

ORAL PATHOLOGY

WLes

A 75-year-old patient comes to your olfice wanting a new set of dentures,


She hasn't ben wearing her old dntures for rbout 2 years. She has a
collapsed \aDO and her physician is concrnd rbout her iron deficiency.
The corners ofher mouth are fissured, dry, and erythernatous.
Which ofthe following conditions is the likely diagnosis?

Squamous cell carcinoma

. Angular cheilitis
. Vemrca vulgaris

Stomatitis nicotina

177
Copyrighr

e 20ll-:0ll

- Dcntal Decks

The etiology ofleukoplakia is thought to be a varied one. Possible etiologic tactors include
tobacco, aicohol, and oral sepsis. It is most often due to tobacco use. Some investigators
believe that pipe smoking is most harmful.

clinical white patch or plaque on the oral mucosa which will not rub off
and which cannot be characterized as any specific disease. Most reports indicate that
leukoplakia is more common in elderly men. Although less common than leukoplakias.
erythroplakias, have a much greater potential for becoming malignant.
Leukoplakia is

Important: Any white or red lesion that does not resolve itself in two weeks should be
reevaluated and considered for biopsy to obtain a definitive diagnosis.
The term carcinoma in situ is applied to mucosal lesions which resemble leukoplakia in
all respects xcept that dysplasia is very pronounced and involves almost all epithelial layers. It shows no tendency to invade or metastasize to other tissues.
The clinical differential diagnosis ofa white patch should include:

. Leukoplakia
. Lupus erlthematosus

. Candidiasis
. Lichen planus

. \\'hite

. Migratory glossitis/stomatitis
. Frictional hyperkeratosis

sponge nevus
-A. chen.rical / thermal bum

It is also associated with the loss of vertical dimension.This situation is generally obsen ed in elderly patients.The corners ofthe mouth become painful, iffitated, red, cracked,
and scaly. The fungus Candida albicans (thrush) may grow in the corners ofthe mouth,
keeping them sore. Note: It can also result from a bacterial (i.e., Staph)'lococcal) infec-

tlon.

It occurs in individuals that habitually lick their lips and deposit small amounts ofsaliva
in the commissural angles. It is also associated with nutritional deficiencies (i.e.. vitamin
B-) [riho.favinJ, vitamin B-3 [niacinJ, vitamin 8-6 [pvridoxineJ. or vitamin B-12

Io

anoc obalaminJ, or a deficiency in iron).

\ote: Nvstatin will invariably eliminate the fungal infection and an antibacterial is used
lo treat the bacterial inlection ifpresent.
See

picture #78 in booklet

. Hyperkeratosis
. Leukoplakia
. Epidermolysis bullosa

. White

sponge nerus

Coplrighr C

201

178
l'2012 - Dental Decks

Hairy tongue is a condition chrracterlzed by hypertrophy of the:

. Filiform papillae

. Fungiform papillae
. Circumvallate papillae
. Foliate papillae

179
Coptrighr O 201l-2012 - Dental Decls

White sponge nevus is an autosomal dominant trait due to mutations ofkeratin 4 and/or 13. tt
has no sex preference. lt presents as an asymptomatic. deeply folded, white or gray lesion
that may affect several mucosal sites. Lesions tend to be somewhat thickened and have a
spongy consistency. The presentation intraorally is almost a)ways bilateral and symmetric and
usually appears early in life, typically before puberty. The buccal mucosa is the usual location. See picture #79 in booklet

Important: There is no treatment for $,hite sponge nevus! howevel since the condition is
perfectiy benign, the prognosis is excellent. There are no serious clinical aomplications.
Note: lt is often mistaken for leukoplakia.

Nlicroscopically, a featufe that unique to prickle cells lbund in rvhite sponge nevus is perinuclear eosinophilic condensation of cytoplasm.
Note: Hyperkeratosis is an abnonnal increase in the thickness of the keratin layer (stratum
corneun) of Ihe epithelium. It is one ofthe most common white cheek lesions ofthe oral mucous rnembranes (often in an area of cht onic cheek biting) and presents as being thick and
scailr.

Epidermolysis bullosa is a general term that encompasses one acquired and several genetic
t ariet\es (dystrophic, .jtnctional, sizpler/ of disease that are basically chamcterized by the
lbnnation ofblisters at sites of minor trauma (especiall ot'er tlrc elbov and kleesl. Several
genetic types range from autosomal dominant to autosomal recessjve. The feature common to
a1l subtypes of epidermolysis bullosa is bulla formation from minor provocation, usually
or er areas ofstress, such as the elbows and knees. Oral lesions are panicularly common and
:er ere in the recessive forms and uncommon in the acquired form. These lesions include bullae. scaning. and hypoplastic teeth. These lesions are most pronounced in the type known as
recessi!e dystrophic epidermolysis bullosa.

Hain tongue is a benign condition ofthe tongue. The dorsum ofthe tongue appears furry
due to the elongated papillae. The color varies lrom yellowish-white to brown or black.
\ote: It is associated with poor oral hygiene, extended use ofantibiotics, cofticosteroids.
hr droven peroxide and smoking. See picture #80 in booklet
The four trpes of papillae present on the tongue:
1. Filiform: most numerous, small cones aranged in "V"- shaped rows paralleling
the sulcus terminalis. They are characterized by the absenc oftaste buds and increa-

keratinization.
l. Fungiform: scattered among the filiform papillae, they are flattened, mushroom
shaped and found mainly at the tip and lateral margins.
,'V',- shaped
-1. Circumyallate: iargest, have circular shape. Arranged in an inverted
rou tolard the back ofthe tongue. Associated with the ducts ofvon Ebner's glands.
Thev are the least numerous of all papillae.
l. Foliat: found on lateral margins as 3-4 vertical folds.
sed

. Lupus erylhematosus
. Erythema multiforme

. Pemphigus vulgaris
. Lichen planus
lace like

180
Cop)Tiglt O 2011-2012 Dental Decks

. Leukoplakia
. White sponge nelus

. Candidiasis
. Lichen planus

t81
CoplriSht O 201 I,20

12 -

Dmral Dcls

erosive

These lacelike white striae, the so-called Wickham's striae, are a classic presentation
lichen planus. They are often bilateral and symmetrical in distribution.

of

Lichen planus is a fairly common inflanmatory disease that usually affects the skin,
the mouth, or sometimes both. It aflects women slightly more than men, and occurs most
often in middle-aged adults. TJymphocytes destroy basal keratinocytes, however. the reason for this immunologically mediated phenomenon is unknown. Lichen planus of the
mouth most commonly affects the buccal mucous membrane. It may also been seen on
the tongue, lips, hard palate and gingiva. The lace-like striae are usually asymptomatic,
but sometimes the patient may complain ofa burning sensation. The intraoral lesions respond to topical steroid therapy. In addition to the usual form of lichen planus, there are
lrvo other foms, bullous and erosive. [n the bullous forrn, fluid-filled vesicles project
from the surface. [n the erosive form, the lesions are intensely red or raw-appearing.
When drese lesions of erosive lichen planus involve the gingiva, they resemble desquamative gingivitis. See picture #81 in booklet

The microscopic appearance of lichen planus is characteristic and pathognomonic:


. Hyperparakeratosis with thickening ofthe granular cell layer

. Deyelopment ofa "salv tooth" appearance ofthe rete pegs


. Degeneration ofthe basal Iayer ofcells
. lnilltration of inflammatory cells into the subepithelial layer ofconnectrve

tissue

Oral candidiasis also kaown as thrush, causes white, curd-like patches in the mouth or
lhroat. These patches typically appear on the tongue, inside ofthe cheeks, or on the palate.
Oral candidiasis typically occurs in people rvith abnormal immune systems. These can
include people undergoing chetrotherapy for canceq people taking immunosuppressive
dmgs to protect transplanted organs, or people with HIV infection.
Candidiasis is caused by a yeasrlike fungus, C. albicans, rvhich causes an inflammatory,
pruritic infection characterized by a thick, white discharge. Acute lesions appear as diftirse. curly or velvety white mucosal plaques on the cheeks. palate and tongue rhat car be
lriped off, leaving a red, raw or bleeding surface. Note: Chronic lesions are erythematous.

This veast-like fungi is a normal inhabitant of the oral cavity and vaginal tract, however
it is normally held in check by the indigenous bacteria of these areas. The treatment for
oral candidiasis is topical through the use of lozenges (also called trouc}esl and mouth
rinses. the most widely used is nystatin, Note: Systemic treatment includes the use of
lluconazole or ketoconazole.

Acute pseudomembranous candidiasis is the most common fom of oral


candidiasis and is usually found on the buccal mucosa, tongue and soft palate.
....r. Oral cytology smears are usel'ul for a diagnosis (it will reveal budding otgqnisnts *ith branching pseudohyphae).
2. Angular cheilitis (Perleche) has also been linked to C. albicans.
3.Factors that may stimulate Candida growth include: the extended use ofantibiotics, steroids, diabetes, pregnancy, or a deficiency in iron, folate. Mtamin
Brr or zinc.

\otes'

ORAL PATHOLOGY

WLes

The "white patch" seen below has been present for nine months on a patient
who is a heavy pipe smoker. What is the treatment of choice?

142
CopyriShr C 2011

20ll

Denral Decks

WLes

ORAL PATHOLOGY

A 3s-year-old healthy female prsents to your office for routine dental work
While completing the restorativ treatment on the LL quadrant you notice that
her tongu has multiple irregularly shaped red lesions that have a white trorder.
You make a note in your chart. When she returns two weeks later for the
restorative work on the LR quadrant, you note that there are still lesions,
but in different locations on the tongue with different shapes.
What is your diagnosis?

. Fissured tongue
. Macroglossia
. Geographic tongue
. Hairy tongue
183
Copvriglrt

a):0ll-l0ll

- Dental Decks

Clinically. this is leukoplakia and should ahvays be biopsied due to the possibility ofit being a
premalignant lesion. Remember: Pipe smoking is thought to bc one of the nrost impona nt predisposing ctiologic factors for thc dcvclopmcnt ofa lcukoplakia.

Important: The floor ofthe moutir,

tongue and lower lip ale the regions at greatcst risk for car-

cinoma occurring in leukoplakia.

Stomatitis nicotina is related to pipe smoking (as uell as cigor snrokfugl and occurs exclusively
on thc palate. It aff'ccts malcs prcdominantly. The palate first appea$ rcd and inflamqd. Soon it
develops a diff-usc, grayish-white, thickened, multinodular popular appearance rvith a small red
"spot" in thc ccntcr ofeach tiny nodule. This "spot" corrcsponds to the orifices ofpalatal salivary gland ducts. Thc treatmcnt for this condition is the cessation ofslnoking. It is usually not
considcrcd to bc a prcmalignant lesion.
See picture #84 in booklet

\ote: In India,

thc usc ofsmokclcss tobacco in various forms is vcry popular. This habit, which
usually involves the chewing ofa betel q\id (combined areca nut, betel le.i, tohacco dnd slack
lindl. has lcd to thc dcvelopment, in a large proportion ofuscrs. ofa unique generalized fibrosis
of thc oral soft tissues, called oral submucous fibrosis. It typically affects the buccal mucosa.
Lips. rctromolar arcas and thc soft palatc. Early lcsions prcscnt as a blanching of the mucosa,
rmpaning a mottlcd, marble-like appcarance. Latcr lesions dcmonstrate palpablc fibrous bands
running vertically in the buccal mucosa and in a circular fashion around the mouth opening or lips.
As the disease progresses the mucosa becomes stiff, causing difficulty in cating and considcrably
rcstricting thc paticnt's ability to open the mouth (risnl ir,r/. Ifthc tongue is involvcd. it becomes
sritTand has a diminishcd sizc. Thqre is a fibroclastic transformation of thc.iuxta-epithclial connecti\ e tissues and an increased risk of oral carcinoma liom the tobacco ofthe quid.

ImDortant: Betel quid and smokcless tobacco also increascs the risk ofvcrrucous carcinoma.

***

lr rs also called eD4hema migrans. benign migratory glossitis, rvandcring rash ofthe tongue, erythema
lreara migrans. and stomatitis arcata migrans.

Grographic tongue is a hannless and very common condition in u'hich there is dcsquamation ofthe filiform papillae. h is charactcrizcd by having one or more iregularly shapcd patches on thc longuc. Thc
a-r.r arca is redder lhan the rcst of the tongue and the edges of the patch are whitish in color Thcsc
r:rchcs eppcar and remain for a short time. heal. then reappear aI anothcr site. The patchcs usually do
rrrr r.sprrnd to reatment but disappcnr spontaneously. Note: The patient may complain ofa slight burnr:1g ofrhe tonguc. Se picture #82 in booklt
Fis!ured tongue /a/.f.r tttlleclsctotultongue) is charactcrized by a deep median fissure wilh laterally ra.iratrns srrrovcs. Thc lateral groovcs vary in number but are usually symmctrical in arrangcmcnt. It is rarc
r:r .illdrcn and increascs in incidence with age. The fissuring occurs across the dorsum ofthe tonere
:r.J rs u'ua11r' asyrnptomatic. but may bccomc painful if infccted with Candida Albicans.
See Dictur #83

\ote:

in booklet

Fissurcd tongue is lbund in NlelkerssorFRosenthal

Syndrome (dlong

ilh gtdnulonatous cheili-

t:t and liaial ner\e parallsis).


Remember: Gcographic tongue often occurs in association \\'ith fissured tonguc

\lacroglossia is tonguc enlargement that leads to ftrnctional and cosmctic problenls. Although this is a
relarivcl-v uncommon disorder. it may causc significant morbidity. Thcrc is no clear definition of
macroglossia and it may bc dcfined in relative, funclional, or structural terms. Causes includci
. An inflammatory cause is chronic glossitis
. Traumatic causes includc postoperative edema

. \Ietabolic

causes are myxedcma, amyloidosis, lipoid proteinosis. chronic steroid therapy, and
acromegal)
. The congenital causes are p n1ary idiopathic rnacroglossia. cretinism, hemangioma, lymphangioma, Beckwith-Weidmann Syndrome, Dorvn Syndrome, generalized gangliosidosis syndrome,
and anv of the MuooDolvsaccharidoses.

Das könnte Ihnen auch gefallen